2ch勢いランキング アーカイブ

■ちょっとした物理の質問はここに書いてね223■


ご冗談でしょう?名無しさん [] 2018/01/27(土) 21:56:28.22:JeYgjzqM
★荒らし厳禁、煽りは黙殺
★書き込む前に      の注意事項を読んでね
★数式の書き方(参考)はこちら      (予備リンク:       )
===質問者へ===
重要 【 丸 投 げ 禁 止 】


・質問する前に
1. 教科書や参考書をよく読む
2.
ttp://www.google.com/
  などの検索サイトを利用し、各自で調べる
3. 学生は自分の学年、物理科目の履修具合を書く
4. 宿題を聞くときは、どこまでやってみてどこが分からないのかを書く
5. 投稿する前に、ちゃんと質問が意味の通る日本語か推敲する、曖昧な質問文には曖昧な回答しか返せない
・「力」「エネルギー」「仕事」のような単語は物理では意味がはっきり定義された言葉です、むやみに使うと混乱の元
・質問に対する回答には返答してね、感謝だけでなく「分からん」とかダメOK
・質問するときはage&ID表示推奨
・高度すぎる質問には住人は回答できないかもしれないけれど、了承の上での質問なら大歓迎

===回答者へ===
・丸投げは専用スレに誘導
・不快な質問は無視、構った方が負け
・質問者の理解度に応じた適切な回答をよろしく
・単発質問スレを発見したらこのスレッドへの誘導をよろしくね
・逆に議論が深まりそうなら新スレ立てて移動するのもあり
・板違いの質問は適切な板に誘導を
・不適切な回答は適宜訂正、名回答は素直に賞賛

前スレ
■ちょっとした物理の質問はここに書いてね222■
ttps://rio2016.5ch.net/test/read.cgi/sci/1512135640/
ご冗談でしょう?名無しさん [sage] 2018/01/27(土) 23:58:12.05ID:???
削除依頼を出しました
ご冗談でしょう?名無しさん [sage] 2018/01/28(日) 00:50:13.03ID:???
でぼぼぼーん
ご冗談でしょう?名無しさん [] 2018/01/28(日) 08:26:00.27:EjWMDsen
リンゴが赤く見えるのは赤以外の光が電子(やポラリトン)励起後戻ってくる際のエネルギーが
格子エネルギーや分子熱運動に使われた後黒体放射されるのに対し、
(電子など励起後に生じるエネルギーであって、直接光エネルギーがこれらに変換
されるわけではないんですよね?)
赤の光はまんま電子など励起後光として放射されると...?
ご冗談でしょう?名無しさん [sage] 2018/01/28(日) 13:12:38.00ID:???
分かるように書け
なんで黒体放射?
ご冗談でしょう?名無しさん [sage] 2018/01/28(日) 13:47:26.32ID:???

ハリボテも良いところだから
そこまで気になるなら自力で物性の勉強すべき
ご冗談でしょう?名無しさん [] 2018/01/28(日) 14:59:36.47:8vAX/f9b
熱力学系の問題です。解ける方いませんか?
教えてください


ttps://i.imgur.com/t75cseW.jpg
ご冗談でしょう?名無しさん [sage] 2018/01/28(日) 16:54:46.28ID:???
エネ管みたいだ
ご冗談でしょう?名無しさん [sage] 2018/01/28(日) 17:04:32.03ID:???
いません
ご冗談でしょう?名無しさん [sage] 2018/01/28(日) 17:40:38.33ID:???
これは物理じゃなくて化学だな
ご冗談でしょう?名無しさん [sage] 2018/01/28(日) 20:33:38.65ID:???
ムニちゃーんポッポ
ご冗談でしょう?名無しさん [sage] 2018/01/28(日) 20:42:31.17ID:???

「解ける方」は沢山居るよ。
「教えてくださる方」が居るかどうかだろ。
ご冗談でしょう?名無しさん [sage] 2018/01/28(日) 22:56:29.12ID:???

へぇ、何の教科書なの?
ご冗談でしょう?名無しさん [sage] 2018/01/28(日) 23:47:35.76ID:???
前スレ976のひとまだいる?
・光線行列使った解法 ・作図での解法
高校物理のしばりがあると後者なんだろうけど書くのがしんどい
前者だけでいい?
ご冗談でしょう?名無しさん [sage] 2018/01/29(月) 12:19:55.46ID:???
1/a+1/b=1/f は習うんじゃないの?
ご冗談でしょう?名無しさん [sage] 2018/01/29(月) 17:23:55.99ID:???

ゴミレスだけど
この式って中3の数学の相似のときに出てくる式そのものだね
今初めて気づいたw
ご冗談でしょう?名無しさん [sage] 2018/01/29(月) 21:04:24.91ID:???
ゴミレスだと思うなら書き込まなければいいのに
ご冗談でしょう?名無しさん [sage] 2018/01/30(火) 02:40:18.59ID:???
しんぷさんもムーニーちゃん
ご冗談でしょう?名無しさん [sage] 2018/01/30(火) 12:56:33.96ID:???

ゴミスレにはゴミレス
ご冗談でしょう?名無しさん [] 2018/01/31(水) 01:32:18.45:wSbSxTSh
回転座標系での見かけの力の質問です
(コリオリの力)+(遠心力)=向心力になる意味がわかりません
遠心力とは向心力と釣り合わせるための見かけの力では?
ご冗談でしょう?名無しさん [sage] 2018/01/31(水) 02:49:20.68ID:???
ちがわい ちがわい
ご冗談でしょう?名無しさん [sage] 2018/01/31(水) 10:39:13.32ID:???

>遠心力とは向心力と釣り合わせるための見かけの力では?
いや、一般に、遠心力と向心力は釣り合わないよ。
なぜなら、もし両者が釣り合ったら全ての物体は等速直線運動をする筈だが、実際にはそうなっていない。
ご冗談でしょう?名無しさん [sage] 2018/01/31(水) 13:09:49.41ID:???
ttps://i.imgur.com/fyGQkRL.jpg
この問題がさっぱり分かりません
(ア)は-198.3J/K
(イ)は僭=僣-T儡よりつじつまが合ってるでよろしいのでしょうか?
ご冗談でしょう?名無しさん [sage] 2018/01/31(水) 15:48:53.22ID:???
ボーズアインシュタイン凝縮ってさ、低温では基底状態になりやすいのと、ボゾンは同じ状態を取れることから自明な気がしてならんのだが
何か見落としてるの?
ご冗談でしょう?名無しさん [sage] 2018/01/31(水) 16:39:25.64ID:???

U(1)が破れるのが本質
ご冗談でしょう?名無しさん [] 2018/01/31(水) 18:28:54.22:SwewUJzv
縦笛の左手の親指で押さえる穴を半分開けると
なんで1オクターブ音が高くなるの?
(´・ω・`)
ご冗談でしょう?名無しさん [] 2018/01/31(水) 20:43:10.24:cwSpCh2B
トレッドミルと、慣性を使った路上での定速走行では空気抵抗以外の違いがありますか?
ご冗談でしょう?名無しさん [] 2018/01/31(水) 23:05:57.07:cwSpCh2B
自分でググっても、慣性系にあるので路上もトレッドミルも同じという意見と
蹴る力がいらないので全く違うという意見がありどちらがただしいのか全くわかりません
実際のとこどうなんでしょうか?
ご冗談でしょう?名無しさん [sage] 2018/02/01(木) 00:06:17.70ID:???

そんなことないだろw
ご冗談でしょう?名無しさん [sage] 2018/02/01(木) 09:12:32.37ID:???

じゃぁ、どうして等速直線運動をしないで円運動をするのさ?
ご冗談でしょう?名無しさん [sage] 2018/02/01(木) 09:33:03.75ID:???

回転座標系上では円運動なんてしてないよ
ご冗談でしょう?名無しさん [sage] 2018/02/01(木) 09:48:59.66ID:???

トリプルアクセルしてみろ。自分以外の全世界が回転運動するぞ。
ご冗談でしょう?名無しさん [] 2018/02/01(木) 10:14:47.56:lPRYM3Lx
自分なりに考えてみたんですが、路上だと空気抵抗無視しても路面との接触で慣性がだんだん減っていって定速を維持するためにはその都度加速が必要になる気がするんですが
ルームランナーやドレッドミルだと路面自体が減速しないので足を送るだけで済む分エネルギーが少なくてすむ気がするのです
路面で言えば下り坂で足を動かしてるような感覚ですね
実際はルームランナーでも加速は必要になってるんでしょうか?
その辺がよくわかりません
ご冗談でしょう?名無しさん [] 2018/02/01(木) 10:16:34.18:lPRYM3Lx
あ、すいません。28=33です。よろしくお願いします
ご冗談でしょう?名無しさん [] 2018/02/01(木) 10:33:09.51:CBFyeo4N
やりすぎ防犯パトロール、特定人物を尾行監視 2009年3月19日19時7分配信 ツカサネット新聞
ttp://headlines.yahoo.co.jp/hl?a=20090319-00000026-tsuka-soci

この記事で問題になった通称やりすぎ防パトは、創価学会と警察署が引き起こしていたようです

掻い摘んで説明すると

・創価学会は、町内会や老人会、PTA、商店会等の住民組織に関し、学会員が役員になるよう積極的に働きかける運動を
 90年代末から開始し、結果、多くの住民組織で役員が学会員という状況が生まれた

・防犯パトロールの担い手は地域の住民と住民組織で、防犯活動に関する会議や協議会には、住民組織の代表に役員が出席する為
 防犯活動や防パトに、創価学会が間接的に影響力を行使可能となった

・防パトは住民が行う為、住民が不審者や要注意人物にでっち上げられるトラブルが起きていたが
 創価学会はその緩さに目をつけ、住民組織を握っている状況を利用し、嫌がらせ対象者を不審者や要注意人物にでっち上げ
 防パトに尾行や監視、付き纏いをさせるようになった

・防パトは地元警察署との緊密な連携により行われる為、創価学会は警察署幹部を懐柔して取り込んでしまい
 不審者にでっち上げた住民への嫌がらせに署幹部を経由して警察署を加担させるようになった

・主に当該警察署勤務と考えられる創価学会員警察官を動かし、恐らく非番の日に、職権自体ないにもかかわらず
 私服警官を偽装させて管轄内を歩いて回らせ、防犯協力をお願いしますと住民に協力を求めて回り
 防犯とは名ばかりの、単なる嫌がらせを住民らに行わせた(防犯協力と称し依頼して回っていた警察官らの正体は恐らく所轄勤務の学会員警察官)
 ※これに加えて防犯要員が同様のお願いをして回る

・こうして防犯パトロールを悪用し、住民を欺いて嫌がらせをさせつつ、創価学会自体も会員らを動員し、組織的な嫌がらせを連動して行った

つまり警察署に勤務する学会員警察官、警察署幹部、創価学会が通称やりすぎ防犯パトロールの黒幕

詳細は下記スレをご覧下さい
やりすぎ防犯パトロールは創価学会と警察署の仕業だった
ttps://rio2016.5ch.net/test/read.cgi/bouhan/1516500769/
ご冗談でしょう?名無しさん [sage] 2018/02/01(木) 12:58:46.84ID:???
やたらとコピペ貼る奴など信用できん
ご冗談でしょう?名無しさん [sage] 2018/02/01(木) 13:15:37.61ID:???

ルームランナーでは摩擦で後ろに加速されるから
位置を保つためにエネルギーを消費する
ご冗談でしょう?名無しさん [] 2018/02/01(木) 16:42:55.61:XntWHYEf
体重50sのスキープレイヤーが高さ30mの所から滑り始める。高さ20mのときの速さを求めよ。また、が20m/sで滑り降りている地点の高さも求めよ。
重力加速度を9.8とする

教えて下さい
ご冗談でしょう?名無しさん [sage] 2018/02/01(木) 19:59:19.23ID:???

有限温度で2次転移があるのは非自明じゃね?
2次元では有限温度でボースアインシュタイン凝縮おこさんし。
ご冗談でしょう?名無しさん [sage] 2018/02/01(木) 20:08:16.28ID:???

単発質問スレの削除依頼を出せ
ご冗談でしょう?名無しさん [sage] 2018/02/01(木) 20:51:15.05ID:???

どうでも良いけど3次元理想ボーズ系なら3次相転移
ご冗談でしょう?名無しさん [sage] 2018/02/01(木) 20:54:56.60ID:???
コンプトン効果がわかりません

電子に光子が衝突することで、光子の波長が変化する。
従って光子は運動量とエネルギーを持ち粒子としての性質も持ち合わせる。
実験によって確かめられていることですし
光子が運動量とエネルギーを持つということは納得してます。

しかし光子はm=0なので電子と衝突をしても
電子の運動を変化させることが出来ないように考えてしまいます。
(ボールとボールが衝突してお互いの運動が変化するイメージで考えています)
質量が0の粒子の衝突というのは、どのようなものとイメージしていますか?
ご冗談でしょう?名無しさん [sage] 2018/02/01(木) 21:01:52.72ID:???

光子が運動量を持つことを認めながら、なぜ光子が電子の運動を変化させることができないと思うのか?
ご冗談でしょう?名無しさん [sage] 2018/02/01(木) 21:02:52.05ID:???
質量に起因しない運動量というものが確かに存在している
という観測事実を認められるか認められないか、という話ですね。
古典力学の範囲にとどまっていれば、そんなものはあり得ないと思うのは当然です。
そこから一歩踏み出して現代物理学が建築されたわけです。
ご冗談でしょう?名無しさん [sage] 2018/02/01(木) 21:28:16.84ID:???
山口人生は天才ですか?
ご冗談でしょう?名無しさん [] 2018/02/01(木) 22:01:34.67:APfSR7Sl
ありがとうございます
ご冗談でしょう?名無しさん [] 2018/02/02(金) 02:22:45.94:xfMtb435
コンプトン効果の実験より
光は粒子性がある光子と確定した。
電子は結晶を使った実験で干渉し
波動性がある(物質波)と確定した。
ご冗談でしょう?名無しさん [sage] 2018/02/02(金) 09:00:22.85ID:???
質問です。
空気中の熱量は湿度によって違いますか?
ご冗談でしょう?名無しさん [sage] 2018/02/02(金) 11:08:46.72ID:???

熱量が何を意味するか不明だが
工学的には湿り空気は乾き空気と水蒸気の混合物と考える
従って熱的性質は湿度によってかなり大きく変わる
ご冗談でしょう?名無しさん [sage] 2018/02/02(金) 12:28:08.88ID:???
潜熱の意味じゃない?
ご冗談でしょう?名無しさん [sage] 2018/02/02(金) 14:43:04.47ID:???

久々にその名前を見た
ご冗談でしょう?名無しさん [sage] 2018/02/02(金) 15:42:40.49ID:???
この世とあの世はどっちの方が広いですか?
ご冗談でしょう?名無しさん [sage] 2018/02/02(金) 19:17:18.05ID:???
その世が広いです
ご冗談でしょう?名無しさん [] 2018/02/02(金) 19:26:50.49:OFz1mMBT
上に上級剣聖神癒し癒し回復食を3回繰り返し、
上に上級剣聖神癒し癒し回復食を3回繰り返し、
下に上級剣聖神癒し癒し回復食を3回繰り返し、
上級剣聖神癒癒し回復食に、
に聖神癒し癒し回復食と、
に聖神癒し癒し回復食を
緑のキレイトと聖神癒し癒し回復食を入れて混ぜて、食べる。
若返りの常食。
登記 👀
Rock54: Caution(BBR-MD5:0be15ced7fbdb9fdb4d0ce1929c1b82f)
ご冗談でしょう?名無しさん [sage] 2018/02/02(金) 20:01:16.53ID:???

熱というのはエネルギーが移動する時に定義される物理量で、状態量ではありません。
ご冗談でしょう?名無しさん [sage] 2018/02/02(金) 21:28:06.55ID:???


ありがとうございます。
ご冗談でしょう?名無しさん [] 2018/02/02(金) 21:53:20.48:9MkHsZ1x
質問です。

さっき頭脳王っていう番組で、「太陽の210億倍の質量を持つブラックホールの直系を求めよ。」っていう問題があったのですがどうやって解いたのかわかりますか。

ちらっと写った条件には確か、太陽の質量と万有引力定数Gと光速度cが与えられていました。
ご冗談でしょう?名無しさん [sage] 2018/02/02(金) 21:55:23.17ID:???
シュバルトシルト半径でググりましょう
ご冗談でしょう?名無しさん [] 2018/02/02(金) 22:19:17.39:OFz1mMBT
集団ストーカーの攻撃を無効化するジャマーなどの機械、集団ストーカーを電磁波攻撃する機械、ttp://www.amazon.co.jp/%E8
%B6%85%E9%9F%B3%E6%B3%A2%E6%8E%A2%E7%9F%A5%E6%A9%9F-%E3%82%B3%E3%82%A6%E3%83%A
2%E3%83%AA%E8%BF%BD%E8%B7%A1-%E9%9B%BB%E7%A3%81%E6%B3%A2%E6%94%BB%E6%92%83-%E9
%9B%86%E5%9B%A3%E3%82%B9%E3%83%88%E3%83%BC%E3%82%AB%E3%83%BC-%E3%83%86%E3%8
2%AF%E3%83%8E%E3%83%AD%E3%82%B8%E3%83%BC/dp/B0771HCG2Z

これがテクノロジー犯罪機器だ(写真付き)
ttp://ameblo.jp/sync-one4all/entry-12307484098.html 

電磁波攻撃に電磁波攻撃で反撃して身を守るという方法があります。
ttp://sites.google.com/site/gangstalkinghksssyk/damage_and_countermeas
ure/damage_measure_goods

集団ストーカー 攻撃
ttp://search.yahoo.co.jp/search?p=%E9%9B%86%E5%9B%A3%E3%82%B
9%E3%83%88%E3%83%BC%E3%82%AB%E3%83%BC+%E9%81%93%E5%
85%B7+%E6%A9%9F%E5%99%A8+%E6%94%BB%E6%92%83%E3%80%
80%E8%B2%A9%E5%A3%B2&aq=-1&oq=&ai=bMnK4eKkQCenEHxdtEilFA&ts=
1984&ei=UTF-8&fr=top_ga1_sa&x=wrt

広めろ
ご冗談でしょう?名無しさん [sage] 2018/02/02(金) 23:12:18.14ID:???
熱影響に詳しい人助けて。

100℃の物体から10cm離れた場所で3秒間晒されるとどの位の熱量を受けることになるんですか?何の法則が当てはまるの?
ご冗談でしょう?名無しさん [sage] 2018/02/02(金) 23:17:07.60ID:???
小山幸司 弘前 犯罪 強姦 危険ドラッグ オウム真理教 密売 詐欺
小山幸司 弘前高等学校 共産党 革マル テロ 変態 偽造 殺人未遂 経歴詐称
小山幸司 弘前学院大学 淫行 性癖 児童ポルノ 親 殺し 差別 長南
小山幸司 弘前大 窃盗 東奥日報 掲載 盗撮 ストーカー
ご冗談でしょう?名無しさん [sage] 2018/02/03(土) 00:12:00.88ID:???

シュテファン=ボルツマンの法則
ご冗談でしょう?名無しさん [] 2018/02/03(土) 00:32:05.30:GP2wtaO6
ttps://i.imgur.com/5OW4BWL.jpg

このオがあまり理解できてないから誰か教えてください。
音速vsで移動する観測者に対する電波の速さをc-vs≒cって近似するときf-Δf1≒fって近似できない?解答だとf-Δf1はそのままで(f-Δf1)λ=cとしてλを求めてるんだけど。
ご冗談でしょう?名無しさん [] 2018/02/03(土) 02:32:39.83:O3xLaqcs
>このオがあまり理解できてないから誰か教えてください。
>音速vsで移動する観測者に対する電波の速さをc-vs≒cって近似するときf-Δf1≒fって近似できない?解答だとf-Δf1はそのままで(f-Δf1)λ=cとしてλを求めてるんだけど。

ふんふん。
この問題は相対論入ってるだろ。

「速さが光速より十分小さいときにはドップラー効果の関係式は音波の場合と同じ」というのは
そうでない場合には相対論効果が入ってくるってことの裏返しであり、
そう言いながら「上昇する観測者に対しても光速cに等しい」ってのは
まさに相対論の仮定である光速不変を使ってんだよなあ。

つまり、
ドップラー効果は「光速不変を捨てて」音波のときのように考えて周波数を求め、
しかし観測者が見る波長を求めるには「光速不変で考えよ」という
相反する分けの分からん相対論がデタラメであるがゆえの無理じいをしている問題だわこれ。

こんな相対論が入ってる問題を高校生に出していいのかおい。
ええ加減にせえよアホンダラー

くっくっく
ご冗談でしょう?名無しさん [] 2018/02/03(土) 02:42:43.05:O3xLaqcs
ってことで

・光速は近似をしていない。光速不変というアホノシュタインの妄想を使う。
 どんな速度の観測者が見ても同じcだ。
・しかしながら、観測者が受ける光の振動数はドップラー効果で変化するとして
 音波のように(つまり光速は不変ではない)として計算せよ。

という、高校生には混乱をもたらす愚問だよ愚問。
よくもまあこんなしょーもない問題出すよな。

要は
「オレっち相対論知ってるぜお前ら混乱して試験落ちろやー」
って異常者が出題したと思っておけばいいだろ。

くっくっく

この2つは相反する考え方なので
相対論がいかにデタラメかの見本のようなしょーもない問題ってことだ。
ご冗談でしょう?名無しさん [] 2018/02/03(土) 02:59:05.19:O3xLaqcs
でな、
どこの高校物理でこんなこと定量的に教えてるんだ?

「速さが光速より十分小さいときにはドップラー効果の関係式は音波の場合と同じ」

定量的に教えていないことをこんなふうに条件みたいに付加して
問題に紛れ込ませていいわけがない。
なぜなら、その条件の意味を解答者側は高校物理の範囲内で考えようとしてしまうからだ。
あとの解答に悪影響を及ぼすのである。

光のドップラー効果の式なんか高校物理で導出するわけないから
大学入試問題であるなら完全に逸脱している。

アホノシュタインのデタラメ相対論を混ぜてる時点で
何もかもあり得んわー

くっくっく
ご冗談でしょう?名無しさん [] 2018/02/03(土) 03:07:49.03:O3xLaqcs
>さっき頭脳王っていう番組で、「太陽の210億倍の質量を持つブラックホールの直系を求めよ。」っていう問題があったのですがどうやって解いたのかわかりますか。

ガクトと同じやらせに決まってるから
気にすんな。

仮想通貨の事件でも分かったろ?、出川のCMな。
カネのためならどんな会社のCMでも流す。

視聴率のためならイケメンにやらせをさせる。
ガクトやらなんやら。

世の中、イケメンは最優遇だから
テレビ見て真に受けんなよ。すべてはやらせだからな。

くっくっく
ご冗談でしょう?名無しさん [sage] 2018/02/03(土) 03:20:27.54ID:???
シュバルトシルト半径も知らないんですか?
ご冗談でしょう?名無しさん [sage] 2018/02/03(土) 07:01:18.18ID:???
くっくっくきもちわるい
ご冗談でしょう?名無しさん [sage] 2018/02/03(土) 11:42:12.74ID:???
くっくっくは相間
ご冗談でしょう?名無しさん [sage] 2018/02/03(土) 11:48:32.73ID:???
自殺をしたら地獄に落ちるのでしょうか?
ご冗談でしょう?名無しさん [sage] 2018/02/03(土) 11:53:49.90ID:???
早く死ねよ、口先だけのヒマラヤ、レス乞食
ご冗談でしょう?名無しさん [] 2018/02/03(土) 13:18:14.23:uERQOa0N
人間が絶滅したときに起こる環境への影響(間接的なもの含む)を教えて下さい
物理じゃなくてすいません
ご冗談でしょう?名無しさん [] 2018/02/03(土) 13:21:26.79:ABo31Alu
【沖縄に謎の発光体!?】.....1月27日、与那原でUFO撮影.....【マイトLーヤの星か?】
ttp://rosie.5ch.net/test/read.cgi/liveplus/1517624262/l50
ご冗談でしょう?名無しさん [] 2018/02/03(土) 13:27:21.97:3RvQLbCY
ポルチンスキーが死んだ
ttps://chancellor.ucsb.edu/memos/?2.2.2018.Sad.News...Professor.Emeritus.Joseph.Polchinski
ご冗談でしょう?名無しさん [] 2018/02/03(土) 13:56:31.19:crgU/YfT
光速で動いてる場所から進行方向にボール投げたらどうなりますか?
漠然としててすみません
ご冗談でしょう?名無しさん [sage] 2018/02/03(土) 14:47:13.00ID:???
慣性系において光速で運動可能なのは質量0の素粒子だけ
ご冗談でしょう?名無しさん [sage] 2018/02/03(土) 15:16:32.06ID:???
高分子誘電材料の誘電率の虚数部を大きくするそとによる利点ってなにがありますか?
吸収と関係ありますか?
ご冗談でしょう?名無しさん [sage] 2018/02/03(土) 17:53:37.80ID:???

!!!
ご冗談でしょう?名無しさん [] 2018/02/03(土) 21:21:49.56:b4pUlj23
現在の空間は3次元ですが
量子の世界において裁定する仕組みを導入できれば
4次元以上の空間になるはずです
裁定に関しては情報熱力学、金融工学、統計確率の数理などを研究ください
同様に、市場においても無限次元化していきます
ご冗談でしょう?名無しさん [] 2018/02/03(土) 21:27:04.30:b4pUlj23

光速で光が飛んで行くのが
その光速で飛んでる何かから見えるはずです
それを見てる地上のあなたには区別がつかないだけ
ご冗談でしょう?名無しさん [] 2018/02/03(土) 22:42:07.08:XFZTwZPa
メロディパイプの音のなる仕組みがわからないので質問します。
あれは回すことで風から音が出てるのですか?
ご冗談でしょう?名無しさん [sage] 2018/02/04(日) 00:25:33.37ID:???
ムーニーちゃんしんぷだい
ご冗談でしょう?名無しさん [sage] 2018/02/04(日) 12:38:32.86ID:???

笛と同じさ
口で吹くか回す風かだけ
ご冗談でしょう?名無しさん [] 2018/02/04(日) 15:55:07.12:ntsHoXqM
走る時ストライドが広いと、燃費が良くなるでしょうか?
ご冗談でしょう?名無しさん [sage] 2018/02/04(日) 17:02:20.08ID:???
難しいだろ
ご冗談でしょう?名無しさん [] 2018/02/04(日) 17:34:28.30:NQdFhACD
よくならないってことですか?
ご冗談でしょう?名無しさん [sage] 2018/02/04(日) 18:18:47.88ID:???
物理の問題に定式化してみろ
ご冗談でしょう?名無しさん [] 2018/02/04(日) 18:29:35.76:NQdFhACD
ストライド回数∝酸素消費量ならば
ストライドが広い→ストライド回数が減る→酸素消費量が少ないと仮定できますか?
ご冗談でしょう?名無しさん [] 2018/02/04(日) 18:48:18.73:NQdFhACD
一休さんですか?
ご冗談でしょう?名無しさん [sage] 2018/02/04(日) 18:48:20.40ID:???
物理のなってない。知恵遅れで聞いたら
ご冗談でしょう?名無しさん [sage] 2018/02/04(日) 18:58:49.66ID:???
何で物理の問題だと思うのかね。なんでも親切に教えてくれると勘違いしてのか。
ご冗談でしょう?名無しさん [sage] 2018/02/04(日) 19:52:34.85ID:???
んじゃ物理の問題として聞くけど
ラグランジュ方程式からなんかニュートンの運動方程式出てくるけどあれってなんで?
ラグランジュ方程式がニュートンの運動方程式を内包してる理由ってはっきりしてるの?
ご冗談でしょう?名無しさん [sage] 2018/02/04(日) 19:56:10.00ID:???
そういう風にラグランジアン設定してるからですよ
ご冗談でしょう?名無しさん [sage] 2018/02/04(日) 20:01:59.44ID:???
作用積分の停留点で複数の異なる基本方程式が求まると言う事実は、
作用という量がより基本的な物理量であることを示唆している。
ご冗談でしょう?名無しさん [sage] 2018/02/04(日) 20:13:02.38ID:???
相対論の結果が出てこないということは、ラグランジアンが古典論専用になっているということを意味しています
ご冗談でしょう?名無しさん [sage] 2018/02/04(日) 20:13:31.39ID:???
ニュートン力学専用
ご冗談でしょう?名無しさん [sage] 2018/02/04(日) 20:13:57.72ID:???
「無」に勝るものはありますか?
ご冗談でしょう?名無しさん [sage] 2018/02/04(日) 20:13:58.70ID:???
相対論は古典論なんだが
ご冗談でしょう?名無しさん [sage] 2018/02/04(日) 20:15:57.79ID:???

「神」です
ご冗談でしょう?名無しさん [sage] 2018/02/04(日) 20:16:55.74ID:???

「神」ですら「無」には勝てません。
ご冗談でしょう?名無しさん [sage] 2018/02/04(日) 20:35:57.59ID:???
そんなことして楽しい?
ご冗談でしょう?名無しさん [sage] 2018/02/05(月) 01:53:58.22ID:???
楽しいさ
ご冗談でしょう?名無しさん [sage] 2018/02/05(月) 08:44:19.55ID:???
磁場がHの所に磁気量mの磁極を置くとF=mHの力が働くというのはいいんだけど、磁極の代わりに磁化されていない鉄球を置いたらどんな力が働くかわからない
ご冗談でしょう?名無しさん [sage] 2018/02/05(月) 11:44:32.30ID:???
最初から意味不明
ご冗談でしょう?名無しさん [sage] 2018/02/05(月) 12:28:46.52ID:???
磁気量て何かと思ったら磁荷のことか
ご冗談でしょう?名無しさん [sage] 2018/02/05(月) 12:29:37.79ID:???
大とろ 時価
ご冗談でしょう?名無しさん [sage] 2018/02/05(月) 16:42:32.45ID:???

一様磁場なら力は働かない
ご冗談でしょう?名無しさん [sage] 2018/02/05(月) 20:48:13.89ID:???
ムーニーちゃんしんぷだ〜い
ご冗談でしょう?名無しさん [sage] 2018/02/05(月) 21:42:02.25ID:???
囲碁棋士の井山裕太氏と、数学者の望月新一氏はどっちの方が頭が良いと思われますか?
ご冗談でしょう?名無しさん [sage] 2018/02/05(月) 22:00:48.18ID:???
E-B対応とかE-H対応とかはマジで意味不明
ご冗談でしょう?名無しさん [] 2018/02/05(月) 22:25:34.03:9GT2WFRG
一方の端を座標平面の原点に固定しもう一方の端に
質量mの小球を取り付けたばね定数k、自然長Lのばねがある。
はじめ小球を(L,0)の位置に持っていき、そのあと静かに離した。
このとき小球が描く軌跡を求めよ。ただしy軸負向きに重力が
かかっているものとする。
友達が考えたものですが全然わからないので教えてください
ご冗談でしょう?名無しさん [] 2018/02/05(月) 22:39:47.89:yra15sVc

研究者がアスペだらけだから
相対性理論の解説までアスペなものばかりになってる
もの凄く単純なことを非常に小難しく書いてる
まさに視点が古典
ご冗談でしょう?名無しさん [sage] 2018/02/06(火) 06:08:17.37ID:???
俺はようやく気づいた!!

女人禁制の高野山を作った弘法大使さま

医学的統計学をそれに従い悪用せず純潔を護ったナイチンゲール婦長

この二人に顔向けできないヤリチンやヤリマンしかいなくなったんだ

だからヤリチンやヤリマンはカスみたいな論理性なんだ
ご冗談でしょう?名無しさん [] 2018/02/06(火) 06:53:04.88:xNCjvIuO
>一様磁場なら力は働かない

アホかボケ。
鉄は磁石に引っ付くだろうがアホンダラー。

鉄が磁化されてその両端のN極S極の距離が大きいほど
N極S極に働く力の差も大きくなって強く引っ張られるんだよ。

つまり、鉄の磁化率と形状によって
磁場の原因である磁石や電流に引っ張られる強さが変わってくる。

なぜ磁石に引っ付くのか、まるで分ってないな。
くっくっく
ご冗談でしょう?名無しさん [] 2018/02/06(火) 06:55:58.51:xNCjvIuO
ああ、一様な磁場って
勝手に条件つけてんのか。

だったら磁化された鉄のN極S極に働く力の大きさは同じだから
相殺してゼロだな。

くっくっく
ご冗談でしょう?名無しさん [sage] 2018/02/06(火) 07:16:12.74ID:???
くっくっく気持ち悪い上に頭まで...
ご冗談でしょう?名無しさん [] 2018/02/06(火) 07:23:25.54:I9s+37cA
眞子さまが絶対に女王陛下だ!! この意見に反対するものはこの文章を読め!!!!

俺はようやく気づいた!!!

ただ一人歩めと言ったお釈迦様!!女人禁制の高野山を作った弘法大使さま!!!

医学的統計学をそれに従いそれを悪用せず純潔を護ったナイチンゲール婦長!!!

この三人に顔向けできないヤリチンやヤリマンや学歴差別者と血液型差別者しかいなくなったんだ!

だからヤリチンやヤリマンや裏口入学者たちはカスみたいな判断力と理解力と論理性なんだ!
ご冗談でしょう?名無しさん [] 2018/02/06(火) 07:47:03.52:I9s+37cA
駅弁だけが男のたった一つの勲章だって
この胸に信じて生きてきた

泣きたくなるような 辛い時もあるけど
いつも俺たち 頑張ってきた
時の重さに流されそうになった時でも
歯をくいしばりたえてきた
ガキの頃 路地裏で見た 夜空にきらめいた
流れる星を見て 誓った思いを忘れちゃいないぜ

駅弁だけが男のたった一つの勲章だって
この胸に信じて生きてきた

氷のように冷たい 世間の壁が
いつも遮る 俺たちの前を
胸に描いた この夢は 半端じゃないから
かじかむこの手 握り締め
ガキの頃 赤とんぼ 追いかけてた時の
燃えてた瞳は 今でも俺たち忘れちゃいないぜ

駅弁だけが男のたった一つの勲章だって
この胸に信じて生きてきた

駅弁だけが男のたった一つの勲章だって
この胸に信じて生きていく 👀
Rock54: Caution(BBR-MD5:0be15ced7fbdb9fdb4d0ce1929c1b82f)
ご冗談でしょう?名無しさん [] 2018/02/06(火) 08:06:28.83:I9s+37cA
風俗に限らず素性の知れない女や男とは簡単にキスするな

  俺がくたばった後に、「詠み人知らず」で遺せんか!?
ご冗談でしょう?名無しさん [] 2018/02/06(火) 08:40:43.49:xNCjvIuO
>E-B対応とかE-H対応とかはマジで意味不明
マシなのはE-H対応だ。
しかし、本当はどちらも大間違いなんだよ。
なぜなら、Eは発散場であり、HやBは回転場だからだ。

形状の違う場に対応もクソもない。
もっと言えば、本質的なのはE、H、Bの3つであり
Dは補助的な量にすぎん。

このことは電磁気学を20年ぐらいはやってないと
気づかんことであって、チミらみたいな若造にはチンプンカンプンでも仕方ないわな。

さて、
・ガウスの法則 divE=(ρ+ρ')/ε0
・分極 divP=ρ'

この2つにより、D=ε0E+PとするとdivD=ρを得るがこれは
「物質の存在をDやPで表すと、右辺の真電荷ρによってあたかも物質の存在を考慮しなくてもいい」
ように見えるだけのものであって、実際はDやPを考慮しないと無意味な式なので
Dは本質的ではないのである。本質的なのは上の2式である。

変位電流∂D/∂tも本質的には∂(ε0E+P)/∂t=∂ε0E/∂t+∂P/∂t
と理解すべきで、右辺第2項の∂P/∂tは分極の振動によって発生する「普通」の電流密度にすぎない。
変位電流の本質は右辺第1項の∂ε0E/∂tにこそある。

くっくっく
ご冗談でしょう?名無しさん [] 2018/02/06(火) 08:41:34.79:xNCjvIuO
このことをはっきりと認識できていない人間がほとんどである。
最近の若造大学教授なんかはまずそうだ。
変位電流とは、∂D/∂tではなくPの振動を除外した∂ε0E/∂tであるというべきなのである。
くっくっく

で、
・ビオサバールの法則 rotB=μ0(i+i')
・磁化 rotJ=μ0i' (磁気双極子モーメント=μ0I'Sとした等価磁化電流モデルから導出する)
この2つによりB=μ0H+JとするとrotH=i、divB=0(divrotは数学的にゼロ)を得るが、本当はいきなり
・rotB=μ0i+rotJ
としてもよい。むしろこのほうが自然であって、
「物質が存在するとiが作る磁場が強められて加算される」と素直に解釈できるからである。
ここから逆算してJは等価磁化電流モデルに行き着き、Jは単位体積当たりの磁気双極子モーメントで
あることにも行き着く。
もちろん、iがなくても最初からrotJだけが存在する場合もある。磁石なんかそうであり、
ヒステリシスもある。

大事なことは真電流iがHを作り、それに誘発されてrotJが発生・変化するのでrotB=μ0i+rotJは
・rotB=μ0rotH+rotJ
であることを忘れてはならないということなのである。
その意味で、HとBはどちらも本質的な量ということになる。
Eが本質的であってDは本質的な量ではないとした違いがここにある。

まとめると
Eは本質的であり、Dは補助的な量にすぎない。
HとBはどちらも本質的な量である。
よってE−HB対応と言うのがもっともマシである。
しかし発散場と回転場で形状が違うので対応させるのはそもそもおかしい。

ということだな。
難しいか?、ああ?
そこの小僧よ。

くっくっく
ご冗談でしょう?名無しさん [] 2018/02/06(火) 09:00:13.60:xNCjvIuO
ヒステリシスループな。
B-H曲線でもよいが、これを測定するときに横軸のHをどうするかという問題がある。

ttp://nsmsxserve02.ph.kagu.tus.ac.jp/ferrotakasaki/page1/page4/page4.html

単純に加えた電流だけのHにするのか、
そうではなく対象物の内部磁界を補正するのかだ。

対象物がドーナツみたいに閉じておらず両端が存在する場合、
そこには磁化による磁荷が発生する。それが対象物内部にHに逆らうH'を作る。

ま、多くの場合が補正しないでそのまま電流のHだけで描いた曲線にする。
きれいな補正などしょせんは不可能だからな。
補正するよりそのままのデータのほうが真実なので良かろう。

大事なのは、
そのことを知っているのかどうかだ。

しょーもない音の問題だして
変位波なのか疎密波なのかで揉めるより
こういった本質的な問題出せよ
アホの若造出来損ないなんちゃって教授どもが。

くっくっく
ご冗談でしょう?名無しさん [sage] 2018/02/06(火) 09:01:53.62ID:???
くっくっく気持ち悪い
ご冗談でしょう?名無しさん [sage] 2018/02/06(火) 09:12:46.39ID:???

微分形式のドラムコホモロジーによる一般ストークスの定理とかベクトルポテンシャルによるAB効果やベールの位相
可換ゲージ理論としての電磁気学

の方が正統的な深め方だろ。そこら辺の話の。
ご冗談でしょう?名無しさん [] 2018/02/06(火) 09:23:38.86:xNCjvIuO
数式いじりの物理知らずな。
アホらしいにもほどがある。
宇宙は数学的論理で出来てるとか思ってんだろうな。

数学的論理が先か物理が先か、
あるいは同じだと思ってんのか知らんが
しょんべん臭すぎて半笑いするしかない。

くっくっく
ご冗談でしょう?名無しさん [] 2018/02/06(火) 09:30:34.12:xNCjvIuO
ああ、それから
ベクトルポテンシャルなんかは
まったくどうでもいい補助量だからな。

あのインチキ臭い実験によって
電子はベクトルポテンシャルを感じたとかマジで思ってんのなら
しょんべんでも飲んでろってこった。

くっくっく
ご冗談でしょう?名無しさん [sage] 2018/02/06(火) 10:13:43.36ID:???
くっくっく気持ち悪い
ご冗談でしょう?名無しさん [sage] 2018/02/06(火) 10:48:57.07ID:???
くくはちじゅういち
ご冗談でしょう?名無しさん [] 2018/02/06(火) 11:13:34.19:C3Wxrorh
【沖縄に謎の発光体!?】.....1月27日、与那原でUFO撮影.....【マイトLーヤの星か?】
ttp://rosie.5ch.net/test/read.cgi/liveplus/1517624262/l50
【UFO】  4つのW◇型″発光体  ≪マイトLーヤの星≫  地上にW思考″を放送  【大宣言】
ttps://rosie.5ch.net/test/read.cgi/liveplus/1517707452/l50
【テレパシー】元国連職員「いきなり声が…『聞こえないフリ、分からぬフリをするな、照れをなくせ』」
ttp://rosie.5ch.net/test/read.cgi/liveplus/1513067436/l50
ご冗談でしょう?名無しさん [sage] 2018/02/06(火) 12:34:42.38ID:???

AB効果をしらんのか
ご冗談でしょう?名無しさん [sage] 2018/02/06(火) 13:06:52.13ID:???
「くっくっく」が付いてると見ないで済むから便利だな
ご冗談でしょう?名無しさん [sage] 2018/02/06(火) 14:41:21.48ID:???
関連スレ

数式なんかどうでも良い、君はその物理現象をどう考えているのか、それを言葉で説明してくれ!
ttps://rio2016.5ch.net/test/read.cgi/sci/1452603554/l50
ご冗談でしょう?名無しさん [] 2018/02/06(火) 17:48:31.71:pJZQmOT7
この問題の(12)〜(14)どなたか教えてくれませんか?
ttps://imgur.com/a/fwPCV
ご冗談でしょう?名無しさん [] 2018/02/06(火) 17:49:57.32:pJZQmOT7
大学一年で
マクスウェルの関係式の使い方がわかりません
ご冗談でしょう?名無しさん [sage] 2018/02/06(火) 17:54:17.55ID:???
こんなの見つけた
ttps://imgur.com/gallery/ZP9xE
ご冗談でしょう?名無しさん [sage] 2018/02/07(水) 00:07:34.85ID:???

適当に代入して波動方程式でも導け
ご冗談でしょう?名無しさん [] 2018/02/07(水) 01:41:40.33:/rlw3epU
マクスウェルの関係式は熱力学だろ
ご冗談でしょう?名無しさん [sage] 2018/02/07(水) 07:38:53.61ID:???
一方の端を座標平面の原点に固定しもう一方の端に
質量mの小球を取り付けたばね定数k、自然長Lのばねがある。
はじめ小球を(L,0)の位置に持っていき、そのあと静かに離した。
このとき小球が描く軌跡を求めよ。ただしy軸負向きに重力が
かかっているものとする。
友達が考えたものですが全然わからないので教えてください
ご冗談でしょう?名無しさん [sage] 2018/02/07(水) 07:44:53.57ID:???
高校生ですか?
ご冗談でしょう?名無しさん [sage] 2018/02/07(水) 08:24:37.75ID:???

139です。自分は高校生ですが、これを考えた
友達はだいぶ物理が進んでいるようなので、
自分には解けませんでした
ご冗談でしょう?名無しさん [sage] 2018/02/07(水) 08:34:10.31ID:???
長さと角度の非線型項があるから
ご冗談でしょう?名無しさん [sage] 2018/02/07(水) 08:34:52.49ID:???
作った本人も解けてないと思うよ
ご冗談でしょう?名無しさん [sage] 2018/02/07(水) 13:26:09.18ID:???

ばねと重力を合わせたポテンシャルは簡単に出るから
あとは Wolfram|Alpha にやらせろ
ご冗談でしょう?名無しさん [sage] 2018/02/07(水) 17:12:53.91ID:???
名問の森 電磁気熱原子 の71がわかりません

n + p → D + γの問題で、線をひいた部分がわかりません
Δmc^2 = Ed + E1 + Ebとならないのはなぜでしょうか?
失った質量がまず結合に使われて余ったエネルギーが運動エネルギーとγ線になる、って想像してしまいます

ttps://i.imgur.com/T6ZDFdK.jpg
ttps://i.imgur.com/sMTqDrW.jpg
ご冗談でしょう?名無しさん [sage] 2018/02/07(水) 20:28:25.78ID:???

ファアインマングラフを書けよ
ご冗談でしょう?名無しさん [sage] 2018/02/07(水) 20:29:21.73ID:???
間抜けな発音すんな
ご冗談でしょう?名無しさん [sage] 2018/02/07(水) 20:39:28.62ID:???

>失った質量がまず結合に使われて余ったエネルギー
これ見るとなんか誤解してるようだ。
結合状態は軽い、つまり エネルギーが低い状態なので
余分なエネルギーを(Dとγの)運動エネルギーとして吐き出すと理解すべき。
だから Δmc^2 = Ed + E1 = Eb なのでは?
ご冗談でしょう?名無しさん [sage] 2018/02/07(水) 20:46:25.18ID:???
反応前後のエネルギー保存を考えれば自然に出てくる。1枚目の画像の右の絵を見よ。

静止質量エネルギーの減少分Δmc^2(絵の薄赤部分)が結合エネルギーの定義であり、
反応後はそれがDとγの運動エネルギーになる。それだけのこと
145 [sage] 2018/02/07(水) 20:47:57.72ID:???

ありがとうございます
分かりやすかったです!
145 [sage] 2018/02/07(水) 20:50:33.96ID:???

ありがとうございました!
ご冗談でしょう?名無しさん [sage] 2018/02/07(水) 20:54:49.12ID:???

専ブラをアップデートしたので間違えた。すまん
ご冗談でしょう?名無しさん [sage] 2018/02/07(水) 23:38:36.56ID:???
難しい物理の質問をしたいと思うのですけど、ここのスレッドのレベルはその質問に答えられるくらい高いものなのでしょうか?
ご冗談でしょう?名無しさん [sage] 2018/02/08(木) 00:35:31.51ID:???
ぷっちーポッポ
ご冗談でしょう?名無しさん [sage] 2018/02/08(木) 09:47:17.12ID:???

知恵遅れにしましょう
ご冗談でしょう?名無しさん [sage] 2018/02/08(木) 20:54:42.40ID:???
今ガロア理論を勉強してるんだけど物理に役に立つことある?
ご冗談でしょう?名無しさん [sage] 2018/02/09(金) 11:41:29.91ID:???
レベル次第
ご冗談でしょう?名無しさん [sage] 2018/02/09(金) 16:30:51.51ID:???
ムニちゃーんポッポ
ご冗談でしょう?名無しさん [sage] 2018/02/10(土) 09:38:53.83ID:???
フェルミ面って何ですか?
ご冗談でしょう?名無しさん [sage] 2018/02/10(土) 14:25:09.53ID:???
運動量空間で見たフェルミ粒子系の基底状態の最高エネルギー面
ご冗談でしょう?名無しさん [sage] 2018/02/10(土) 14:44:44.85ID:???
まっとうな回答ありがとうございます
ご冗談でしょう?名無しさん [sage] 2018/02/10(土) 15:12:06.98ID:???
それは厳密には正しくない
ご冗談でしょう?名無しさん [sage] 2018/02/11(日) 13:07:25.32ID:???
めんどくせーからオマエが書け
ご冗談でしょう?名無しさん [sage] 2018/02/11(日) 18:54:05.91ID:???
無限大の空間で、無限大の動く物体が、無限の速さで移動したり、無限の高さをジャンプしたりしたらどんな感じになるのでしょうか?
ご冗談でしょう?名無しさん [sage] 2018/02/11(日) 19:39:18.08ID:???

最近釣れないぞ、ウンコヒマラヤおっさん
ご冗談でしょう?名無しさん [sage] 2018/02/12(月) 02:16:57.79ID:???
ムーニーちゃんしんぷとセバスチャンしんぷはどっちの方が凄いですか?
ご冗談でしょう?名無しさん [sage] 2018/02/12(月) 11:39:30.04ID:???
そんな事して楽しい?
ご冗談でしょう?名無しさん [] 2018/02/12(月) 17:30:17.72:8PXYlVIU
ccp構造の第一BZは斜方切頂立方八面体ですか?
ご冗談でしょう?名無しさん [sage] 2018/02/13(火) 18:22:37.19ID:???
ムニちゃーんポッポ
ご冗談でしょう?名無しさん [sage] 2018/02/14(水) 11:02:00.72ID:???
ある物質のある物理量を測定すると、測定前と後とでは波動関数が変わってしまうのでしょうか?
ご冗談でしょう?名無しさん [sage] 2018/02/14(水) 12:54:29.28ID:???
記述次第
ご冗談でしょう?名無しさん [sage] 2018/02/14(水) 13:01:19.16ID:???
ちょっとした観測の質問
ご冗談でしょう?名無しさん [sage] 2018/02/14(水) 21:55:46.03ID:???

物質量の固有状態で測定したときは変わらない
ご冗談でしょう?名無しさん [sage] 2018/02/14(水) 22:35:41.75ID:???
ぷっちーポポ
ご冗談でしょう?名無しさん [sage] 2018/02/14(水) 22:50:29.36ID:???

もし波動関数が演算子Fの固有関数以外のものも含んでいたら、
Fを測定した瞬間に波動関数は測定値の固有関数のみに変わってしまうということでしょうか?
ご冗談でしょう?名無しさん [sage] 2018/02/15(木) 13:16:06.21ID:???
そんな瞬間などねーよ
ご冗談でしょう?名無しさん [sage] 2018/02/15(木) 19:43:06.70ID:???
ちょっと思いついたんですけど、ドーナツ状の導体(電荷は中性)を急回転したら
磁場が生じたりはしないのでしょうか?
最初のうちは自由電子が回転から取り残されて結果的に正電荷の流れになるのではないかと。
等速回転ならそのうち自由電子が追いついて磁場は消える。だから加速回転してる間は磁場は消えない。
自由電子が追いつく原因はたぶん導体抵抗、それなら超伝導体なら?

ドーナツ状でなくてもよさそうですが、
この種の効果(があるなら)に名前ついてたりするんでしょうか?
ご冗談でしょう?名無しさん [sage] 2018/02/15(木) 20:09:34.09ID:???
電子には一切力を書けず回転させるなら生じるけど
等速で回転させると磁場は電子の緩和時間程度で消える
銀の単体とかなら数十フェムト秒,超伝導体なら無限大
ご冗談でしょう?名無しさん [sage] 2018/02/16(金) 00:21:00.90ID:???
くっくっくが来るぞ
ご冗談でしょう?名無しさん [] 2018/02/16(金) 08:06:42.88:6gvQF6Uq
ちょっと思いついたんですけど、導体(電荷は中性)を交流電圧を加えたら、
電流の位相がズレるのではないでしょうか?

V = eE = m・dv/dt = (m/e)・dI/dt


はこのくらいアホな思考。
DJ学術  [] 2018/02/16(金) 08:47:00.66:A6PwTZe3
単純な図解だな。絵画のセンスが大事。抽象 具現。
DJ学術  [] 2018/02/16(金) 08:48:27.97:A6PwTZe3
電子のコンディション、予後がいい 家具 インテリア 電化製品なんか、
たかいよねえ。
ご冗談でしょう?名無しさん [sage] 2018/02/16(金) 09:03:01.34ID:???
アホが来た
DJ学術  [] 2018/02/16(金) 09:20:34.72:A6PwTZe3
バカが去った。
DJ学術  [] 2018/02/16(金) 09:22:23.55:A6PwTZe3
数学式は文系みたいな意味論ではかたずけられないほど短絡的じゃないのに。
世界の万民が数式が連続で流ちょうに描けるようになったら、
僕もまだましな存在だったかもね。僕は経済分野の方が伸びると思う。
純粋数学より。
DJ学術  [] 2018/02/16(金) 09:36:55.44:A6PwTZe3
更には経営数術
ご冗談でしょう?名無しさん [sage] 2018/02/16(金) 10:08:26.85ID:???

"フライホイール" で適当にググって出てきたページ
ttp://academy.maxonjapan.co.jp/book/_017
>アルミのフライホイール(直径78mm、重量約300g)を7秒以内に17,000rpmに加速します。
の数値を参考にして計算してみる。(他に加速性能が分かる物はあまり見つからなかった)

外周の(円周方向)平均加速度: a = 3.14 * 78e-3 * (17000/60)/7
加速回転系における(円周方向)慣性力: f = ma
起電力: V = πR * f/e = (3.14 * 78e-3)^2 * (17000/60)/7 * 9.11e-31 /1.60e-19 = 1.38e-11 [V]

予想はしてたがかなり微弱である。他の製品で数桁稼ぐくらいの余裕はあるだろうから
計測可能な ナノボルト(〜1e-9 [V]) 程度にはなるかもしれない。

起電力に応じて電流が流れ、全体で中性なので加速回転系で生じる電流は実験系での電流と等しくなる。
とにかく一瞬にせよ磁場は生じるはずである。
ご冗談でしょう?名無しさん [] 2018/02/16(金) 10:27:29.17:5lyUh0uE
【アポロ11号の嘘】 高須医院長「行ってるわけがない」 高原剛一郎「アポロもキリストの復活も事実」
ttp://rosie.5ch.net/test/read.cgi/liveplus/1518660021/l50
ご冗談でしょう?名無しさん [187] 2018/02/16(金) 10:50:40.70ID:???
(続き)
ちなみに円盤状のホイールだとしたら、遠心力に起因する半径方向の起電力も生じる

ω = 2 * 3.14 *17000/60 [rad/s]
V = ∫ [0, R/2] m/e r ω^2 dr = m/e * R^2 * ω^2 / 8 = 1.37e-8 [V]

この程度なら、外周に導電ブラシを当てれば軸側との電位差が測れるはず。
DJ学術  [] 2018/02/16(金) 11:11:07.41:A6PwTZe3
しかし計算や数式って耐久性があって版型やパターンも豊富なの?
ご冗談でしょう?名無しさん [sage] 2018/02/16(金) 11:45:17.84ID:???
文系脳、ゴミを書き散らす
DJ学術  [] 2018/02/16(金) 12:11:49.03:A6PwTZe3
右と左で文系理系昔分けて出来てました。
ご冗談でしょう?名無しさん [] 2018/02/16(金) 12:17:17.46:TkyulDDp
意味論って文系もやるのか
DJ学術  [] 2018/02/16(金) 12:20:57.67:A6PwTZe3
言語の遺伝子ぐらいしか執筆歴はないけど。
ご冗談でしょう?名無しさん [sage] 2018/02/16(金) 13:32:32.05ID:???
おならぷっぷーたろう
ご冗談でしょう?名無しさん [sage] 2018/02/16(金) 13:33:39.77ID:???
DJウンコ
ご冗談でしょう?名無しさん [] 2018/02/16(金) 20:50:21.95:C4X4Nmmj
エネルギー 循環 システム
ttps://search.yahoo.co.jp/search;_ylt=A7YWOSrzxIZa_hUAgvOJBtF7?p=%E3%82%A8%E3%83%8D%E3%83%AB%E3%82%AE%E3%83%BC%E5%BE%AA%E7%92%B0%E3%80%80%E3%8
2%B7%E3%82%B9%E3%83%86%E3%83%A0&search.x=1&fr=top_ga1_sa&tid=top_ga1_sa&ei=UTF-
8&aq=-1&oq=%E3%82%A8%E3%83%8D%E3%83%AB%E3%82%AE%E3%83%BC%E5%BE%AA%E7%92%B0+%E3%
82%B7%E3%82%B9%E3%83%86%E3%83%A0&at=&aa=&ai=HLRK_d_8QfyjQ9DjpZBbMA&ts=9864
ご冗談でしょう?名無しさん [] 2018/02/16(金) 20:52:22.55:C4X4Nmmj
エネルギー  増幅  システム
ttps://search.yahoo.co.jp/search?p=%E3%82%A8%E3%83%8D%E3%83%AB
%E3%82%AE%E3%83%BC%E3%80%80%E5%A2%97%E5%B9%85+%E3%82%
B7%E3%82%B9%E3%83%86%E3%83%A0&aq=-1&oq=&ai=
.jO07PP9Tv6LrQimdhwRJA&ts=126890&ei=UTF-8&fr=top_ga1_sa&x=wrt
ご冗談でしょう?名無しさん [] 2018/02/16(金) 20:55:10.08:C4X4Nmmj
圧縮 ジェネレーター これは、名前で考えてください。
ttps://search.yahoo.co.jp/search?p=+%E5%9C%A7%E7%B8%AE+%E3%82%
B8%E3%82%A7%E3%83%8D%E3%83%AC%E3%83%BC%E3%82%BF%E3%
83%BC&aq=-1&oq=&ai=8E1JumpPSIeCHTkeGvcAGA&ts=1335&ei=UTF-8&fr=
top_ga1_sa&x=wrt
ご冗談でしょう?名無しさん [] 2018/02/16(金) 20:58:50.04:C4X4Nmmj
エネルギー 循環 ジェネレーター
ttps://search.yahoo.co.jp/search?p=%E3%82%A8%E3%83%8D%E3%83%AB%E3%82%AE%E3%83%BC%20%E5%BE%AA%E7%92%B0%20%E3%82%B
8%E3%82%A7%E3%83%8D%E3%83%AC%E3%83%BC%E3%82%BF%E3%83%BC&sp=1&search.x=1&tid=top
_ga1_sa&ei=UTF-8&aq=undefined&oq=%E3%82%A8%E3%83%8D%E3%83%AB%E3%82%AE%E3%83%BC+%E5%BE%AA%E7%92
%B0+%E3%82%B8%E3%82%A7%E3%83%B3%E3%83%AC%E3%83%BC%E3%82%BF%E3%83%BC&at=s&ai=PuF
gnhQ2Re.JzLPXAUYAVA&ts=14674&fr=top_ga1_sa
ご冗談でしょう?名無しさん [] 2018/02/16(金) 21:43:51.59:C4X4Nmmj
圧縮ジェネレーターは、東北大学に注文すればいいだけだから。ARMMO現実化システムとか、京都大学、
佐藤幸治先生に注文すればいいだけだから。
ご冗談でしょう?名無しさん [] 2018/02/16(金) 21:44:54.77:C4X4Nmmj
圧縮ジェネレーターは、東北大学に注文すればいいだけだから。ARMMO現実化システムとか、京都大学佐藤研、
佐藤幸治先生に注文すればいいだけだから。
ご冗談でしょう?名無しさん [] 2018/02/16(金) 21:46:43.76:C4X4Nmmj
たまご型ブルーハードディスク、たまご型イエローハードディスク、たまご型ブルーレイハードディスク、たまご型イエローレイハードディスクとかは、
東北大学に注文すればいいだけだから。
ご冗談でしょう?名無しさん [] 2018/02/16(金) 22:00:01.13:CFW/SBcM

どうなんですか?
ご冗談でしょう?名無しさん [sage] 2018/02/16(金) 23:25:59.53ID:???
ここの回答者は電気力線すらわからない人達ですから聞くだけ無駄ですよー
ご冗談でしょう?名無しさん [sage] 2018/02/17(土) 00:41:19.65ID:???
でぼぼぼーん
ご冗談でしょう?名無しさん [sage] 2018/02/17(土) 00:41:43.10ID:???
劣等感婆まだ電気力線引きずってんのか
マジモンやんけw
ご冗談でしょう?名無しさん [sage] 2018/02/17(土) 01:42:32.28ID:???
婆は数学板からいなくなった?
ご冗談でしょう?名無しさん [sage] 2018/02/17(土) 05:19:29.39ID:???
以下以外にも属性はありますか?


熱(超高温、超低温)、冷凍(凍結)、擬似太陽、超重力、超圧縮、加速、減速
倍速、鈍速、エントロピー、波長攻撃、雷撃、炎、腐敗、爆発、反作用爆発
放射能、超音波、重量増加、吹き飛ばし、精神、魂操作、憑依、精神汚染
狂化、不信、幻覚、催眠、睡眠、感情吸い上げ、戦意喪失、気、取り込み
物理無効攻撃、加速無効、攻撃反射無効、方向感覚無効、解毒無効、異常回復無効
結界無効、無限蘇生無効、再生無効、強化無効、不思議攻撃無効、核無効
属性無視攻撃、特殊能力無視攻撃、攻撃力無視攻撃、防御力無視攻撃
素早さ無視攻撃、世界観無視攻撃、法則支配、支配、魂支配、精神支配
攻撃模写・模倣、能力模写・模倣、弱点解析、攻撃解析、麻痺、五感麻痺
方向感覚麻痺、細菌、ウィルス、悪臭、毒、腐食、腐敗、酸、スタン、バインド
不動縛、攻撃不能、防御能力剥奪、盗み、物質変換、異形化、次元切断
時間切断、空間切断、魂切断、運気切断、切断、精神破壊、時間破壊、空間破壊
次元破壊、霊子破壊、内部破壊、寿命破壊、法則破壊、運気破壊、概念破壊
武器破壊、防具破壊、原子破壊、分子破壊、消滅、情報消失、即死、一撃必殺
生命速度加減、次元操作、時間操作、空間操作、因果律操作、運命操作、確率操作
記憶操作、認識操作、情報操作、行動操作、可能性操作、死体操作、運気操作
命中率低下、回避率低下、攻撃力低下、防御力低下、素早さ低下、耐性低下
知能低下、レベル低下、不思議攻撃(超高温、超低温、冷凍、武器、レーザー
ご冗談でしょう?名無しさん [sage] 2018/02/17(土) 05:20:07.70ID:???
プラズマ、酸、斬撃、猛毒、腐食、腐敗、雷、火炎、影、爆発、ブラックホール
超音波、溶解、疫病等)、超能力(透視、千里眼、精神干渉、精神感応、同念同調
念動力、発火、電撃、空力、念写、予知、幽体離脱、瞬間移動、時間移動等)
特殊攻撃(超高温、超低温、冷凍、武器、レーザー、プラズマ、酸、斬撃、猛毒
腐食、腐敗、雷、火炎、影、爆発、ブラックホール、超音波、溶解、疫病等)
魔法攻撃、音系魔法、呪い、石化、無属性攻撃、虚属性攻撃、分断攻撃、火属性攻撃
水属性攻撃、風属性攻撃、地属性攻撃、氷属性攻撃、光属性攻撃、聖属性攻撃
闇属性攻撃、吸収(血液、ガソリン、水、精気、魂、寿命、運気、銀河、宇宙
空間歪曲、物理攻撃、不思議攻撃、電撃、炎、熱、核、重力場等)、吸収同化耐性
HPドレイン耐性、MPドレイン耐性、ステータスドレイン耐性、レベルドレイン耐性
吸引(魂、物理等)、世界観規模の吸引攻撃、素粒子分解、ステータス反転、追放
転移、時空間移動、魂融合、封印、異空間障壁・幽閉、全知、全能、世界改変
浄化、魔除け、探知、情報収集、考察人操作、全宇宙全次元全階層全知全能攻撃
真の全知全能攻撃、超知超能攻撃、全宇宙全次元全階層超知超能攻撃
ご冗談でしょう?名無しさん [sage] 2018/02/17(土) 05:20:59.10ID:???
真の超知超能攻撃、世界図破壊攻撃、内包攻撃、メタ攻撃、設定破壊攻撃
設定操作攻撃、文字破壊攻撃、次回予告操作攻撃、アオリ文操作攻撃
他作品転移攻撃、補正無視攻撃、作品打ち切り攻撃、n倍強化攻撃、無限ループ攻撃
ランキング操作攻撃、「勝利していることがココに書いてある」攻撃
テンプレ・非テンプレ破壊攻撃、テンプレ自体を無効化攻撃、常時勝利能力攻撃
バグ攻撃、チート攻撃、ルールの変更、新要素
「実は支配済みでした」的攻撃など。
属性の種類はあらゆる全てより膨大な数(当然∞や∞^∞も含める)を超越した
数である。
ここに書かれていない存在する、存在しない属性も含まれる。
非全属性攻撃(全属性に含まれない属性)も全属性に含める。
ご冗談でしょう?名無しさん [sage] 2018/02/17(土) 13:26:38.38ID:???
いかにもな廚二
ご冗談でしょう?名無しさん [age] 2018/02/17(土) 15:41:12.01ID:???
ビームライフルって結局なにを撃ち出してるの [857186437]
ttp://leia.5ch.net/test/read.cgi/poverty/1518849036/
ご冗談でしょう?名無しさん [] 2018/02/17(土) 18:31:36.89:VnWeCU3b
物理を勉強したことは無いのですが、
ミクロに見たときに積がプランク定数になる物理量は本質的に同じものだと聞きました、何故二元的なペアで存在しているのですか?
とあるペアともう一つのペアの間の変換等をなして等価に解釈することは可能ですか?
また、全てを一元的定数として解釈することはできないのですか?
ご冗談でしょう?名無しさん [sage] 2018/02/17(土) 18:51:08.77ID:???

元が大した話ではないですし、聞きかじりの知識でわかるものでもないので、忘れるのが一番でしょうね
ご冗談でしょう?名無しさん [] 2018/02/17(土) 20:54:02.44:LsIrshgQ
退職後の手慰みに電磁気学を勉強しようかと思っています(ゆくゆくは相対性理論を理解したい)。
数学は大学受験レベルで、現在教科書を見ながら思い出している最中です。
物理は力学のほんの基礎を覚えている程度です。

その上で、いきなり以下の3冊に当たっても理解できるでしょうか。
田舎に住んでいて近くの書店でこれらを参照することができないので、ご教示いただければ幸いです。

・前野昌弘『よくわかる電磁気学』
・馬場敬之『スバラシク実力がつくと評判の電磁気学キャンパス・ゼミ』
・竹内淳『高校数学でわかるマクスウェル方程式』(ブルーバックス)

どうぞ、よろしくお願いいたします。
ご冗談でしょう?名無しさん [sage] 2018/02/17(土) 21:57:50.18ID:???
ムーニーちゃんしんぷだ〜い
ご冗談でしょう?名無しさん [sage] 2018/02/17(土) 22:43:31.87ID:???

前野さんの本がいいよ

物理の本質を理解する近道は
数学のちゃんと勉強することだよ
ご冗談でしょう?名無しさん [] 2018/02/17(土) 23:39:23.35:xgI8ZETi

考えてることが自分と似ている
私は通勤時にマックスウェルの方程の解説本ちょっと見て「こりゃ無理だ」と一度挫折してます
ご冗談でしょう?名無しさん [] 2018/02/17(土) 23:43:41.22:xgI8ZETi
質問
物体の質量は、球なら半径、立方体なら一辺の長さの3乗に比例していますが、
これは宇宙空間が3次元であることの証拠にはなりませんか?
隠れた4,5,6...次元なんて存在していないと
ご冗談でしょう?名無しさん [sage] 2018/02/17(土) 23:50:17.10ID:???
量子力学で、離散量である量子を扱ってるはずなのに
差分じゃなく微分を使える理由って何?
ご冗談でしょう?名無しさん [sage] 2018/02/18(日) 00:03:42.53ID:???

離散量を微分することなんてないが
ご冗談でしょう?名無しさん [sage] 2018/02/18(日) 02:49:04.56ID:???

時空間の量子化に成功してないから
ご冗談でしょう?名無しさん [sage] 2018/02/18(日) 03:14:33.78ID:???
全宇宙全世界全次元全階層全空間全領域を完全に消滅させたらどうなりますか?
ご冗談でしょう?名無しさん [sage] 2018/02/18(日) 03:33:56.69ID:???

正解! だから20世紀まで宇宙空間は3次元と関係のない時間
だと誰もが思っていた。でも、物体の速度を変えると質量が変わるように
思われたので4つめの次元が必要となったわけ。
ご冗談でしょう?名無しさん [sage] 2018/02/18(日) 03:41:09.64ID:???


物理の先生なら、概念を覚えていけば簡単1〜2年でたどり着けるでしょうが、
時間があるのであれば、放送大学のテキストを買って毎週定期的に視聴されては
どうでしょうか? BSを見られるのであればテキスト代だけで無料で勉強できる
と思います。必要な数学の講義もありますから効率的かも知れません。
単位が必要なら入学する方法もありますね。
ご冗談でしょう?名無しさん [sage] 2018/02/18(日) 07:20:52.26ID:???
んー、放送大学なめてるようだけどアレ結構レベル高いよ。
放送大学テキスト(+視聴)だけだと絶対わからんと思う。
俺も前野本あたりから始めるのがいいと思う。
ご冗談でしょう?名無しさん [] 2018/02/18(日) 08:58:09.26:tn+7PKBX

速度が異なる2つの系では4次元目以降のサイズが異なるようなイメージ?
ご冗談でしょう?名無しさん [sage] 2018/02/18(日) 10:42:03.32ID:???

電磁気学 (物理テキストシリーズ 4)(砂川)がお勧め
ご冗談でしょう?名無しさん [sage] 2018/02/18(日) 11:58:06.57ID:???

舐めてるつもりはないけれど、放送大学の物理もグレードが3段階ぐらいある。大学の講義で、
教科書だけしか参照しない、手取り足取りが必要なゆとり世代はどうか知らないが、
ネットが使える環境では教員経験者なら当然付いていけるレベルだよ。
ご冗談でしょう?名無しさん [sage] 2018/02/18(日) 11:59:18.91ID:???

砂川先生には初等物理を直接教えていただいたが、本当に教えるのが上手
な先生だった。
ご冗談でしょう?名無しさん [sage] 2018/02/18(日) 12:06:27.36ID:???

感覚で理解した方が良いのか?きちんと数式で理解した方が良いのか?
この板でも議論が分かれるところなんだけれど、実際に質量を測定するための
物差しと時計が変わるところから始まる、ぐらいかな?
ご冗談でしょう?名無しさん [] 2018/02/18(日) 13:49:02.70:3Qbupe6v
この問題って液柱の中と浸透圧がつり合ってるんですよね?下の広い口での浸透圧は細い液柱だと大きくなりませんか?

ttps://i.imgur.com/k1PgC1W.jpg
ご冗談でしょう?名無しさん [sage] 2018/02/18(日) 14:02:26.30ID:???
つ パスカルの原理
ご冗談でしょう?名無しさん [sage] 2018/02/18(日) 14:28:33.81ID:???

3次元空間の中でも紙の面積は辺の2乗だ
その理由は紙の厚みを変えてないから
立方体のサイズを変えても高次元方向を変えなきゃ3乗なのさ
ご冗談でしょう?名無しさん [] 2018/02/18(日) 17:59:55.38:cYhbKNS3

ありがとう
自分には難しそうなので少し考えてから出直します
ご冗談でしょう?名無しさん [] 2018/02/18(日) 18:05:43.89:M+AkiTty

よいヒントをありがとう、考えてみます
ただ体積ではなく質量として質問したのは、質量なら便宜上4次元目の厚さを無視するみたいな誤魔化しが効かないと思ったから
なので面積での例えはまだちょっとイメージ掴めてない
ご冗談でしょう?名無しさん [] 2018/02/18(日) 18:31:32.80:M+AkiTty
電磁波について
「1.電磁誘導で発生する」「2.励起した電子が元の軌道に戻るときに発生する」というように覚えた記憶があるんですが、
1.の方は間違ってます?

電磁誘導で発生してしまうと、磁石を地球上で見た場合と月から見た場合で結果(電磁波だけでなくエネルギーとか質量とかも)が変わってしまうのかなと思ったので
ご冗談でしょう?名無しさん [] 2018/02/18(日) 21:11:14.52:K502gcsO
帯電させた棒などを水道の流水に近づけると流水が引き寄せられるそうですが
これは水分子が電気双極子で正と負の部分が分かれているためなのですか?そう説明しているブルーバックス本がありましたが・・・

近づける物がプラスに帯電してる場合
酸素原子側のマイナス帯電部分が引き寄せられても水素原子側のプラス帯電部分が反発する力と拮抗するのではないでしょうか?

近づける物がマイナスに帯電してる場合
水素原子側のプラス帯電部分が引き寄せられても酸素原子側のマイナス帯電部分が反発する力と拮抗するのではないでしょうか?
ご冗談でしょう?名無しさん [sage] 2018/02/18(日) 21:13:20.66ID:???

電気双極子の感じる電場に空間勾配があれば正味の力が残る
ご冗談でしょう?名無しさん [sage] 2018/02/18(日) 21:43:49.27ID:???

正負に分かれた電気量は等しいが、引き寄せられた異符号の電荷のほうが近いため、引力のほうが強くなる。
ご冗談でしょう?名無しさん [] 2018/02/18(日) 22:20:17.67:K502gcsO


早速の回答ありがとうございます。
個々の水分子内での負・正電荷部分の距離の差に基づくわずかな引力と斥力の差でもその分子1つを移動させるだけの力にはなるってことですかね〜
ご冗談でしょう?名無しさん [] 2018/02/18(日) 23:18:33.46:j7M83Z81
ピストンとシリンダーから成る系において
はじめピストンが固定されていて
固定を外し、ピストンが滑らかに動き気体が断熱変化を行なった際 圧力一定と見なせるのは何故なのでしょうか?断熱変化において定圧の条件は成り立たないのでは無いのでしょうか?
どうかご教授願います
ご冗談でしょう?名無しさん [sage] 2018/02/18(日) 23:21:35.83ID:???
それは嘘です
ご冗談でしょう?名無しさん [sage] 2018/02/19(月) 02:06:15.00ID:???

電磁気力は直感的な重力に比べて10^40も大きい
極々わずかな差でも重力より強い、静電気力や磁力が実例。
ご冗談でしょう?名無しさん [sage] 2018/02/19(月) 03:46:12.19ID:???

シリンダー内部の気体の圧力は変わると思うけれど、何の圧力の話かな?
もう少し状況を細かく〜
ご冗談でしょう?名無しさん [sage] 2018/02/19(月) 11:26:27.34ID:???
すまない質問です

voigtモデルにおいて時刻t=0において一定荷重Fをかけた場合、時刻tでの伸びδ(t)を求めよ(t=0でδ(0)=0)

って問題解ける方いらっしゃいますか?特性方程式かラプラス変換で解けるらしいのですが…
ご冗談でしょう?名無しさん [sage] 2018/02/19(月) 11:30:52.91ID:???
すまん
ご冗談でしょう?名無しさん [sage] 2018/02/19(月) 13:57:05.86ID:???

ttps://en.wikipedia.org/wiki/Kelvin%E2%80%93Voigt_material
ご冗談でしょう?名無しさん [] 2018/02/19(月) 14:28:59.54:4LHVkxLd
ある実験をA君がやると結果は〜正規分布(1.4, 0.1)に従い、
B君がやると結果は〜正規分布(1.6, 0.05)に従ったとき、
結果が1.84の場合は、B君がやった確率が高いのでしょうか?
ご冗談でしょう?名無しさん [] 2018/02/19(月) 14:39:11.25:HnNCXlIu
【砂糖玉】  動物や幼児にも効く  ≪≪≪ホメオパシー≫≫≫  被爆や霊障にも効く  【新常識】
ttps://rosie.5ch.net/test/read.cgi/liveplus/1519004807/l50
ご冗談でしょう?名無しさん [] 2018/02/19(月) 15:29:10.66:PzPvODgz
物理現象と心理現象を一緒にするのがユダヤ思想だ

例えば簡単な物理原理ですが

アルキメデスの比重の源となった浮力の原理ですが、

それから割り出された比重も永遠に成り立つのですよ。

人類や生命体が滅んでも木は水に浮きます。石は水に沈みます。

これを発見したアルキメデスは神に近き理性の持ち主です
ご冗談でしょう?名無しさん [sage] 2018/02/19(月) 15:30:07.50ID:???
すべてユダヤの陰謀です
ご冗談でしょう?名無しさん [sage] 2018/02/19(月) 18:04:44.11ID:???
ムニちゃーんポッポ
ご冗談でしょう?名無しさん [sage] 2018/02/19(月) 18:25:42.92ID:???
風と熱の違いってなんですか?
大学の説明会で教授に聞いたのですがテンソルが関係あると言っていました
高校生でテンソルはまだ習っておらず、調べてもちんぷんかんぷんでした
回答よろしくお願いします
ご冗談でしょう?名無しさん [sage] 2018/02/19(月) 19:14:17.14ID:???
風は空気の流れ、熱は分子の微視的運動エネルギーの伝播のことです
ご冗談でしょう?名無しさん [sage] 2018/02/19(月) 19:31:08.51ID:???
ベクトル場とスカラー場
ご冗談でしょう?名無しさん [] 2018/02/19(月) 23:07:45.20:MCbEj47N
NHK教育を見て54640倍賢く2355
ttp://nhk2.5ch.net/test/read.cgi/liveetv/1519048027/
255 [sage] 2018/02/20(火) 01:59:50.08ID:???

回答ありがとうございます
風はベクトル場で熱はスカラー場なんですね

回答ありがとうございます
風は運動エネルギーの伝播とは違うんですか?
質量を持つ風がある速度で吹いていたらその風は運動エネルギーを持っていますし、その風が空間中を移動してたら風も運動エネルギーを伝えていると考えて良さそうに思えます
ご冗談でしょう?名無しさん [sage] 2018/02/20(火) 02:42:19.56ID:???
エネルギーに着目すればそのようにすることも可能ですけど、風といったらあくまで空気の流れそのものを意味するわけで、エネルギーには着目していません
他の人が言ってますけど、風はベクトル場、すなわち空気の流れの方向によって表現されるわけですね
ご冗談でしょう?名無しさん [sage] 2018/02/20(火) 11:54:30.06ID:???

風船みたいな適当な空気塊をイメージしろ。
空気塊の全体質量、全体速度 (風速) に対して考えられる運動エネルギーが 風のエネルギー (U[風]) で、
全体速度を差し引いた(※) 内部分子個々の運動エネルギー総和が熱エネルギー (U[熱]) である。

現実は風船じゃないし構成分子はすぐに散り散りになってしまうが、ゴム膜があるとかないとかはどうでもいいのである。
瞬間毎に考えればよいので、仮想的に区切りをつけて考える事が大事。
空間各点にて極微小な空気塊を考える事で、風速分布(ベクトル場)、熱エネルギー密度分布(スカラー場) が決まる。

熱エネルギー密度から温度、温度から熱エネルギー密度が求まる。(統計力学)

※差し引かずにとにかく全分子の運動エネルギー総和を取ると
U = Σ 1/2 m[i] ( v[i] + V )^2
 = Σ 1/2 m[i] (v[i]^2 + 2 v[i]・V + V^2 )
= Σ 1/2 m[i] v[i]^2 + 0 + Σ 1/2 m[i] V^2 ( "差し引いた" 時の重心速度 V0 = Σ m[i]v[i] / M = 0 なので)
= U[熱] + U[風]
結局のこのように分離できる。
255 [sage] 2018/02/20(火) 13:07:40.38ID:???

回答ありがとうございました
熱運動をする粒子の集団を塊としてみたときの塊の運動が風にあたるのですね
熱運動は完全にランダムといえますが、この運動に指向性を与えれば熱運動のエネルギーの一部が風の運動エネルギーに変換されるのでしょうか?
ご冗談でしょう?名無しさん [sage] 2018/02/20(火) 13:46:42.47ID:???
> 完全にランダム
何をもってランダムと言うか.... 分子運動の方向分布は 「完全にランダム」(等方的) といっていいが、
その大きさ分布は マックスウェル分布 となっている。
大雑把に言うと、極端に運動エネルギーの大きい/小さい 分子は少ない適当なピークを持った分布で、温度が高いほどピーク位置は高エネルギー側にシフトする。
この辺りは 統計力学の入門編あたりで導出できるようになる。

> この運動に指向性を与えれば
分子の運動速度の大きさそのままで方向を揃える程度ならノーコストでエネルギー変換いけるのでは?という事ですかね。
自分には詳しく証明する能力ないけど、そういうのは熱力学の法則に反するので原理的にできないのだろうと思う。
SFの世界ですわ。
ご冗談でしょう?名無しさん [sage] 2018/02/20(火) 13:58:49.34ID:???
> この運動に指向性を与えれば
関連ワード: 「マックスウェルの悪魔」
ご冗談でしょう?名無しさん [sage] 2018/02/20(火) 14:26:03.98ID:???
通りがかりの渉ですが、何か?
ご冗談でしょう?名無しさん [sage] 2018/02/20(火) 21:25:27.35ID:???

[マックスウェルの悪魔]とかより「ファインマンのラチェット」のほうが物理解析し易い。
逆方向に回転できない歯車を分子スケールしたもので、回転方向の分子の衝突力で回転する。
実際に同様なナノマシンは作られているが、持続的に機能できない詳細はググレ。
255 [sage] 2018/02/21(水) 01:07:09.65ID:???



ありがとうございます
熱を風にしようとすると問題が生じるんですね
キーワードありがとうございました
調べてみます
ご冗談でしょう?名無しさん [sage] 2018/02/21(水) 01:29:29.79ID:???

大事なのは風とか暖かい空気とかが固まりで動く方じゃないのか?
この場合、まるで膜が存在するかのような振る舞いを示す方の説明が難しい。
その原理を教えないまま、運動量ガーと言っても説得力ないよ。
ご冗談でしょう?名無しさん [sage] 2018/02/21(水) 10:08:47.63ID:???
説得力ないと思ったんなら がちゃんとした説明をすればいいのでは?
ご冗談でしょう?名無しさん [sage] 2018/02/21(水) 13:08:09.89ID:???
以上に説明するのは無理だな
ご冗談でしょう?名無しさん [] 2018/02/21(水) 16:32:26.80:EmJxzCfs
高校物理の熱力学で、加熱されて温度が上昇した時は低圧変化か定積変化が起こるわけですが、体積も圧力も両方変化するケースって取り扱われると思いますか?
ご冗談でしょう?名無しさん [sage] 2018/02/21(水) 16:33:33.13ID:???
断熱変化で結構取り扱われていますね
ご冗談でしょう?名無しさん [] 2018/02/21(水) 16:54:23.92:Y5t1VB+b
力学の運動量原理と運動量保存則の違いを教えて下さい
ご冗談でしょう?名無しさん [sage] 2018/02/21(水) 17:02:49.20ID:???
運動量原理の式を関係する物体すべてについて足し合わせると運動量保存則になります
ご冗談でしょう?名無しさん [] 2018/02/21(水) 17:14:05.94:Y5t1VB+b

ありがとうございます
ご冗談でしょう?名無しさん [] 2018/02/21(水) 21:27:35.79:G33lmYvS

ベイズ統計的な話になり、きちんとした定式化が難しい
事前分布が半々、つまりA,Bのどちらがやりやすいという傾向もないと仮定する。
また確率分布が連続的だから単純にベイズの定理を持ってこれない。
ここでは、「平均からの外れ度合」を指標にする
1.84という値は、正規分布(1.4, 0.1)からすれば
平均からのズレは 0.44/0.1=4.4 シグマだ
正規分布(1.6, 0.05)からすれば
平均からのズレは 0.24/0.05=4.8 シグマだ
どっちにしろまずありえない外れ値が出ていることになるけど、
どうしてもAかBか選べと言われれば、
「まだ、Aがやったと考えたほうが妥当性は高い」かな
ご冗談でしょう?名無しさん [276] 2018/02/21(水) 21:32:20.48ID:???

ごめんミスった 平方根をとらないといけない
0.44/(0.1)^(1/2)=1.39
0.24/(0.05)^(1/2)=1.07
より、
「Bがやった蓋然性のほうが高い」
ご冗談でしょう?名無しさん [] 2018/02/21(水) 21:40:28.97:G33lmYvS

標準的な解釈ではそうなる(ボルンの射影規則)
もうちょっと正確に言えば、測定値の属する固有(ベクトル)空間
に射影した関数を規格化したもの、に変わる
(測定値を固有値とする従属でない固有関数が2個以上あるかもしれない
線形代数読むと良いよ)
ご冗談でしょう?名無しさん [] 2018/02/21(水) 21:51:30.28:qH9FzvZB
バークレー物理学コース 物理入門コース

どっちがおすすめかつ詳しいですか?
ご冗談でしょう?名無しさん [] 2018/02/21(水) 22:43:04.78:D9y64TaW
皆さんは量子コンピューターは実現できると思いますか?

ショアやグローバーのアルゴリズム勉強して凄いと思うんですが。
ご冗談でしょう?名無しさん [sage] 2018/02/22(木) 00:23:10.19ID:???
ムニぽぽだのん
ご冗談でしょう?名無しさん [sage] 2018/02/22(木) 08:16:20.00ID:???
ぺニスの膨張を物理モデルによって説明してください
239 [sage] 2018/02/22(木) 09:17:00.07ID:???

重力は他の「四つの力」に比べ非常に弱いという話につながるんですね!
超亀ですがありがとうございます。
ご冗談でしょう?名無しさん [sage] 2018/02/22(木) 09:50:57.74ID:???

他の本にしては?
271 [] 2018/02/22(木) 11:42:14.72:tszdsBsF
昨日は解答ありがとうございます。
もう一つお尋ねしたいのですが、高校物理の普通の力学において、
N=mgcosθという解法になる場合と、Ncosθ=mgという解法になる場合で、どういう違いが現象にあるのか、が分かりません。
どなたか御指南おねがいします。
ご冗談でしょう?名無しさん [sage] 2018/02/22(木) 12:03:36.02ID:???
解法がどうこうってのはわからないけど、座標軸がどの方向を向いているかの話じゃないの?

Ncosθ=mg って、どこで出てくるのかちょっと思いつかなかったけど。
(θ=Pi/2のとき抗力∞で装置壊れる?)
ご冗談でしょう?名無しさん [sage] 2018/02/22(木) 14:08:34.76ID:???
ムーニーちゃんしんぷだい
ご冗談でしょう?名無しさん [sage] 2018/02/22(木) 21:59:44.54ID:???
単振動の変位の一般解でサインだったりコサインで書いてある本両方存在するんですがどっちが正しいんですか?
x=Asin(ωt+Φ)
x=Acos(ωt+Φ)
ご冗談でしょう?名無しさん [sage] 2018/02/22(木) 22:12:30.93ID:???
両方正しいから好きな方使え
ご冗談でしょう?名無しさん [sage] 2018/02/22(木) 23:59:24.74ID:???

奇関数と偶感数どっちがお好きって感じかな?
A exp(-i(ωt+Φ))にすれば両方扱えるね。
ご冗談でしょう?名無しさん [sage] 2018/02/23(金) 00:05:55.42ID:???
変位が複素数っておかしくね?
ご冗談でしょう?名無しさん [sage] 2018/02/23(金) 00:29:33.13ID:???
波動方程式が線型なら気にしなくていい
ご冗談でしょう?名無しさん [] 2018/02/23(金) 00:57:35.89:iUbO6Ke1
初期位相入ってんのに偶関数も奇関数もねえだろ
ご冗談でしょう?名無しさん [sage] 2018/02/23(金) 01:14:47.95ID:???
強磁性イジング模型って|↑↑↑…>と|↓↓↓…>の二つの状態が縮退していますが
これらの線型結合が基底状態として実現しないのはなぜですか
ご冗談でしょう?名無しさん [sage] 2018/02/23(金) 02:04:26.13ID:???

いや,線形結合も基底状態だろ
295 [sage] 2018/02/23(金) 02:06:23.76ID:???
すまん
勘違い
ご冗談でしょう?名無しさん [sage] 2018/02/23(金) 02:22:11.44ID:???


ありがとうございます
どっちも正しいんですね
ご冗談でしょう?名無しさん [sage] 2018/02/23(金) 09:30:21.54ID:???
ムニちゃーんポポ
ご冗談でしょう?名無しさん [sage] 2018/02/23(金) 13:03:51.00ID:???
,
> 線形結合も基底状態
それで合ってるんだけど現実に生成、保持するのは極めて困難
近くの方位磁針(に相当するなにか) がちょっとでも反応したら、状態がどっちかに確定してしまうから。
シュレディンガーの猫より少しマシな程度
ご冗談でしょう?名無しさん [sage] 2018/02/23(金) 16:09:00.96ID:???

「実現」という言葉がよくありませんでしたが
純粋に理論的な話です
ご冗談でしょう?名無しさん [sage] 2018/02/23(金) 19:15:50.14ID:???
相変わらずここの回答者は低レベルだのぅ
答えは理論的に線形結合も基底状態だ
これで満足か?
ご冗談でしょう?名無しさん [sage] 2018/02/23(金) 20:08:50.88ID:???

のような線型結合で表される状態は全スピンがxy面内にも成分を持つことになりますが
直観的には奇妙な感じがします
イジング模型の特殊性でしょうか
ご冗談でしょう?名無しさん [sage] 2018/02/23(金) 21:06:23.82ID:???

それのどこが奇妙なのか分からない
磁場をかける前ならxyzに区別がないのだから当然だろ
形式的に方向を決めても重ね合わせの観測は容易にできる
ご冗談でしょう?名無しさん [sage] 2018/02/23(金) 21:10:58.31ID:???
全部のスピンがz↑の状態とx↑の状態が縮退してるなんて当然だろ
これが当然だと思うのは俺が賢いからか?
ご冗談でしょう?名無しさん [sage] 2018/02/23(金) 21:12:43.05ID:???

相互作用に異方性があります
ご冗談でしょう?名無しさん [sage] 2018/02/23(金) 21:15:35.41ID:???

イジング模型とハイゼンベルグ模型の区別が付いていないからだと思います
ご冗談でしょう?名無しさん [sage] 2018/02/23(金) 21:38:51.95ID:???
ちがわい ちがわい
ご冗談でしょう?名無しさん [sage] 2018/02/23(金) 21:46:37.48ID:???
考えてみると S > 1/2 であれば全 S^{x,y} は期待値ゼロですね
すみません
ご冗談でしょう?名無しさん [sage] 2018/02/23(金) 22:06:04.43ID:???

↑と↓だけの組み合わせなのだからイジング模型に他ならないが
重ね合わせでxになってもそれはイジング模型の範疇
z↑z↓の重ね合わせでxが作られるなんて量子力学の基本だろ
この形式的な軸の決定は物理的な異方性でないことを理解しないとな
ご冗談でしょう?名無しさん [sage] 2018/02/23(金) 22:12:25.01ID:???
イジング模型はz↑とz↓だけだがxyが重ね合わせで出てくるから、イジング模型はハイゼンベルク模型だ
こういうことを言いたかったのか
つまりイジング模型なんて虚構だと
ご冗談でしょう?名無しさん [sage] 2018/02/23(金) 22:14:32.73ID:???

すみませんが意味が分かりません
「範疇」とはヒルベルト空間のことですか
それともイジング模型に異方性はないという主張でしょうか
ご冗談でしょう?名無しさん [sage] 2018/02/23(金) 22:22:57.93ID:???

どこへ向けたレスか分かりませんが
少なくともそれは質問者の主張ではありません
ご冗談でしょう?名無しさん [sage] 2018/02/23(金) 23:34:37.06ID:???
イジング本は結局ターゲットがよくわからなかった

おもちゃとしては面白い本だったけど
ご冗談でしょう?名無しさん [sage] 2018/02/23(金) 23:39:14.20ID:???
>>312
重ね合わせが基底状態として実現しないのは何故かという質問自体が間違いと指摘したわけ
重ね合わせも基底状態だからな
で、重ね合わせだとxになるから奇妙という指摘に対して、本来xyzの異方性はないのだからどれが基底状態でも問題ないというわけ
寧ろ重ね合わせでxを作ったときにそれが基底状態にならない方が奇妙
電子一つのスピンをz↑↓と表現して異方性があるから重ね合わせのxがzと対等なのは奇妙レベルの暴論
ご冗談でしょう?名無しさん [sage] 2018/02/23(金) 23:50:52.03ID:???

熱力学極限では重ね合わせの状態は基底状態になりませんし
イジング模型はハイゼンベルグ模型と異なりSU(2)対称性を持たないので等方的ではありません
そうではないというあなたの主張と同様の主張をしている人は他にいますか?
ご冗談でしょう?名無しさん [sage] 2018/02/23(金) 23:56:58.10ID:???
の最後の文などはかなり分かりづらいですがこれら読む限り
もしかしてイジング模型の話だということをお忘れではありませんか
ご冗談でしょう?名無しさん [sage] 2018/02/24(土) 00:51:29.96ID:???
>>316
イジング模型の量子化軸をもって異方性というのは電子の量子化軸をもって異方性というようなもの
実際にイジングがxでもyでも問題ないことは電子同様重ね合わせで分かる
当然重ね合わせのxやyで議論していっても同じ結論が得られる
異方性は磁場をかけて初めて出てくるからな
その場合もzで議論して磁場をz方向にかけるのとxで議論して磁場をx方向にかけるのは全く同じになる
寧ろこのような量子化軸の設定を異方性と紹介している本を知りたい
ご冗談でしょう?名無しさん [sage] 2018/02/24(土) 00:54:34.70ID:???
そもそも異方性があるなら重ね合わせで基底状態にならない方向があるということか 
それとも励起状態だけ異方性があるのか
ご冗談でしょう?名無しさん [sage] 2018/02/24(土) 00:58:56.08ID:???

どんな磁性の本にも書いてあることですが
磁気異方性はゼーマン項や一イオン異方性だけでなく相互作用自体にも一般には存在します
イジング模型だけでなくXXZ模型やXY模型もそうです
あなたの仰っていることは H = JΣS^zS^z でも H = JΣS^xS^x でも本質的な違いはないということだと思いますが
このようなSU(2)対称性を持たない模型を等方的な模型であるとは普通言いません
量子化軸をどうとるかという問題ではないのです
ご冗談でしょう?名無しさん [sage] 2018/02/24(土) 01:01:24.85ID:???

事実として,熱力学極限における強磁性イジング模型の基底状態は|↑↑↑…>と|↓↓↓…>の重ね合わせにはなりません
それは「熱力学極限」の定義から自明です
ご冗談でしょう?名無しさん [sage] 2018/02/24(土) 01:05:35.16ID:???
それとでも述べましたがxy面内の成分はゼロでした
それは訂正します
ご冗談でしょう?名無しさん [sage] 2018/02/24(土) 03:25:00.88ID:???
>>320
磁気異方性の原因S・S'はなす角で決まるのであってS自身の方向は自由
Sの方向に向こうとするのが強磁性的でSがzかxかは自由
ゼーマン項はSに関する異方性があるので向きが決まるわけ
SS相互作用とは性質が異なる
これは電子の量子化軸の話と同じ
電子スピンをz↑↓として議論するから異方的なんて聞いたこともない
勿論x方向の磁場で縮退を解いて冷やせばz↑+z↓の基底状態が得られる
当然z↑が得られるなんてことは起きない
SS相互作用だけじゃスピンの方向なんて決まらない
ご冗談でしょう?名無しさん [sage] 2018/02/24(土) 07:27:00.06ID:???
かわいそうに
この人独学かな
勘違いばかりだ
ご冗談でしょう?名無しさん [sage] 2018/02/24(土) 10:22:00.92ID:???
電子の名前の由来は琥珀(古代ローマではelectrum)なんだってね
ご冗談でしょう?名無しさん [sage] 2018/02/24(土) 12:43:19.95ID:???

さすがに確信しました
あなたはハイゼンベルグ模型と他の模型の区別が付いていないようですね
もう結構です
他の回答者を待ちます
ご冗談でしょう?名無しさん [sage] 2018/02/24(土) 13:01:46.21ID:???
無駄無駄
ご冗談でしょう?名無しさん [sage] 2018/02/24(土) 14:12:55.34ID:???

線形結合も基底状態ですよね

> のような線型結合で表される状態は
> 全スピンがxy面内にも成分を持つことになりますが

持たないと思います
ご冗談でしょう?名無しさん [sage] 2018/02/24(土) 14:23:57.58ID:???

熱力学極限では線型結合は基底状態ではありません()

xy面内の成分を持たないのはその通りだと思います(>>321)
ご冗談でしょう?名無しさん [sage] 2018/02/24(土) 14:31:34.77ID:???
xy面内の成分は持ちませんがS^zの分散が非零になります
つまり基底状態で縦帯磁率が発散します
その結果としてz方向の完全偏極が実現する,ということでしょうか
ご冗談でしょう?名無しさん [sage] 2018/02/24(土) 21:10:01.63ID:???
ぷっちーポポ
ご冗談でしょう?名無しさん [] 2018/02/25(日) 09:09:42.17:Ajh/YUGz
世界中で、現実仮想世界をつくり、反重力で空を飛び、飛行機、船などで、移動し、ARMMOMMOシステムで、敵、モンスターなどを倒し
上級流通通貨金447kで、飯を食い、宿を取り、結婚し、現実世界と仮想現実を繋ぎ、ゲーム世界を行き来する、星々を行き来する、移住もあり、ARMMOMMOゲーム。もちろん、日常の学校に行ったりする。
上記 登記
ご冗談でしょう?名無しさん [] 2018/02/26(月) 00:58:27.30:MmZdFedS
100円ショップのコイン型ネオジム磁石をドライバーにこすりつけたら着磁できるのでしょうか?
磁力の方向が一定にならないから着磁はできませんか?
ご冗談でしょう?名無しさん [sage] 2018/02/26(月) 01:43:32.95ID:???
普通に磁化すると思うが
ご冗談でしょう?名無しさん [] 2018/02/26(月) 11:36:17.61:gRpvOGDb
(気体においての粘性式 サザーランドの式 1893年)についてですが、
気体の粘度が、気体の密度に依存しないのは何故でしょうか?
密度が高い方が、運動量を運ぶ粒子が多いから粘度が大きい筈だと思っていたのに
粘度が密度に依存しないと言うのが理解できません。

粘度
ttps://ja.wikipedia.org/wiki/%E7%B2%98%E5%BA%A6
ご冗談でしょう?名無しさん [sage] 2018/02/26(月) 11:37:37.69ID:???
24
ご冗談でしょう?名無しさん [sage] 2018/02/26(月) 11:39:16.69ID:???

効率的に磁化する方法
1)磁石を動かすべき
2)時間をかけるべき
3)引っ付けるだけ
4)それ以外
どれだと思う?
ご冗談でしょう?名無しさん [sage] 2018/02/26(月) 11:48:54.06ID:???
磁石なっているドライバーを買ってくる
ご冗談でしょう?名無しさん [sage] 2018/02/26(月) 12:25:03.11ID:???

カツンカッツーンと軽く衝撃を与えるといいよ
ご冗談でしょう?名無しさん [sage] 2018/02/26(月) 12:36:57.19ID:???
ドライバーが知らんうちに磁化してて困る
ご冗談でしょう?名無しさん [sage] 2018/02/26(月) 12:51:26.92ID:???
キュリー点以上に熱すれば磁化は消えるのでは
ご冗談でしょう?名無しさん [sage] 2018/02/26(月) 17:09:17.82ID:???
磁石をこすりつけてると磁石の成分が鉄に溶けだすから
鉄属性が磁石属性に変化しやすくなる
ご冗談でしょう?名無しさん [sage] 2018/02/26(月) 17:44:18.84ID:???
ムニちゃーんポポ
ご冗談でしょう?名無しさん [sage] 2018/02/26(月) 17:50:24.48ID:???

これ何なの?
ご冗談でしょう?名無しさん [sage] 2018/02/26(月) 18:10:39.96ID:???
しっ、目を合わせちゃいけません
ご冗談でしょう?名無しさん [sage] 2018/02/26(月) 21:58:07.05ID:???
脱着式のドライバー用マグネットまであるんだね〜
いろいろ考える人いるんだなぁ。
ご冗談でしょう?名無しさん [] 2018/02/27(火) 09:57:20.14:FFbCAT43
TA-68が上級、不老不死の薬、

若返りの薬 TA‐65 TA−68 の順に入れて、半不死身の薬

若返りの薬 TA‐68 TA−68 の順に入れて、半不死身の薬

若返りの薬 TA−65 TA‐74 の順に入れて、半不死身の薬上級

上記 登記
ご冗談でしょう?名無しさん [sage] 2018/02/27(火) 13:13:54.17ID:???

冷ます時に磁場があれば元の木阿弥
ご冗談でしょう?名無しさん [sage] 2018/02/27(火) 17:30:13.10ID:???
お前らの好きな電池こと劣等感婆の実力

大学学部レベル質問スレ 9単位目
ttps://rio2016.5ch.net/test/read.cgi/math/1513222085/582-1000
ご冗談でしょう?名無しさん [] 2018/02/27(火) 20:19:56.99:s89q4oMd
電線a に交流電流を流すと、電磁波a が発生する。
別の電線b を密着させて同振幅・同周期の電流を流し、電磁波b を発生させる。
二つの電線は極めて細いので実質的に同じ空間位置と見なせる。
電磁波のエネルギーは振幅の二乗に比例する。

電磁波a と電磁波b が逆位相のとき、
電磁波は破壊干渉して振幅が 0、エネルギーも 0になる。
この分のエネルギーは何処に消えたのか?

電磁波a と電磁波b が同位相のとき、
電磁波は建設干渉して振幅が 2倍になり、
エネルギーは 2²=4倍になる。
この分のエネルギーは何処から湧いて出たのか?

が何処かで勘違いしてることは確かだが、
いったい何処で勘違いしてるんだろうか?
ご冗談でしょう?名無しさん [sage] 2018/02/27(火) 20:46:53.29ID:???
電流の帰線はどこにあるんだろうか
そもそも電磁波が発生するんだろうか
発生したとしてそのエネルギーはどこから来るんだろうか
ご冗談でしょう?名無しさん [低能な書き込みするなボケ!] 2018/02/27(火) 23:03:50.90ID:???
>二つの電線は極めて細いので実質的に同じ空間位置と見なせる。

二つの電線は(観測者のサイズによっては)極めて細いので、
(観測者の位置によっては)実質的に同じ空間位置と見なせる。
ご冗談でしょう?名無しさん [sage] 2018/02/28(水) 00:36:40.39ID:???
ムーニーちゃんしんぷだい
ご冗談でしょう?名無しさん [sage] 2018/02/28(水) 01:18:43.74ID:???

水面に木の板を浮かせて屈伸運動をする。
板が上下するので周囲に波が生じる。
このとき、水中で反対向きの力をかける人がいたとする。
理論的には板は動かず、波は生じない。陸上で屈伸しているのと同じだ。

じゃあ、屈伸運動をするのはどちらが楽か? 当然、板が動かない方が
楽だ。周りの水を動かすエネルギーが不要だからだ。2つの導線で消費
される電力は2つ逆位相の電流が流れている方が少ないと言える。
ご冗談でしょう?名無しさん [sage] 2018/02/28(水) 01:30:56.56ID:???

同じじゃね?
エネルギーが系の外に放出され続けるか、内部に蓄積され続けるかの違い。
系といっても、領域は無限だがな。
ご冗談でしょう?名無しさん [sage] 2018/02/28(水) 07:20:12.15ID:???
電磁波が打ち消して空間に放射されなくても不思議でも何でもない
電磁波源の内部熱に変換されるだけ、エネルギー放射効率ゼロ。
ご冗談でしょう?名無しさん [sage] 2018/02/28(水) 10:53:33.28ID:???

互いに、一方の電線が発した電磁波により、他方の電線に交流電流が流れ易くなる。
即ち、エネルギーは、互いに他方の電線に電流を流すことに消費される、とか?
ご冗談でしょう?名無しさん [sage] 2018/02/28(水) 11:17:21.57ID:???

等価回路で考えてみると
電磁エネルギーを絡め取る場所(L, C) 、消えてく場所(r)が共通なので
同位相なら単に線の太さを二倍にしただけ。
逆位相なら電場(E)と磁場(H)はそれぞれで打ち消し合って電磁波は放出されず、
単に内部抵抗(R)による発熱にエネルギーが消費されるだけだと予想できる。

二電線が離れている場合とはやはり違うのである。
電子回路のトランスも磁場を絡め取る場所が共通なので相互インダクタンスなんてのを考えるんだな。
電線が中途半端に離れている場合は、磁場漏れのあるトランスと同じような解析になるだろう。
ご冗談でしょう?名無しさん [] 2018/02/28(水) 12:28:11.54:hNG7mHlp
【元国連】  笑うセールスマンの声で   ≪テレパシー?≫  聞こえないふりをするな  【非言語】
ttp://rosie.5ch.net/test/read.cgi/liveplus/1519784789/l50
ご冗談でしょう?名無しさん [sage] 2018/02/28(水) 12:50:20.86ID:???
逆に考えると、逆方向電流で電磁波が放射されないってことは電流もないってことだな
相互誘導で電流消去なんだろう
実質的な絶縁体になってしまうわけだ
ご冗談でしょう?名無しさん [sage] 2018/02/28(水) 13:34:07.15ID:???
(理想的な)同軸ケーブルは外部に電磁波が放射されない。
外部に放射される電磁波を打ち消すようにしてロスを抑制する設計。
ご冗談でしょう?名無しさん [sage] 2018/02/28(水) 13:55:12.10ID:???
東京大学理学部数学科を目指そうかな・・・・・。
歳をとってから東京大学を目指すってのはやっぱりいろんな意味で苦行でしか無いのでしょうか?
歳をとってから大学に入る人は、よっぽどしっかりしてないと、歳下から馬鹿にされたりするだけであって、
辛い思いをするだけなのでしょうか?
ご冗談でしょう?名無しさん [sage] 2018/02/28(水) 15:34:03.01ID:???
またレス乞食ヒマラヤ
ご冗談でしょう?名無しさん [sage] 2018/02/28(水) 15:38:53.73ID:???

ちゃんとレスしてくれるのはありがたい。
電磁波は遠隔効果なのか近接効果なのかが問題だが、
きちっと計算すると変動する電場は両方あるんだよね。

1)電磁波を発振するぐらいのエネルギーが回路に流れない
2)いや、流れるが空中線で熱になる
3)空間が温まる

どれかな?
ご冗談でしょう?名無しさん [] 2018/02/28(水) 16:18:11.16:NWL0VPd9
癌に有効に効く。
多少お兄ちゃんのつくった薬効くから60歳が20歳に若返る薬「NMN」! ネットで大反響「いつ買えるのか」について、製造研究元に聞いたttps://www.excite.co.jp/News/odd/Tocana_201501_post_5522.html …不老不死の薬、絶賛販売中!?
その薬”TA65”の効果や主成分とは?ttps://t.co/ruObSax3VL
上記 登記 👀
Rock54: Caution(BBR-MD5:b73a9cd27f0065c395082e3925dacf01)
ご冗談でしょう?名無しさん [sage] 2018/02/28(水) 16:53:35.90ID:???

どれでもない。

外部の電磁波が干渉して消えるような状況のときはその分電源が供給するエネルギーは少なくなっている。
逆に言うと、外部に電磁波を出してしまうような状況のときはその分のエネルギーを電源が供給している。
2本線の場合に電源が供給しているエネルギーは単線の場合の単純に2倍であるという仮定が間違っている

それだけ
ご冗談でしょう?名無しさん [sage] 2018/02/28(水) 18:22:43.57ID:???

デタラメな絵の回路では1本でも2本でも電磁波がまともに放射されない、出ても微々たる比率。
ほとんどミスマッチで反射され定常波になる、エネルギーは伝搬されず回線の損失熱になる。
ご冗談でしょう?名無しさん [sage] 2018/02/28(水) 18:37:55.54ID:???

>電磁波は遠隔効果なのか近接効果なのかが問題だ
電気理論を全く理解してなくても「電磁波」は知ってるらしいな
コイルに電流を流すと磁場が発生するくらいは中高で習うから知ってるよな
折り返した二本の線で密にコイルを巻けば磁場が殆ど打ち消され、無いと同じになる。
ご冗談でしょう?名無しさん [sage] 2018/02/28(水) 20:02:26.08ID:???

交流波の場合は1本線でも電磁波は放射されるよ。磁界が変動するんだからどうしたって電場も変動するんだよ。
そして電磁波が運ぶエネルギーは ポインティングベクトル( E × H ) で表せるから、内部抵抗による発熱以外にもエネルギーは出て行くよ。
微々たる比率だなんてのは知ってる。

直流の場合でも静磁界を形成するためには最初だけエネルギーが必要だよ。
もちろん E × H = 0 だから、静磁界の部分はエネルギーを運ばないよ。
どういう事かというと最初だけ電磁的な衝撃波が形成されて変動が無限遠まで伝わる、変動の先端部が去った後に静磁界が残るってわけ。
衝撃波生成に必要なエネルギーは、よく知られた静磁界のエネルギー密度を空間積分すれば 求まる。

それから、伝送線路/導波路における等価回路はふつうによく出てくる概念だ。デタラメなポンチ絵じゃねーし。
ご冗談でしょう?名無しさん [sage] 2018/02/28(水) 20:16:02.23ID:???
電磁波のエネルギーが注目される理由は、系の外側まで運べる波だから。
距離の逆自乗関数で表されるエネルギーは面積分で定数化し、
無限遠まで伝わっても、0に収束しないのだ。
ところが、近傍に互いに逆位相の電磁波源対があれば、0に収束する。
電磁波エネルギーが系の外に放出されなくなるわけだ。
ご冗談でしょう?名無しさん [sage] 2018/02/28(水) 20:59:30.48ID:???
角運動量の計算についてなのですが、自分の計算では教科書のようにならなかったので間違いを教えてください

ttps://i.imgur.com/Gf5DO1p.jpg
ttps://i.imgur.com/jwISerb.jpg
ご冗談でしょう?名無しさん [sage] 2018/02/28(水) 21:24:16.41ID:???

教科書が間違ってます
ご冗談でしょう?名無しさん [sage] 2018/02/28(水) 21:27:51.96ID:???

ただ、自分のだと回転運動方程式が成り立ちません
ご冗談でしょう?名無しさん [sage] 2018/02/28(水) 21:30:43.86ID:???

慣性モーメントの定義は、回転軸からの距離の2乗×質量の積分ですからあなたのが正しいんです
ご冗談でしょう?名無しさん [sage] 2018/02/28(水) 21:43:46.43ID:???

計算が正しいのはわかったのですが、自分の式から回転運動方程式を立てる方法を教えてください
ご冗談でしょう?名無しさん [sage] 2018/02/28(水) 22:19:14.24ID:???

それは1)だ。電磁波の分のエネルギーは最初から必要ではないの意味。
日本の学問における最大問題は日本語の読解力の問題だ。
ご冗談でしょう?名無しさん [sage] 2018/02/28(水) 22:22:58.23ID:???

クーロンの法則を拡張して、時間変化する電荷の時の
クーロンの法則を書いて見せてみ。知識もないのに偉そうに書くなよ。
恥をかくだけだぞ wwwww
ご冗談でしょう?名無しさん [sage] 2018/02/28(水) 23:44:42.17ID:???
↑基地
ご冗談でしょう?名無しさん [sage] 2018/02/28(水) 23:46:19.93ID:???
くっくっく
ご冗談でしょう?名無しさん [sage] 2018/03/01(木) 00:23:32.77ID:???

教科書で位置ベクトルと速度ベクトルの内積が0になってる理由を考えてみよう
ご冗談でしょう?名無しさん [sage] 2018/03/01(木) 01:32:36.79ID:???

回答ありがとうございます。カッコ内の第二項には速度ベクトルV=ω×rはないのですが、第二項が0になる理由がわからないので教えてください。
ご冗談でしょう?名無しさん [] 2018/03/01(木) 05:11:55.84:wnZCeHHw
 若返りの薬 TA‐68 TA−68 の順に入れて、半不死身の薬


若返りの薬 TA−65 TA−68 TA‐74 の順に入れて、半不死身の薬上級

上記 登記
ご冗談でしょう?名無しさん [sage] 2018/03/01(木) 08:00:51.32ID:???

第二項はゼロになりません。教科書が間違っています。
ご冗談でしょう?名無しさん [sage] 2018/03/01(木) 08:46:12.05ID:???

その腐った本を投げ捨てて
ttp://www.asem.kyushu-u.ac.jp/qq/qq02/kikanbuturi/chap12.pdf
でも呼んだ方がいい。
ご冗談でしょう?名無しさん [sage] 2018/03/01(木) 10:01:26.54ID:???

書いたやつの日本語表現力を棚にあげて、何とか読もうとしたやつの読解力の問題にするのは卑怯だな
ご冗談でしょう?名無しさん [] 2018/03/01(木) 10:19:45.20:wnZCeHHw
若返りの薬 TA‐68 TA−68 の順に入れて、半不死身の薬


若返りの薬 TA−65 TA−68 TA‐74 の順に入れて、半不死身の薬上級


若返りの薬 TA−65 TA−68 TA‐74  TA-76 の順に入れて、プロトタイプ半不死身の薬上級(寿命は3分の2)


若返りの薬 TA−65 TA−68 するめ の順に入れて、半不死身の薬上級

若返りの薬 TA−65 TA−68 TA‐74 するめ の順に入れて、プロトタイプ半不死身の薬上級

上記 登記
ご冗談でしょう?名無しさん [] 2018/03/01(木) 10:47:36.80:wnZCeHHw
若返りの薬 TA‐68 TA−68 の順に入れて、半不死身の薬


若返りの薬 TA−65 TA−68 TA‐74 の順に入れて、半不死身の薬上級


若返りの薬 TA−65 TA−68 TA‐74  TA-76 の順に入れて、プロトタイプ半不死身の薬上級(寿命は3分の2)


若返りの薬 TA−65 TA−68 するめ の順に入れて、半不死身の薬上級

若返りの薬 TA−65 TA−68 TA‐74 TA-82 TA-84 するめ の順に入れて、プロトタイプ半不死身の薬上級(寿命は3分の1)

若返りの薬 TA−65 TA−68 TA‐74 TA-84 TA-86 するめ の順に入れて、プロトタイプ半不死身の薬上級(寿命は3分の1)

牛乳でつくったやつで寿命は伸ばせばいい。

上記 登記
ご冗談でしょう?名無しさん [] 2018/03/01(木) 11:04:20.59:wnZCeHHw
若返りの薬 TA‐68 TA−68 の順に入れて、半不死身の薬


若返りの薬 TA−65 TA−68 TA‐74 の順に入れて、半不死身の薬上級

若返りの薬 TA−65 TA−68 するめ の順に入れて、半不死身の薬上級

若返りの薬 TA−65 TA−68 TA‐74 するめ の順に入れて、半不死身の薬上級


若返りの薬 TA−65 TA−68 TA‐74  TA-76 の順に入れて、プロトタイプ半不死身の薬上級(寿命は3分の2)


若返りの薬 TA−65 TA−68 するめ の順に入れて、半不死身の薬上級

若返りの薬 TA−65 TA−68 TA‐74 TA-82 TA-84 するめ の順に入れて、プロトタイプ半不死身の薬上級(寿命は3分の1)

若返りの薬 TA−65 TA−68 TA‐74 TA-84 TA-86 するめ の順に入れて、プロトタイプ半不死身の薬上級(寿命は3分の1)

牛乳でつくったやつで寿命は伸ばせばいい。

上記 登記
ご冗談でしょう?名無しさん [sage] 2018/03/01(木) 12:26:45.10ID:???

このPDFでも第二項は無視してたのでよく見たら角運動量のZ軸方向についてのみ取り上げられていました。確かに成分で計算するとZ軸方向はr^2ωになりました。ありがとうございました。
ご冗談でしょう?名無しさん [sage] 2018/03/01(木) 12:55:53.55ID:???

これ、誰の何ていうタイトルの本?
ご冗談でしょう?名無しさん [sage] 2018/03/01(木) 13:19:08.74ID:???
自分の字が汚くて間違えただけやん
ご冗談でしょう?名無しさん [] 2018/03/01(木) 13:22:56.21:wnZCeHHw
ベクトル スクエア
不死身だと倒せない相手がいる。使えない技がある。
ご冗談でしょう?名無しさん [sage] 2018/03/01(木) 13:26:25.86ID:???

実教出版/鈴木浩平/機械力学です

自分の式なんか間違ってました?
ご冗談でしょう?名無しさん [sage] 2018/03/01(木) 15:13:25.48ID:???
ムーニーちゃんしんぷとセバスチャンしんぷはどっちの方が凄いですか?
ご冗談でしょう?名無しさん [sage] 2018/03/01(木) 18:09:53.17ID:???

なりません
ベクトルrもz方向持ってますよ
翁A [sage] 2018/03/01(木) 18:47:34.64ID:???

計算は間違ってないけど 何を計算すべきかが違っているのでは
r×dm vは原点まわりの角運動量ベクトル
r^2 dm θdot e_zは指定軸まわりの角運動量ベクトル
両者は別物です
本の著者のしと たね本から写すときにうっかり間違えたものとおもわれ
ご冗談でしょう?名無しさん [sage] 2018/03/01(木) 19:37:15.00ID:???
固定軸の回りの角運動量を計算するときに、何も分かってない奴は原点からの距離を使ってしまう。そういう奴が本を書くこともある。
翁A [sage] 2018/03/01(木) 19:45:14.99ID:???
「全」を完全消滅させたらどうなりますか?
ご冗談でしょう?名無しさん [sage] 2018/03/01(木) 20:29:17.17ID:???

PDFの方でやってみました。位置ベクトルr=(rcosθ,rsinθ,z)となっていたのでこれとベクトルω=(0,0,ω)とで成分で計算すると、確かに角運動量のZ成分はmr^2ωになりましたしZ軸上からの距離もこのベクトルrの起き方だとrになりましたのでL=Iωは成立しました。
ただこの教科書のベクトルrの置き方だとこのようにはなりませんね...
ご冗談でしょう?名無しさん [sage] 2018/03/01(木) 20:32:10.82ID:???

どの軸周りかに注意していきます
395 翁A [sage] 2018/03/01(木) 20:32:20.00ID:???
現れたな、なりすまし。 はBな
Aはワシじゃ
ご冗談でしょう?名無しさん [sage] 2018/03/01(木) 21:22:17.43ID:???

の本は捨てた方がいいと思う。冗談ではなくて。
ご冗談でしょう?名無しさん [sage] 2018/03/01(木) 21:36:50.58ID:???


おすすめの機械力学の本教えてください
395 翁A [sage] 2018/03/01(木) 21:40:48.79ID:???
「無」は頂点ですか?
ご冗談でしょう?名無しさん [sage] 2018/03/01(木) 21:53:32.29ID:???

機械工学のための力学 (JSMEテキストシリーズ)

もっと色々知りたければ物理学科向けの連続体力学の本
ご冗談でしょう?名無しさん [sage] 2018/03/01(木) 22:00:56.37ID:???

すまん、オレの間違いや
ご冗談でしょう?名無しさん [sage] 2018/03/01(木) 22:05:41.56ID:???

機械工学のための〜は持ってますね、指定教科書でした。見かけによらずかなり詳しいのですが他の分野でもJSMEシリーズの物は良書なのですか?
ちなみに写真の教科書は振動学メインで動力学は申し訳程度でした...
ご冗談でしょう?名無しさん [sage] 2018/03/01(木) 22:38:39.64ID:???
角運動量: L = ∫dv ρ r × v = ∫ dv ρ r × (ω × r) = ∫ dv ρ ( r^2 ω - (r・ω) r )
よって成分表示で
L[i] = ∫ dv ρ ( r[k]r[k] δ[i,j] - r[i]r[j] ) ω[j] (同じ添え字は和を取るものとする)
慣性モーメント: I[i,j] = ∫ dv ρ ( r[k]r[k] δ[i,j] - r[i]r[j] ) と置けば
L = I ω と書ける。
この I は対称行列なので座標を適当に回転すれば対角行列になる (後は省略)

の著者、その辺のこと全然分かってないと思う。
でなけりゃあれで慣性モーメントについて「詳しく示す」なんて書けないよ...
ご冗談でしょう?名無しさん [sage] 2018/03/01(木) 22:42:51.87ID:???
あーすまん、次のページにはちゃんと書いてあるのかもしれないな
ご冗談でしょう?名無しさん [sage] 2018/03/01(木) 22:43:26.78ID:???

おまえもな www
ご冗談でしょう?名無しさん [sage] 2018/03/02(金) 00:25:34.53ID:???
でぼぼぼーん
ご冗談でしょう?名無しさん [] 2018/03/02(金) 02:17:36.21:IMITnCSD
質問しまーす!

この板の流儀にしたがって
c=3*10^8m/s=30万km/s
v=0.866*cのとき、正確にγ=1/√(1-v^2/c^2)=2となるとします。
L0=0.866*c*1秒≒26万kmとして、↓のコースをv=0.866*cで回ってくる
ことを考えます。

地球では行き1秒ターン1秒帰り1秒=トータル3秒のはず。
宇宙船の中の時計は地球から見ていてΔt'=1/2・Δtだから1.5秒経っているはず。
逆に宇宙船から見たとき、自分の時計は行き0.5秒ターン0.5秒帰り0.5秒
ここまではOK

ここからが疑問、
宇宙船から地球の時計を見ると、等速直線運動の時Δt=1/2・Δt'だから、
行き0.25秒帰り0.25秒、
最後に相対性理論を成り立たせるためにターン2.5秒でOKでしょうか?
ご冗談でしょう?名無しさん [] 2018/03/02(金) 02:30:35.88:IMITnCSD
の続き
もう一つ質問、
もし行き帰りの距離が2倍の2*L0とします。
地球から見たとき
地球の時計 Δt=5秒、宇宙船の時計Δt'=2.5秒
宇宙船から見た時
宇宙船の時計 Δt'=2.5秒
地球の時計 Δt=行き1秒+行き1秒+ターン3秒のはず、
と違ってターンの時間が3秒、同じターンをしているのに
距離によって地球の経過時間が変わっていいのでしょうか?
ご冗談でしょう?名無しさん [] 2018/03/02(金) 02:44:56.06:IMITnCSD
の続き
もう一つ質問、
ターンしているときに宇宙船からターンの半径を光を使って計測したとき
r=26/3.14≒8.3万kmが正しいのでしょうか?
それとも
r=26/2/3.14≒4.1万kmが正しいのでしょうか?
お願いします。
ご冗談でしょう?名無しさん [] 2018/03/02(金) 03:02:19.48:IMITnCSD
の訂正

地球の時計 Δt=行き1秒+行き1秒+ターン3秒のはず、


地球の時計 Δt=行き0.5秒+帰り0.5秒+ターン4秒のはず、
ご冗談でしょう?名無しさん [sage] 2018/03/02(金) 03:47:18.92ID:???
天才数学者と天才画家はどっちの方が真の天才と言えますか?
ご冗談でしょう?名無しさん [sage] 2018/03/02(金) 08:36:46.73ID:???

OKです


いいんです


加速度運動をしている宇宙船にとって光を使っての計測は単純な計算はできないでしょうが、
宇宙船と円軌道の中心点との距離は8.3万kmと考えていいと思います
ちなみに円軌道は、宇宙船にとって常に進行方向に潰れていますから真円ではありません
ご冗談でしょう?名無しさん [] 2018/03/02(金) 09:28:15.93:OUHe0rWw
基礎的な力学の質問です

底部におもりを付けた中空円柱が鉛直に液体に浮かんでいて、それを液中に押し込んで離すときの振動の周期を求めろという問題です
中空円柱の質量と液体及び空気による抵抗は無視します
押し込む力をF、液体の密度をρ、円柱底部のおもりの質量をm、円柱の断面積をS、重力加速度をgとします

解説を読むと最初に「円柱がxだけ沈んだとするとアルキメデスの原理によりF=ρxSgとなる」として
そのあと円柱の加速度をaと置いてma=mg-ρxSgなどなどと解いていますが
まず何故浮力=押し込んだ力なんですか?
押し込んだ時点で力が釣り合っているというのなら何故重力のmgは無視されているのですか?
下向きを正と取るならばF=ρxSg-mgなのでは?

ma=mg-ρxSg以降は分かるのですが最初の設定が分かりません
よろしくお願いします
ご冗談でしょう?名無しさん [sage] 2018/03/02(金) 11:02:00.13ID:???
解説の日本語がおかしいか、読んだ側の日本語解釈がおかしい
アルキメデスの原理から出てくるのは浮力

日本語が変だから写真でもアップしてくれた方がいい
ご冗談でしょう?名無しさん [sage] 2018/03/02(金) 11:11:11.76ID:???

>同じターンをしているのに
>距離によって地球の経過時間が変わっていいのでしょうか?
一般相対論的に言うと、重力ポテンシャルの違いが時間経過の違いになる。
加速度運動する宇宙船から見ると地球までの距離が遠ければ
その分重力ポテンシャルの差が大きくなるから時間の経過も違ってくる。

特殊相対論的に言っても、加速度運動は慣性系の乗り換えであり、その際に遠方の
時間にジャンプが発生する。それも距離に依存するから、やはり距離が違えば
時間経過が変わる

いずれにしても、同じターンでも距離によって地球の経過時間が変わっていい
ご冗談でしょう?名無しさん [sage] 2018/03/02(金) 11:16:57.83ID:???
ここまでの流れ読んでないけど
特殊相対論で済むのは地球側だけで、そこから見た宇宙船の固有時に距離は関係ないぞ
ご冗談でしょう?名無しさん [sage] 2018/03/02(金) 11:18:22.62ID:???

>そのあと円柱の加速度をaと置いてma=mg-ρxSgなどなどと解いていますが
円柱に働く力は下向きの重力mgと上向きの浮力ρxSgで、
下向きを正とすればまさにその運動方程式になる

>まず何故浮力=押し込んだ力なんですか?
何に対する疑問なのか不明。少なくともに説明されている限りではそんなの関係ない。
別のところに書かれている話なら、その説明がなければ理解できない

>押し込んだ時点で力が釣り合っているというのなら何故重力のmgは無視されているのですか?
これも同じ。何の話をしているのか説明がなくて理解できない

>下向きを正と取るならばF=ρxSg-mgなのでは?
上記の説明のとおり、逆です。F=mg-ρxSgで正しい
ご冗談でしょう?名無しさん [sage] 2018/03/02(金) 11:20:54.76ID:???

宇宙船から見た地球の経過時間の話ね。
ご冗談でしょう?名無しさん [sage] 2018/03/02(金) 11:24:04.56ID:???

「特殊相対論的に」のくだりは宇宙船の静止系でそもそも特殊相対論が使えないことと
特殊相対論のローレンツブーストに距離が関係ないことを考えるとおかしいと思うんだけど
俺間違ってる?
ご冗談でしょう?名無しさん [sage] 2018/03/02(金) 11:52:30.64ID:???

加速度運動は特殊相対論では使えない、というのはありがちな間違い。
ただの加速度運動なら特殊相対論は使える。
使えないのは真の重力場があって時空の曲率が0でない場合。

双子のパラドックスを宇宙船の側から説明するときにもよく出てくるから調べてみ。
キーワードはにも書いたけど慣性系の乗り換え
413 [] 2018/03/02(金) 12:17:08.04:30kS0dFp

例えばこの方法で半径を測るとします。
1. ターンの入り口で星の中心に向けて光を放射する。
2. 星の中心に鏡がおいてあり全ての方向に光が放射される。
3. 宇宙船が移動した先でこの光を受け取った時間を記録する。
この方法でいうと、Δt'=2r'/c、r'=cΔt'/2のはずです。Δt'が
1/2・Δtであればr'=1/2 rを思われますが、どうでしょうか?
412 [] 2018/03/02(金) 12:23:10.96:30kS0dFp

なるほど、この星の重力場の影響が距離によって違うから、
地球での経過時間も違うという説ですね。
1. 26万kmと52万kmの差で2.5秒の時間経過と4秒の
時間経過が説明できるでしょうか?
2. この星の重力場は0で、外向きのロケットの噴射でターン
しているとしても同じでしょうか?
422 [] 2018/03/02(金) 12:35:51.41:30kS0dFp

宇宙船がターンしているときの計算は局所慣性系を適用して、
dt'=1/2・dtとしています。
ご冗談でしょう?名無しさん [] 2018/03/02(金) 12:36:54.11:30kS0dFp

失礼、412です。
ご冗談でしょう?名無しさん [sage] 2018/03/02(金) 13:06:24.66ID:???

「加速度運動が使えない」じゃなくて加速系では使えないって言ってるんだけどな
曲率ゼロなら特殊相対論が使えるってよく言うけどそれ一般相対論を暗に使ってない?
どういう理由で特殊相対論的効果だと言ってるの?
ご冗談でしょう?名無しさん [] 2018/03/02(金) 14:15:43.55:30kS0dFp
が正しければ、ターンのところだけ取り出して↓でOK?
ご冗談でしょう?名無しさん [sage] 2018/03/02(金) 15:27:08.36ID:???

>「加速度運動が使えない」じゃなくて加速系では使えないって言ってるんだけどな
言葉が足りなかったけどその意味で指摘してる。
加速度系は瞬間瞬間で速度が違うような慣性系への
ローレンツ変換を連続適用することで扱える。

その昔、原子核の周りを回る電子の静止系(当然加速度系だね)で考えたとき
電子のエネルギー準位が実験値と2倍違うという問題があったのを、上記の考えで
きちんと扱ってみるとトーマス歳差というそれまで知られていなかった効果が出てくることに気づき、
見事に2倍の食い違いを解消した、という古典的な事例もある

まぁとりあえずは、双子のパラドックスを宇宙船の側から説明する解説を探してみ
ご冗談でしょう?名無しさん [sage] 2018/03/02(金) 15:42:56.31ID:???

面白いな。
大きさのない素粒子はそれでいいんだが、回転座標系でrやθが変わると曲率の問題が
発生し成り立たないんだよね。大きさのあるものは特殊相対論では扱えないんだよね。
アインシュタインは簡単な回転運動で特殊相対論が成り立たないことが分かるんだけれど、
その延長としての重力理論で重力の本質については何も語っていないわけだね。
特殊相対論を補うための一般相対論でしかないのが現実だと思う。
ご冗談でしょう?名無しさん [sage] 2018/03/02(金) 16:00:56.90ID:???
が宇宙船の大きさを問題にしているとは思えんのだが。

加速していくロケットの長さはどうなる?というような問題だったら別だが。
それもロケットの先端と後端を代表する2質点の運動と考えれば
同じように特殊相対論で考察できなくはないのだが
ご冗談でしょう?名無しさん [sage] 2018/03/02(金) 16:05:41.10ID:???

加速度系のその扱い自体がなぜ正当化されるのかは特殊相対論の枠組みのなかで説明されないけど
それでも頑なに双子のパラドクスの解決に一般相対論は不要だと言う理由はなに?
ご冗談でしょう?名無しさん [sage] 2018/03/02(金) 16:07:24.62ID:???
それ言いだしたら一般相対論的扱いはどう正当化されるのかということになって切りがないでしょ
ご冗談でしょう?名無しさん [sage] 2018/03/02(金) 16:13:26.36ID:???
例えば速度が遅いときのガリレイ変換もローレンツ変換の|v/c|<<1の近似によって正当化されるのだから
速度が遅いときの速度化法則には特殊相対論が必要だ、と主張するのなら首尾一貫はしているね。
賛同はしないけど。

この点に関し、これ以上深入りして議論する気はない
ご冗談でしょう?名無しさん [sage] 2018/03/02(金) 16:15:30.43ID:???
>速度が遅いときの速度化法則には特殊相対論が必要だ
ごめん、速度加法則ね
ご冗談でしょう?名無しさん [sage] 2018/03/02(金) 16:33:42.16ID:???

いや一般相対論の枠組みのなかで「慣性系の移り変わり」と考えて一致することが保証されるから
話が別
ご冗談でしょう?名無しさん [sage] 2018/03/02(金) 16:34:32.05ID:???
これも散々議論されているテーマなのだが、
近似的に重力源(エネルギー源)を無視できる時空間では、
一般相対論は不要。
一様重力場の時空間(リンドラー時空)などは、「適当な」座標変換で、
大域的にミンコフスキー時空にできるから、一般相対論は不要。

しかし、「適当な」は要注意で、なんでもありではない。
逐次ローレンツ変換の範囲で「適当な」ってこと。
重力源が無視できない場合は、大域的にミンコフスキー時空にできる様な、
「適当な」座標変換が存在しない。
これは、一般相対論の範疇で、エネルギー・運動量テンソルの密度分布を、
計算に入れる必要が生じるケース。
ご冗談でしょう?名無しさん [sage] 2018/03/02(金) 17:15:28.73ID:???

だから、加速度を伴った系においてはそんな単純な計算はできませんて
面倒だから私は正しい計算式を示せませんけどね
ご冗談でしょう?名無しさん [sage] 2018/03/02(金) 17:31:43.16ID:???

一般相対論で保証されているのではなく、一般相対論そのものが
そのように構築されている(局所慣性系の解析接続)から、
一般相対論で保証されているというのは結論先取の誤謬だと思う
ご冗談でしょう?名無しさん [sage] 2018/03/02(金) 17:34:34.26ID:???

いずれにせよ特殊相対論の枠組みで閉じてないのは変わらないと思うけど?
ご冗談でしょう?名無しさん [sage] 2018/03/02(金) 21:31:53.79ID:???
横レスするが、
お二人の(三人以上かも知れないが)主張はそれぞれに一理ある。

結論から言えば、特殊相対性理論でも加速系を扱える。ただし、その正当性が保証されるのは慣性系の場合だ。
加速系の場合だと、特殊相対性理論だけでは、その正当性は保証されない。
なぜなら、特殊相対性理論が光速不変原理と特殊相対性原理から成り立っているから。
特殊相対性原理は、全ての慣性系で物理法則が共変というもの。つまり、加速系ではその限りではない。

しかしながら、素朴に考えれば、加速系も瞬間に着目すれば慣性系とみなせそうに思える。
瞬間毎に速度が異なる慣性系へのローレンツ変換の連続適用で加速系を扱えそうな気がする。
そして、これは正しい。実際に加速系は瞬間毎に速度が異なる慣性系として扱える。

以上をまとめると、こうだ。
事実として、特殊相対性理論でも加速系を扱うことができる。だが、特殊相対性理論自体は、その正当性を保証しない。
ご冗談でしょう?名無しさん [sage] 2018/03/02(金) 22:34:24.81ID:???

散々議論したというのは、単発の質問では許されない。
今年の学生に、去年散々講義したから今年は説明しないという説明は成り立たない。

それはそれとして、光速の86.6%で円運動をするという時点で、相対論は巨大な
質量やエネルギーを必要としているだよ。アインシュタインは等価原理で重力と加速度は
区別できないと言っている。アインシュタインは重力の本質については語っていない。分から
ないが加速度と同じとだけ言っている。だから、距離に依存して経過時間が変化しても
おかしくはない。単純な回転運動を説明しようとしたら必ずおかしなことになる。アインシュ
タインが1年後に早々と一般相対論を発表する理由はそこにあるね。
ご冗談でしょう?名無しさん [sage] 2018/03/02(金) 22:46:08.31ID:???

相対論の最大の仮説は
「パラレルワールドを認めない」
ということ。地球にずーっといても、亜光速でどこかの星を回ってきても、2つの結果は同じになる
という仮説。地球にいて2つの時計を比べるのと、亜光速でどこかの星を回って2つの時計を
比べても同じにならなければならないのが前提、それが崩れるのなら別の宇宙に進む科学理論として
存在して良いわけなのね。

の                行き          ターン         帰り
地球の慣性系の観測者の結果  特殊相対論OK  局所慣性系としてOK 特殊相対論OK
宇宙船の観測者の結果      特殊相対論OK    ???       特殊相対論OK
で、パラレルワールドを認めないとすれば、???の部分はトータルの経過時間が等しいことから
説明ができるというのが相対論的な結論だと思うよ。
ご冗談でしょう?名無しさん [sage] 2018/03/03(土) 00:58:06.67ID:???
アホンダラの「ひゃま」が居るな。
ご冗談でしょう?名無しさん [sage] 2018/03/03(土) 01:08:13.79ID:???
座間市9人解体事件って一気に廃れたな

1: 風吹けば名無し
2018/01/11(木) 03:49:54.29 ID:qg7k9ECCM.net [1/1] AAS
いつものことやけど

5: 風吹けば名無し
2018/01/11(木) 03:50:52.64 ID:HZW74yrup.net [1/1] AAS
ああいう異常な事件って頑なに続報出さないよな

7: 風吹けば名無し[sage]
2018/01/11(木) 03:51:23.08 ID:bZDCv4Z10.net [1/3] AAS

実際語るようなこともなくね?

10: 風吹けば名無し
2018/01/11(木) 03:51:57.29 ID:J5QHr02h0.net [1/1] AAS
狂気度だと北九州一家殺人やJKコンクリ事件には及ばない

11: 風吹けば名無し[sage]
2018/01/11(木) 03:52:56.91 ID:bZDCv4Z10.net [2/3] AAS
週一で解体はすげーなーとは思うが
それだけだわな

13: 風吹けば名無し
2018/01/11(木) 03:53:24.29 ID:qBXb33QOa.net [1/1] AAS
事件自体に面白みがなかった
411 [] 2018/03/03(土) 01:18:02.67:zOWKr7mc

のみなさん、すっきりしました。ありがとうございます。
ご冗談でしょう?名無しさん [sage] 2018/03/03(土) 11:37:07.32ID:???

オレ様相対論乙

相対性理論では
座標系が異なる観測者ごとに空間距離も時間経過も異なり、因果範囲も異なるが「相間」はそれを認めない。
相対性理論は座標系の「パラレルワールド」そのもの、数学の群で定義すれば一つの変換群。
量子力学になれば、それより無限に大きい観測世界の「パラレルワールド」と解釈できる。
ご冗談でしょう?名無しさん [sage] 2018/03/03(土) 12:58:53.20ID:???

お前は文盲。
ご冗談でしょう?名無しさん [sage] 2018/03/03(土) 13:27:42.73ID:???

>正当性を保証しない。
これがなきゃ良かったんだがな
ご冗談でしょう?名無しさん [sage] 2018/03/03(土) 16:00:42.76ID:???
この流れだから聞くけど
ニュートリノが質量あるけど光速で飛んでるっぽいのとは逆に
質量ないけど光速未満で飛んでるみたいな素粒子や物質って想定できるの?
ご冗談でしょう?名無しさん [sage] 2018/03/03(土) 16:05:22.90ID:???

> >正当性を保証しない。これがなきゃ良かったんだがな
アインシュタインの特殊相対性理論の論文でも「折れ線近似が連続曲線でも成り立つと仮定すれば」になってる。
特殊相対性理論はあくまでも慣性座標系ありきの理論、質量天体による回転運動等などの
重力の効果に対して何もいえないから不完全な力学理論。
ご冗談でしょう?名無しさん [sage] 2018/03/03(土) 16:11:44.59ID:???

物質中では光は光速未満で運動する、超伝導体中では「質量効果」
ご冗談でしょう?名無しさん [sage] 2018/03/03(土) 18:11:41.70ID:???
ディラック電子とか言うのはどうなの
ご冗談でしょう?名無しさん [sage] 2018/03/03(土) 18:19:27.06ID:???
マスレスでも別に光速で運動するわけではない
ご冗談でしょう?名無しさん [sage] 2018/03/03(土) 20:07:45.21ID:???
オフシェルなら制限なし
ご冗談でしょう?名無しさん [sage] 2018/03/03(土) 22:03:55.66ID:???
ぷっちーポポ
ご冗談でしょう?名無しさん [sage] 2018/03/03(土) 22:42:33.12ID:???
電子electronと陽電子positronはe-とe+で
陽子protonと反陽子antiprotonはpとp-bar

なんか対称性が悪くない?
p+に対して陰陽子negaton:p-とかにしなかった理由って何?
統一マニアの物理学者がやることにしては行き当たりばったり感が強い
ご冗談でしょう?名無しさん [] 2018/03/03(土) 22:58:37.67:GX4Wv2DC
質問しまーす。

電子のドブロイ波の波長はλ=h/mvなので、v=1mm/sなら
70cm程度ということで良いんですか? ゆっくり動かした方が
波としての性質が出るんですよね?
ご冗談でしょう?名無しさん [sage] 2018/03/03(土) 22:59:44.81ID:???

波長はそうだけど波動性が現れるかは別
ご冗談でしょう?名無しさん [] 2018/03/03(土) 23:01:13.34:GX4Wv2DC

その意味は?? 波長が短いほど粒子に近いのでは?
ご冗談でしょう?名無しさん [sage] 2018/03/03(土) 23:02:38.50ID:???

静止した物体の物質波の波長は無限大になるが
実際はデコヒーレンスが起こるので波動性は現れない
ご冗談でしょう?名無しさん [] 2018/03/03(土) 23:04:27.43:GX4Wv2DC

なるほど、バラバラだと波としての性質が出ないと言うことですか。
分かりました。
ご冗談でしょう?名無しさん [] 2018/03/03(土) 23:08:19.97:GX4Wv2DC

もし、電子1個を取り出せて真空中で1mm/s程度にできるとすれば、
普通の大きさの波の干渉装置で干渉縞が見られるということですか。
ご冗談でしょう?名無しさん [sage] 2018/03/03(土) 23:12:42.27ID:???

外乱が無いなど理想的な状況なら多分見られる
ご冗談でしょう?名無しさん [] 2018/03/03(土) 23:15:47.20:GX4Wv2DC

ありがとうございました。
ご冗談でしょう?名無しさん [] 2018/03/04(日) 03:56:56.39:74jrracU
質問しまーす。

1. 以下の図にある摩擦のない壁に立てかけた棒のラグランジアンは
L=(1/2)m((L/2)θ.)^2-mg(L/2)cosθ
で良いでしょうか?
2. ラグランジュ方程式に入れて解くと
θ..=2gsinθ、近似を使ってθ..=2gθ
一般的な微分方程式の解法で
θ=C1 exp(√(2g) t)+C2 exp(-√(2g) t)
となるのは分かるのですが、C2=0とする理由が分かりません、
お願いします。
ご冗談でしょう?名無しさん [] 2018/03/04(日) 04:10:35.36:74jrracU
の続き
3. 実際には棒が右の方に行って壁から離れることは
ないのでしょうか?
お願いします。
ご冗談でしょう?名無しさん [sage] 2018/03/04(日) 06:48:05.74ID:???
数学解の珍現象が起きない理由を考えるのが物理
ご冗談でしょう?名無しさん [sage] 2018/03/04(日) 07:02:00.36ID:???
画像が見れないから、エスパー解答するんだけど…
解析力学の教科書にもあると思うけど、拘束がおかしなことにならないか(抗力が負になったりとか)は
きちんと確かめないとだめだよ。
抗力をあらわに取り込んだ運動方程式を書き下してチェックね。
ラグランジュの未定乗数法のあたりに詳しく書いてあるかと。
ご冗談でしょう?名無しさん [] 2018/03/04(日) 10:24:45.10:rZWOK2B5
【中尾翔太(21)が癌】「実は東京が危ないということは報道できない」と全面カット【報道ステーション】
ttp://rosie.5ch.net/test/read.cgi/liveplus/1520121841/l50
468 [] 2018/03/04(日) 11:25:35.87:f2fxza0Z

失礼しました。
ttp://iup.2ch-library.com/i/i1893362-1520130245.jpg
です、お願いします。
ご冗談でしょう?名無しさん [468] 2018/03/04(日) 11:32:47.80ID:???

C1≦0は起こりそうにないですが、C1>0, C2=一定はありそうに
思いますが?
ご冗談でしょう?名無しさん [sage] 2018/03/04(日) 11:45:25.61ID:???
t=0でθ=θ_0、θ.=0で積分定数は求まる気がする
468 [] 2018/03/04(日) 11:50:06.83:f2fxza0Z

例えば、重心の座標(x, y) x横右正、y上下上正
としたとき、右の壁からの垂直抗力Fxは
Fx=mx..で、x=(L/2)sinθ、近似の段階でx=(L/2)θ
Fx=m(L/2)θ..

θ=C1 exp(√(2g) t)+C2 exp(-√(2g) t)なら
θ..=2g( C1 exp(√(2g) t)+C2 exp(-√(2g) t) )
∀t, Fx=mgL( C1 exp(√(2g) t)+C2 exp(-√(2g) t) )>0
ということでしょうか?

で、にもどるのですが、棒が勢い余って壁から離れることは
ないのでしょうか?
468 [] 2018/03/04(日) 11:56:20.97:f2fxza0Z

なるほど、
θ(t)として、
θ.(0)=√(2g)(C1-C2)=0
であればC1=C2じゃないでしょうか?
ご冗談でしょう?名無しさん [sage] 2018/03/04(日) 11:59:34.52ID:???

その初期条件ならそうだと思うよ
ご冗談でしょう?名無しさん [468] 2018/03/04(日) 12:02:29.41ID:???

ttp://www.research.kobe-u.ac.jp/csi-viz/members/kageyama/lectures/H26_FY2014_latter/Analytical_Mechanics/note_141023b.pdf
を参考に勉強していたのですが、C2=0と限らないということですね。
ご冗談でしょう?名無しさん [sage] 2018/03/04(日) 12:09:15.73ID:???

その初期条件を課すほうが自然。C2=0とするのは不自然。
ご冗談でしょう?名無しさん [sage] 2018/03/04(日) 12:12:21.16ID:???

棒が勢い余って壁から離れることはないのでしょうか?
もしそうなら斜面を滑る玉も飛び跳ねるだろうよ
数式いじくってるだけで物理的条件も考える知能も無いらしい
そもそも壁面に沿って棒が運動するのが前提条件
468 [] 2018/03/04(日) 12:13:59.51:f2fxza0Z

もう一度整理して
θ(0)=C1+C2=θ0
θ.(0)=C1-C2=0
なら、C1=C2=(1/2)θ0>0
のFx>0ですね。分かったように思います。
ありがとうございました。
ご冗談でしょう?名無しさん [sage] 2018/03/04(日) 13:09:45.30ID:???
バウンドして宙に浮くだろうな
ご冗談でしょう?名無しさん [sage] 2018/03/04(日) 13:47:06.28ID:???

左の壁は棒に対して仕事をしていないからLとは無関係
左右にズレることを気にしなければ、倒れることに関して左の壁の有無は関係ない
壁から離れても倒れ方は関係ない
ご冗談でしょう?名無しさん [] 2018/03/04(日) 15:18:54.83:f2fxza0Z
質問しまーす!

H2(g)+1/2O2(g) → H2O(L)
燃焼によって得られる熱量は標準生成エンタルピーΔH=-285.83kJ/mol
標準ギプス自由エネルギー変化はΔG=ΔH-TΔS=-237.13kJ/mol
という意味が分かりにくいのですが、シリンダーの中で断熱でガスを燃やし、
標準状態になった時点に得られる仕事量がギブス自由エネルギーで良いでしょうか?
お願いします。

ttp://iup.2ch-library.com/i/i1893400-1520144207.jpg
ご冗談でしょう?名無しさん [sage] 2018/03/04(日) 15:59:56.05ID:???

そもそも剛体の運動考えるなら回転運動のエネルギーも考えなくちゃだめっすよ。
絵を見る限り棒は一様っぽいし、慣性モーメントくらい簡単に求められるでしょ?

まあ、 のpdf では 「棒の中心に固着した質量 m の質点」ってあるから
慣性モーメント=0 ですけどね。
ご冗談でしょう?名無しさん [sage] 2018/03/04(日) 16:30:29.94ID:???
無間地獄で物理法則は通用しますか?
ご冗談でしょう?名無しさん [sage] 2018/03/04(日) 21:56:31.74ID:???

まず日本語の勉強をして下さいね。
ご冗談でしょう?名無しさん [] 2018/03/04(日) 22:06:08.97:KKjTIYu+
後出しジャンケン!文系のデーモン小暮閣下!俺の誕生日を名乗って

世を惑わすな 忌まわしい 世を惑わす占星術師!

万有引力の法則の球体の相似条件と逆二乗の公式の関係は俺が発見した

デカルトの理性のヒラメキなんてシェイクスピアのハムレットにセリフがあるわ!
ご冗談でしょう?名無しさん [] 2018/03/04(日) 22:09:08.62:ELRAgQpf

回答願いまーす
ご冗談でしょう?名無しさん [] 2018/03/04(日) 22:10:13.78:KKjTIYu+
後出しジャンケン!文系のパクリのデーモン小暮閣下!訴えるぞ!俺の誕生日を名乗って

世を惑わすな 忌まわしい 世を惑わす占星術師!殺されたくなかったら俺の誕生日を名乗るな!

万有引力の法則の球体の相似条件と逆二乗の公式の関係は俺が発見した

デカルトの理性のヒラメキなんてシェイクスピアのハムレットにセリフがあるわ!
ご冗談でしょう?名無しさん [] 2018/03/05(月) 00:21:53.45:TuxWqzzg
殺されたくなかったら俺の誕生日を名乗るな!
ご冗談でしょう?名無しさん [] 2018/03/05(月) 00:54:31.31:RQdMBvP9
ヤンキーゴメンゴメンスイッチ押死テ昔カラ戦争シテルンデシタッケ忘レテマ死多世モヤシテ殺ッテマス100麻薬王薬害盗難ケタチガイ違反重々振戦
ttps://anago.5ch.net/test/read.cgi/welfare/1442312377/
ご冗談でしょう?名無しさん [sage] 2018/03/05(月) 02:44:36.30ID:???

1/12だけ足せば良いだけだが、
神戸大学の資料は焦ったのか、微分方程式の解の基底の一つを安易に捨ててしまうと
いう痛いミスだな。
ご冗談でしょう?名無しさん [sage] 2018/03/05(月) 05:17:43.12ID:???
ぷち ぷらーい
ご冗談でしょう?名無しさん [sage] 2018/03/05(月) 10:10:20.47ID:???

科学の進歩は基本的に行き当たりばったりなんだよ。
将来どのように統一されていくかなんてその時点で知りようもないことだからしゃーない。

電気的に中性のものに対しても反粒子が存在しうるとわかった時点で
posiとnegaで粒子反粒子を区別することはできなくなった。
以後はanti-をつけることで統一されていく。
当然、positronもantielectronにすべきという声はあっただろうが、
既にpositronで普及しまくっていてantielectronは広がらなかった。それだけ

電子の電荷を+に定義しときゃよかったのにとか、
sクォークのストレンジネスが-1になっちゃったとか、
後から思うとまずい定義の数々はあるけどいまさら直すと大混乱。
で、現在に至る
ご冗談でしょう?名無しさん [] 2018/03/05(月) 10:15:12.41:RdjUPWsb
フックの法則が成り立つか調べたいです
そこでレポートは2通りのやり方をして、その誤差について調べることにしました。
輪ゴムを引っ張る力と長さの関係について調べたくて、2通りのやり方を試し、その誤差の原因について調べたいです。
1つ目は、輪ゴムを吊るし、それに重りを吊るして伸びを測るという方法にしようと思っているんですが、他の方法が思いつきません...
何か面白いやり方があれば教えて頂きたいです!
ご冗談でしょう?名無しさん [sage] 2018/03/05(月) 10:21:22.76ID:???
物理
ご冗談でしょう?名無しさん [sage] 2018/03/05(月) 10:32:08.35ID:???
弾性体、剛体の力学
ご冗談でしょう?名無しさん [] 2018/03/05(月) 10:41:28.57:lfWOvw8Y

ポジトロンの発見が今から思えば早過ぎたってイメージか
納得した
ありがとうございます
ご冗談でしょう?名無しさん [sage] 2018/03/05(月) 11:32:29.79ID:???

・ 色の違う輪ゴムを使う たぶん可塑剤が違う
・ 重さを変えて長さを測る と 長さを決めて重さを量る
ぐらいしか今思いつきません orz
ご冗談でしょう?名無しさん [sage] 2018/03/05(月) 11:48:09.11ID:???
Please, GGRKS, Boy.
ご冗談でしょう?名無しさん [sage] 2018/03/05(月) 11:51:10.40ID:???
こっちは結論がわかってて言うんだけど、温かい場所と冷たい場所で実験してみるとか・・・
ご冗談でしょう?名無しさん [sage] 2018/03/05(月) 11:56:33.70ID:???
のはいったい何が目的でその実験なの?
中学校の先生の理科実験とか、科学部とかの発表用実験、小学生の春休みの自由研究
あたりから
こんな場所できいてるけど、大学生の実習授業、卒論・修論の題材さがしとか色々あるわけで…。

ただ、ゴムは自然長より短くなった時がつまらないと思います。
ご冗談でしょう?名無しさん [] 2018/03/05(月) 12:44:09.25:RdjUPWsb
物理オリです
ご冗談でしょう?名無しさん [sage] 2018/03/05(月) 12:57:40.40ID:???
ゴムの温度依存性とか。
ご冗談でしょう?名無しさん [sage] 2018/03/05(月) 13:57:10.82ID:???

おまえには無理
ご冗談でしょう?名無しさん [sage] 2018/03/05(月) 14:08:28.72ID:???

形や自重の影響がある実験てアホやん
もっと他の影響が少ない実験方法を考えろよ
ご冗談でしょう?名無しさん [sage] 2018/03/05(月) 15:06:06.52ID:???
水平にして、バネ秤で引っ張る。
ご冗談でしょう?名無しさん [sage] 2018/03/05(月) 17:31:40.56ID:???
ムーニーちゃんしんぷだ〜い
ご冗談でしょう?名無しさん [sage] 2018/03/05(月) 19:00:46.63ID:???
ゴムってフックの法則成り立つの?
ご冗談でしょう?名無しさん [sage] 2018/03/05(月) 19:03:30.33ID:???
ひずみが小さい範囲ならほとんどの物質で成り立つ
ご冗談でしょう?名無しさん [] 2018/03/05(月) 19:12:43.47:3jQg8yuo
お願いします。
なぜXの絶対値を二階微分したら 2δ(x)になるのでしょうか。

一回微分なら  x>0で1 x<0で-1 x=0では?
ご冗談でしょう?名無しさん [sage] 2018/03/05(月) 19:13:02.66ID:???

成り立たない物質って例えば何?
ご冗談でしょう?名無しさん [] 2018/03/05(月) 19:42:17.76:KZ6V4vzp

f(x):=|x|は普通の意味では原点で微分不能だから、厳密なことを言えば
シュヴァルツの超関数の話になる
f'(x)={x>0で1 x<0で-1 x=0で定義されない
という、ヘヴィサイドの階段関数みたいなのになる。これを『微分』したい

(大雑把な説明)
もしφ'が普通に微分できる関数だったら、微積分学の基本定理から
∫_[a,b]φ''(x)dx = φ'(b)-φ'(a)
これを逆手に取り、任意の実数a,bに対し
∫_[a,b]ψ(x)dx = φ'(b)-φ'(a)
を成り立たせるような関数ψ(x)を、φ'(x)の微分とみなそう、と考える

∫_[a,b]2δ(x)dx=0(a,bが同符号の時) =2(a,bが異符号の時)
だけど、 よく見るとこの右辺は上で出てきたf'(b)-f'(a)と完全に一致するので
f''(x)=2δ(x)
ご冗談でしょう?名無しさん [sage] 2018/03/05(月) 20:08:35.37ID:???
流体
ご冗談でしょう?名無しさん [sage] 2018/03/05(月) 20:15:42.39ID:???

超関数でGGR
ご冗談でしょう?名無しさん [] 2018/03/05(月) 20:27:53.16:Zjn0CHLh
ttps://i.imgur.com/EBBWd81.jpg
hの扱いについて質問です

(1)でベルヌーイの式で
全圧側と静圧側の高さをh(A)とh(B)と置いてそれらが微小な差であるから
h(A)=h(B)より両辺のρghを消去して解答のように立式するそうですが
そもそもこのh(A)やh(B)ってどこの話ですか?
何を基準にh(A)という長さを取ってるんですか?
類似の問題の解答を見ても最初から
「ベルヌーイの定理を用いて〜」といきなりh(A)が出てきて困惑します
この問題はU字管マノメータ?が下についてますが
ピトー管は上向きに管が伸びてる問題もよくありますしベルヌーイを使うときのhの扱いが分かりません……
ピトー管の問題を解くにあたってこのhが何を指してるのか教えてください
というか2つの水面の高低差でしたら0扱いなんですよね?
(2)もそうすると何を言ってるのかさっぱり……
ご冗談でしょう?名無しさん [] 2018/03/05(月) 21:29:06.11:49S5inYu

純粋な数学の問題は数学板へ
便利だからと言って居間にトイレや浴室を持ち込むのはNG

分からない問題はここに書いてね441
ttp://rio2016.5ch.net/test/read.cgi/math/1519050603/l50
ご冗談でしょう?名無しさん [sage] 2018/03/05(月) 21:57:09.52ID:???
ガウスの法則では、電荷Q[C]あたり4πkQ本の電気力線が出るそうですが、1Cとかだと4π本となって整数ではなくなってしまいますね
どういうことなんですか?
ご冗談でしょう?名無しさん [sage] 2018/03/05(月) 22:16:18.79ID:???

物理でも扱うだろw
ご冗談でしょう?名無しさん [sage] 2018/03/05(月) 22:24:20.62ID:???

1時間でπkmを走ったら時速が整数でなくなってしまいますね。
なにか困りますか?
ご冗談でしょう?名無しさん [sage] 2018/03/05(月) 22:43:01.27ID:???

それはわかりますよね
電気力線がπ本あるとはどのようなことですか?
たとえば、絵に描いてみてください
255 [sage] 2018/03/05(月) 23:01:49.11ID:???

時間が整数として理解されていた時代があった
でも今は違う
同様に、電気力線が線として理解されていた時代はもう古い
「電気力線」という呼称はそんな時代の名残
ご冗談でしょう?名無しさん [sage] 2018/03/05(月) 23:07:02.94ID:???

電気力線の正しい定義を教えてください
255 [sage] 2018/03/05(月) 23:17:59.70ID:???

電気力線は、電場という場がベクトルで表されていることを模式的に描いた線です
ご冗談でしょう?名無しさん [sage] 2018/03/05(月) 23:18:28.87ID:???

単位面積あたりの場の強さ
ご冗談でしょう?名無しさん [sage] 2018/03/05(月) 23:18:39.47ID:???

え?電気力線は線ではないんじゃないんですか?


>同様に、電気力線が線として理解されていた時代はもう古い
>「電気力線」という呼称はそんな時代の名残
ご冗談でしょう?名無しさん [sage] 2018/03/05(月) 23:19:01.56ID:???

電気力線とはスカラー場だということですか?
ご冗談でしょう?名無しさん [sage] 2018/03/05(月) 23:19:53.36ID:???

電場はベクトル場です
255 [sage] 2018/03/05(月) 23:20:08.78ID:???

そうです
ガウスの法則を「電気力線」で理解しようとすること自体に無理があります
ご冗談でしょう?名無しさん [sage] 2018/03/05(月) 23:23:53.01ID:???

単位面積当たりの電場の強さはベクトルなんですか?


でも線なんですよね?
電気力線は、電場という場がベクトルで表されていることを模式的に描いた線です
255 [sage] 2018/03/05(月) 23:28:38.90ID:???

電気力線は、電場がベクトル場であることを理解させるために使われています
矢印付きの線で表せば、電場がベクトル場であることをイメージしやすいからです
でも実際は線なんて出てません。あるのは場です。線だとか矢印だとかは人間がそれを理解するために勝手に設定した概念です
ご冗談でしょう?名無しさん [sage] 2018/03/05(月) 23:29:42.04ID:???

なんとなくわかりました
では、電気力線がπ本あるとはどのようなことですか?
255 [sage] 2018/03/05(月) 23:38:56.97ID:???
そうですねー
こう考えてみるのはどうでしょう
電荷Qに対して4πkQ本の電気力線が出ている
線の数は無理数なのでこのままだと数を数えることが出来ません
なので線を一旦まとめて、それを可算無限個に分けます
ご冗談でしょう?名無しさん [sage] 2018/03/05(月) 23:41:03.81ID:???
可算無限個に分けるとはどのようなことですか?
255 [sage] 2018/03/05(月) 23:41:25.61ID:???
この可算無限個に分けられた線を足すと4πkQ本になります
この操作はつまるところ積分です
255 [sage] 2018/03/05(月) 23:44:16.87ID:???
無理数はどんな数で割っても無理数です
でも無理数は実数なので無限個の微小な実数の足し算で表すことができます
ご冗談でしょう?名無しさん [sage] 2018/03/05(月) 23:45:23.42ID:???
実数に無限小という数は存在しません
超実数を考えているということですか?
255 [sage] 2018/03/05(月) 23:50:22.24ID:???
そうですね
厳密には実数じゃないです
作用素として与えるのがいいでしょうか
積分を見たことがあるなら分かると思いますが、dxがその微小な数を現す作用素です
ご冗談でしょう?名無しさん [sage] 2018/03/05(月) 23:54:55.97ID:???
超実数を用いると、微分は無限小の割り算として、積分は本当に無限小の足し算として表現できます
もしわからないのならば、無理に知ったかぶりをしなくてもいいんですよ、念のため

さて、作用素ですか
電気力線1本当たりの量dxを具体的に定義しなければならないのではないですか?超実数を用いるとするならば

素朴な微分積分の定義を用いるとするならば、総和の極限として定義する必要があります
極限を用いるとするならば、この場合、電気力線とは、そこにあるものではなく、どんどん分割されていく過程そのものだと考える必要があるわけです
これでは電気力線の元のイメージとはかけ離れていますね
255 [sage] 2018/03/05(月) 23:58:42.90ID:???
ありがとうございます541さん
実際足りない頭で唸っていました
超実数ってなんすか知りませんよそんなの
ご冗談でしょう?名無しさん [sage] 2018/03/06(火) 00:00:07.18ID:???
超準解析も知らんゴミはROMってろ
ご冗談でしょう?名無しさん [sage] 2018/03/06(火) 00:01:14.56ID:???
つまりですね、電気力線とはあなたのいうように昔の話なんです
数学のセンスのないファラデーが考え出したインチキです
それをさも数学的な根拠があるような教育がなされているのには驚きを隠せませんね
255 [sage] 2018/03/06(火) 00:01:15.26ID:???
それは嫌です
超準解析なんて知らなくてもちょっとした積分くらいなら分かります
ご冗談でしょう?名無しさん [sage] 2018/03/06(火) 00:31:19.24ID:???
このスレって定期的にメンバ入れ替わるんだな
なぜこのネタにまた食いつく?
ご冗談でしょう?名無しさん [sage] 2018/03/06(火) 00:42:58.88ID:???

フックの法則が成り立たない物質って例えば何
ご冗談でしょう?名無しさん [sage] 2018/03/06(火) 00:44:48.12ID:???
定期的にデタラメ書いて暴れる奴が出る
ご冗談でしょう?名無しさん [sage] 2018/03/06(火) 00:52:44.95ID:???
流体力学でも「流線」とか視覚化ツールがあるじゃん。
「電気力線」もそんなもんじゃね?
ご冗談でしょう?名無しさん [] 2018/03/06(火) 01:50:44.72:LRTG4/Hd
ありがとうございます。
まだ検討してないし吟味してないんですが
わざわざご丁寧に回答していただいて心より感謝いたします。
ご冗談でしょう?名無しさん [sage] 2018/03/06(火) 07:15:47.17ID:???
まーた劣等感婆が現れたのか
ご冗談でしょう?名無しさん [sage] 2018/03/06(火) 08:58:22.85ID:???
喜んで食いつく馬鹿が多いことw
ご冗談でしょう?名無しさん [sage] 2018/03/06(火) 14:51:31.59ID:???
ぷっちー ポポ
ご冗談でしょう?名無しさん [sage] 2018/03/06(火) 15:35:54.58ID:???
下線部についてなのですが、回転角がこのようになるイメージがわきません。理論的に解説お願いいたします。
ttps://i.imgur.com/PSyzfLz.jpg
ご冗談でしょう?名無しさん [sage] 2018/03/06(火) 15:45:33.67ID:???

回転角が0の場合、A'はGの真下に来ているでしょ。
そこからの回転角だと思えば自明だと思うが
ご冗談でしょう?名無しさん [sage] 2018/03/06(火) 16:39:50.02ID:???
量子力学の二重スリット実験について質問です。
有名なDr Quantumの動画は、量子に全く影響を及ぼさずに観測することは出来ないという点で、
前提から間違いがあることはわかりました。
ですが、こちらの動画の実験で、人間が念じるだけで結果が変わるという理屈がわかりません。
ttps://www.youtube.com/watch?v=yLufAR6k8tw
わかる方居ましたら教えて頂けませんか。
ご冗談でしょう?名無しさん [sage] 2018/03/06(火) 17:03:20.33ID:???

それでしたら滑らない条件の式もφではなくφ-θじゃないんですか?
ご冗談でしょう?名無しさん [sage] 2018/03/06(火) 17:15:59.87ID:???

そんなもんわかるか
ご冗談でしょう?名無しさん [sage] 2018/03/06(火) 17:17:01.97ID:???

滑らない条件は弧ABと弧A'Bの長さが等しいこと。だからφで正しい。

φ-θだと、Gの真下の位置からA'に至る弧の長さが弧ABと等しいことになってしまう。
ご冗談でしょう?名無しさん [sage] 2018/03/06(火) 17:21:11.19ID:???

つまり、物体がφ回転したから弧ABだけ進んだってことですよね?それなら回転角はφではないんですか?
ご冗談でしょう?名無しさん [sage] 2018/03/06(火) 17:34:00.37ID:???

横レスだが、月の公転周期は27.3日なのに朔望周期が29.5日である理由を考えてみよう。
ご冗談でしょう?名無しさん [sage] 2018/03/06(火) 18:09:37.25ID:???

成り立たない物質って例えば何
ご冗談でしょう?名無しさん [sage] 2018/03/06(火) 18:17:24.78ID:???

ということは人間の意志が物理現象に干渉する可能性はまだあるってことでいいんですかね?
ご冗談でしょう?名無しさん [sage] 2018/03/06(火) 18:20:41.13ID:???
手を動かす意思で手が動くのは当然
ご冗談でしょう?名無しさん [sage] 2018/03/06(火) 18:22:52.10ID:???

すべての物質
ご冗談でしょう?名無しさん [sage] 2018/03/06(火) 20:16:32.63ID:???

続きはオカルト板で
ご冗談でしょう?名無しさん [sage] 2018/03/06(火) 20:17:29.16ID:???

訂正するならそう明言して謝罪しろ
ご冗談でしょう?名無しさん [sage] 2018/03/06(火) 20:26:22.89ID:???

ごめんね
なんでもするから許して
ご冗談でしょう?名無しさん [sage] 2018/03/06(火) 21:20:25.25ID:???
ttps://i.imgur.com/HuRo1ef.jpg
ttps://i.imgur.com/QtcXzfR.jpg
線で引いたところは面積の微小部分で積分するためのものなんですか?
それと何故円形なのか教えて欲しいです。
ご冗談でしょう?名無しさん [sage] 2018/03/06(火) 22:02:25.28ID:???
導体表面を微小円環に分割して積分しているからそうなる。
平面極座標系(r, θ)の微小面積dSは、
dS = rdθdr
で、θを一周積分すると2πになるから
dS = 2πrdr
になるのは二番目の画像の通り。
ご冗談でしょう?名無しさん [sage] 2018/03/06(火) 23:17:13.28ID:???
運動エネルギーについてです。(2)bなのですが、回転運動の運動エネルギーでIcを用いていますがこれは重心(問題文では質量中心)周りの慣性モーメントであって、本来は回転軸周りの慣性モーメントで運動エネルギーを求めるのではないですか? 誤植なのでしょうか...
ttps://i.imgur.com/hDJyB8n.jpg
ttps://i.imgur.com/2qMnRWw.jpg
ご冗談でしょう?名無しさん [sage] 2018/03/07(水) 00:13:17.01ID:???

訂正なんぞするわけねーよ  はオレじゃねーし
ご冗談でしょう?名無しさん [sage] 2018/03/07(水) 00:14:11.22ID:???

じゃあフックの法則が成り立たない物質って例えば何
ご冗談でしょう?名無しさん [sage] 2018/03/07(水) 02:43:29.34ID:???

なんとなく分かりました。ありがとうございます。
ご冗談でしょう?名無しさん [sage] 2018/03/07(水) 11:25:41.07ID:???
ムーニーちゃんしんぷなんだい(><;)
ご冗談でしょう?名無しさん [sage] 2018/03/07(水) 12:54:02.55ID:???

平行軸の定理
ご冗談でしょう?名無しさん [sage] 2018/03/07(水) 13:30:14.34ID:???

オマエは日本語が読めんのか
何でもいいから目の前にあるものを手に取ってみろ
ご冗談でしょう?名無しさん [sage] 2018/03/07(水) 13:55:09.42ID:???

平行軸の定理使ってるんですか?
ご冗談でしょう?名無しさん [sage] 2018/03/07(水) 14:20:53.05ID:???

分かりました、そもそも並進と回転の合わさった運動では重心周りの慣性モーメント使うんでした。ありがとうございました。
ご冗談でしょう?名無しさん [sage] 2018/03/07(水) 14:49:31.56ID:???

成り立たない物質の例を聞いてるのに「すべての物質」なんて言ってるのは
日本語読めないか物理を微塵も理解してないかのどちらかでしかないが
ご冗談でしょう?名無しさん [sage] 2018/03/07(水) 15:03:31.89ID:???
ポテンシャルエネルギーがなぜこのような式になるかわかりません。弾性エネルギーのほうには自然長からの伸びも必要で、位置エネルギーはθ=0基準ならmg(l-(l+x)cosθ)だと思います。
ttps://i.imgur.com/N0lI2SQ.jpg
ttps://i.imgur.com/Pb3EFfL.jpg
ご冗談でしょう?名無しさん [sage] 2018/03/07(水) 15:07:20.01ID:???

回答のおかげで理解できました。ありがとうございました。
ご冗談でしょう?名無しさん [] 2018/03/07(水) 17:14:05.89:5ezsxhYT
>位置エネルギーはθ=0基準ならmg(l-(l+x)cosθ)だと思います。

そうだな。
それで正解だ。

くっくっく
ご冗談でしょう?名無しさん [] 2018/03/07(水) 18:16:33.41:XgRj8xXB
借金して、銀行などにお金返せなくなったら、金融証書で銀行をつくって、銀行の経済をつくって、相殺する方法がある。上記 登記
ご冗談でしょう?名無しさん [] 2018/03/07(水) 22:17:21.01:XgRj8xXB
創世の石と貴世石を、創世の石で、反重力板に繋ぐと、反重力板になる。 上記 登記
ご冗談でしょう?名無しさん [] 2018/03/07(水) 22:18:17.12:XgRj8xXB
創世の石と貴世石と貴世石を、創世の石で、反重力板に繋ぐと、強力な反重力板になる。 上記 登記
ご冗談でしょう?名無しさん [] 2018/03/07(水) 22:18:38.14:XgRj8xXB
創世の石と貴世石つ2つ半を、創世の石で、反重力板に繋ぐと、強力な反重力板になる。 上記 登記
ご冗談でしょう?名無しさん [sage] 2018/03/08(木) 10:09:28.18ID:???
ここまであぼーん
ご冗談でしょう?名無しさん [] 2018/03/08(木) 10:31:37.01:LEfTz7xZ
ttp://goodlg.seesaa.net/article/451406304.html
ご冗談でしょう?名無しさん [] 2018/03/08(木) 15:45:03.30:E/qAg1Ah
あらためて、大変わかりやすい説明をありがとうございました。
お手間をとらせました。
心から感謝いたします。
ご冗談でしょう?名無しさん [sage] 2018/03/08(木) 17:18:44.49ID:???

分かった分かった、オマエ負け犬だろ
無意味な粘着など相手にせんぞ
ご冗談でしょう?名無しさん [age] 2018/03/08(木) 17:45:17.34ID:???
重力は空間のゆがみらしいけど、それなら木からリンゴが落ちる時に少しカーブして
落ちるのが普通だと思うけど直線で落ちるのはなぜ?
ご冗談でしょう?名無しさん [sage] 2018/03/08(木) 17:57:32.52ID:???
リンゴの落下は時間の歪み効果
ご冗談でしょう?名無しさん [sage] 2018/03/08(木) 18:00:21.34ID:???
リンゴの落下は熟したせいだろ
ご冗談でしょう?名無しさん [sage] 2018/03/08(木) 18:17:40.90ID:???


両方バカです
ご冗談でしょう?名無しさん [sage] 2018/03/08(木) 18:39:21.30ID:???
ムニちゃーんダンス
ご冗談でしょう?名無しさん [sage] 2018/03/08(木) 18:46:47.23ID:???

たったひとつでもフックの法則が成り立たない物質の例を挙げるかを訂正するか
どちらかをすればいいのに何故できないの?
ご冗談でしょう?名無しさん [sage] 2018/03/08(木) 19:03:04.85ID:???
書いたの俺だけど、久しぶりに来たら何かモメてる...

平衡点のまわりでポテンシャルをテイラー展開すると

U=U0+ax^2+bx^3+... (平衡点なので一次の項は0)

なので、a≠0なら|x|<<1の近似でフックの法則が成り立つ。
だけどどんな物質に対しても二次の係数a≠0と言えるのか自信がないので
では「ほとんど」と書いた。
今も自信がないので「ほとんど」を撤回もできかねる。

a=0となる物質はないとあなたが証明してくれるなら
喜んでは「全ての物質で成り立つ」と訂正させていただく
ご冗談でしょう?名無しさん [sage] 2018/03/08(木) 21:18:38.93ID:???

>U=U0+ax^2+bx^3+...
テイラー展開??? ただの多項式
ご冗談でしょう?名無しさん [sage] 2018/03/08(木) 22:57:18.85ID:???
↑これが物理板の実力です
専門板なのに異常にレベルが低い
せいぜい数学の少しできる高校生レベル
ご冗談でしょう?名無しさん [sage] 2018/03/08(木) 23:25:37.93ID:???

>重力は空間のゆがみらしいけど、
誤りだ。重力は『時空』のゆがみ。

>それなら木からリンゴが落ちる時に少しカーブして
>落ちるのが普通だと思うけど直線で落ちるのはなぜ?
時空の中では、リンゴはカーブしている。
ご冗談でしょう?名無しさん [sage] 2018/03/08(木) 23:41:22.76ID:???

ただの多項式を見て「テイラー展開」と思うバカ
ご冗談でしょう?名無しさん [sage] 2018/03/08(木) 23:43:37.92ID:???
どんな物質でも壊れるまで引き延ばせばフックの法則なんて成り立たなくなるから
成り立つ物質なんてないよってのも言えるわな
しょうじきどうでもいいけど
ご冗談でしょう?名無しさん [sage] 2018/03/08(木) 23:48:20.69ID:???

壊れるまで引き延ばして成り立たないと言うバカ
ご冗談でしょう?名無しさん [sage] 2018/03/08(木) 23:52:10.82ID:???

↑これが物理板の実力です
専門板なのに異常にレベルが低い
せいぜい数学の少しできる高校生レベル
ご冗談でしょう?名無しさん [sage] 2018/03/08(木) 23:53:45.70ID:???
そんなこと言うならオームの法則だってそうだしな

小学校の時
「振り子は持ち上げた高さと同じ高さまで上がる」
って答えてその場で実験してその通りにならなかったのも当時は空気抵抗のせいだと思ってたけど
実際は過度に持ち上げると非線形項が効いてくるせいだし
ご冗談でしょう?名無しさん [sage] 2018/03/08(木) 23:54:33.57ID:???

対称性から制限されない限り係数が厳密に零になることはない
数学と物理学は違う
ご冗談でしょう?名無しさん [sage] 2018/03/08(木) 23:56:56.09ID:???
文系です。物理は高校の時にやったきりです。

フォトンというものは質量mがゼロなのに運動量pを持つと聞きました。
運動量は質量と速度vの積、p=mvではなかったのですか?
ここがよくわかりません。

pはmやvとは独立に存在する物理量であって、物質(原子核と電子の集まり)の場合にはmとvに分けられる。というような解釈なのでしょうか?
それだったら非物質のフォトンがm=0でも運動量を持つと納得できるのですが、この解釈は正しいものでしょうか?
ご冗談でしょう?名無しさん [sage] 2018/03/08(木) 23:58:12.67ID:???

その運動量の定義は高校生向けのいい加減なもの
正しくはラグランジアンを一般座標の時間微分で微分したもの
ご冗談でしょう?名無しさん [sage] 2018/03/09(金) 00:00:06.79ID:???
文系に伝える気0だな
ご冗談でしょう?名無しさん [sage] 2018/03/09(金) 00:04:22.59ID:???
いい加減というか、運動量の定義にはいくつかある、といったほうが正しいですよね?
動力学的運動量と正準運動量は異なるわけです
ご冗談でしょう?名無しさん [sage] 2018/03/09(金) 00:06:08.50ID:???
質量0の実在の粒子は運動量を持つ?持たない?
ご冗談でしょう?名無しさん [sage] 2018/03/09(金) 00:06:57.26ID:???

それとの話は別物
ご冗談でしょう?名無しさん [sage] 2018/03/09(金) 00:07:49.79ID:???

どうしてですか?
ご冗談でしょう?名無しさん [sage] 2018/03/09(金) 00:12:46.72ID:???

高校で習う kinetic momentum なんてものは観測可能でも何でもない無意味な量
ご冗談でしょう?名無しさん [sage] 2018/03/09(金) 00:14:04.99ID:???

で、結局どういうことですか?
ご冗談でしょう?名無しさん [sage] 2018/03/09(金) 00:14:24.49ID:???

p = mv なんて式は定義でもなんでもないから忘れろ
ご冗談でしょう?名無しさん [sage] 2018/03/09(金) 00:15:55.50ID:???

kinetic momentumは運動量ですよ?
他の定義もあるということなら、運動量といっても色々な定義がある、ということですよね?
ご冗談でしょう?名無しさん [sage] 2018/03/09(金) 00:17:34.03ID:???

kinetic momentum は「じゃあ mv って何だったの」って問に対して無理やり答えるために付けられた名前であって
物理的に意味のある運動量ではない
運動量の定義はしかない
ご冗談でしょう?名無しさん [sage] 2018/03/09(金) 00:19:09.46ID:???

物理的に意味のある、とはどういうことですか?
また、kinetic momentumのmomentumは運動量ですよ?
あなたがどう屁理屈こねようと、運動量と呼ばれる量です
ご冗談でしょう?名無しさん [sage] 2018/03/09(金) 00:20:22.12ID:???

意味があるとはゲージ不変であること
ご冗談でしょう?名無しさん [sage] 2018/03/09(金) 00:23:07.97ID:???

p=mvはゲージ普遍ではないんですか?
ご冗談でしょう?名無しさん [sage] 2018/03/09(金) 00:24:14.59ID:???

そう言ってる
ご冗談でしょう?名無しさん [sage] 2018/03/09(金) 00:24:35.25ID:???

なんでですか?
ご冗談でしょう?名無しさん [sage] 2018/03/09(金) 00:32:25.88ID:???

どういうレベルでの質問なんだ
ゲージ変換すら知らないのか接続の概念を知らないのか
大体最初の疑問から逸脱しているが真面目に聞くつもりあるのか
ご冗談でしょう?名無しさん [sage] 2018/03/09(金) 00:38:03.08ID:???

p=mv+eAゲージ変換したら違くなっちゃいませんか?
ご冗談でしょう?名無しさん [sage] 2018/03/09(金) 00:41:12.40ID:???

そこから丁寧に説明するほどの気力はないから自分で勉強するか他の暇人を待て
みたいな疑問は運動量というものを前提から誤解しているために生じるもので悩むだけ時間の無駄
最後の段落のように「質量を持つ普通の物質ではたまたま p = mv と書けるだけだ」と言われて納得できるならそれでいい
ご冗談でしょう?名無しさん [sage] 2018/03/09(金) 00:46:15.28ID:???

つまり、あなたはわからないということですか?
ご冗談でしょう?名無しさん [sage] 2018/03/09(金) 00:46:31.68ID:???
わかるなら答えを書くはずですからね、書かないということはわからないということです
ご冗談でしょう?名無しさん [sage] 2018/03/09(金) 00:47:28.05ID:???
劣等感婆出たか?
ご冗談でしょう?名無しさん [sage] 2018/03/09(金) 00:50:59.46ID:???
でたでた
ご冗談でしょう?名無しさん [] 2018/03/09(金) 02:12:02.54:13ftGrrV

>フォトンというものは質量mがゼロなのに運動量pを持つと聞きました。
>運動量は質量と速度vの積、p=mvではなかったのですか?

高校物理(ニュートン力学)では質量がゼロなら運動方程式にならないから
フォトンうんぬんはニュートン力学でない物理理論での話だと気づくべき。
ご冗談でしょう?名無しさん [sage] 2018/03/09(金) 03:03:51.76ID:???
これが劣等感の人なんだね
回答に時間がかかる問題を投稿してリアクションした人に矛盾する質問を重ねてする
相手が匙を投げたら解答出来ないとみなす
ご冗談でしょう?名無しさん [sage] 2018/03/09(金) 03:43:21.51ID:???
やめて!ババアの運動量も質も量もゼロよ!
ご冗談でしょう?名無しさん [sage] 2018/03/09(金) 03:43:55.38ID:???
存在価値はマイナスなのに!!
ご冗談でしょう?名無しさん [sage] 2018/03/09(金) 08:26:29.03ID:???
仕事=0
ご冗談でしょう?名無しさん [sage] 2018/03/09(金) 09:34:05.64ID:???

>運動量は質量と速度vの積、p=mvではなかったのですか?
yes
p=mvではなかった、と量子力学で分かった

運動量の現在の定義は

ニュートン力学時代に知られていた物質ではがp=mvとなる
フォトンは違う
ご冗談でしょう?名無しさん [sage] 2018/03/09(金) 09:42:37.41ID:???
名前は正直どうでもいいんだけど、運動量の一般化としての共役運動量でしょ
ご冗談でしょう?名無しさん [] 2018/03/09(金) 10:08:55.12:XSaoqBOH
相対論だと
p=γmv (γ=√1/(1-v^2/c^2))
なんだけど、光速で運動する粒子に対してはγ=1/0になるので
この式は適用できない。
一方、自由粒子のエネルギはー E=γmc^2 となるので、E/p=c^2/v
この関係を光子にあてはめれば、光子の運動量は E/c になる。
ご冗談でしょう?名無しさん [sage] 2018/03/09(金) 10:11:44.09ID:???

展開係数がUの微分であらわに書かれていないとただの多項式だと思い込むバカなのか?
ご冗談でしょう?名無しさん [sage] 2018/03/09(金) 10:14:39.31ID:???

>自由粒子のエネルギはー E=γmc^2 
それだって光子に対しては∞×0になっちゃうから適用しちゃダメだろ。

光子の運動量がE/cになるのはマクスウェル方程式から直接に言えること。
むしろその事実から、光子の質量が0だとわかる
ご冗談でしょう?名無しさん [sage] 2018/03/09(金) 12:54:49.59ID:???

エネルギー保存に非線形項あったっけ
ご冗談でしょう?名無しさん [sage] 2018/03/09(金) 13:06:24.78ID:???
振り子は糸と固定点との摩擦のせいですよ
ご冗談でしょう?名無しさん [sage] 2018/03/09(金) 13:13:24.37ID:???
等時性かなんかと勘違いしてんだろ
ご冗談でしょう?名無しさん [sage] 2018/03/09(金) 13:21:25.37ID:???
ぷっちー ポポ
ご冗談でしょう?名無しさん [sage] 2018/03/09(金) 13:22:24.97ID:???
ムニちゃーんポッポ
ご冗談でしょう?名無しさん [] 2018/03/09(金) 17:04:21.54:XSaoqBOH

もちろん、p, Eどちらの形式も光子には適用できない。
E/p=c^2/v だけはなぜか使える(笑)とすれば簡単だって話。
いずれにせよ、p=mvはもともと正しくないよ、ってこと。
ご冗談でしょう?名無しさん [sage] 2018/03/09(金) 17:18:29.66ID:???
でもmvはなんだときかれたら、あなたは運動量だと答えるのではないですか?
ご冗談でしょう?名無しさん [sage] 2018/03/09(金) 17:26:49.02ID:???
高校生に毛の生えたレベルの婆の相手をするなよ
ご冗談でしょう?名無しさん [sage] 2018/03/09(金) 17:29:05.35ID:???
物理は実験ありきの学問だから
このスケールだとこの式では誤差が生じるのでこう定義すると上手くいく
みたいな感じです
運動量もあるスケール(ニュートン力学の成り立つ範囲)ではp=mvという定義でオッケーですし、高校物理はニュートン力学です
でもその式では誤差が生じてしまう場合があると分かりました
物体が小さすぎたり(量子力学)、速すぎたり(相対性理論)する場合です
ご冗談でしょう?名無しさん [sage] 2018/03/09(金) 17:39:19.67ID:???
mvには動力学的運動量という名前が付いてますよ?
スケール関係なく
ご冗談でしょう?名無しさん [sage] 2018/03/09(金) 17:51:02.94ID:???
そうなんですか
動力学的運動量は動力学的です
動力学の想定するスケールはご存じですか?
608 [] 2018/03/09(金) 21:20:27.30:nhGh9hs0
多くの回答を下さった皆様、ありがとうございました

p=mvというのはニュートン時代のやり方であり、現在の運動量は全く違った形式で表されるのですね
私の足りない頭では理解できそうもないですが、疑問は解決しました

ありがとうございました
ご冗談でしょう?名無しさん [sage] 2018/03/09(金) 21:23:31.29ID:???

そうではないらしいですよ
頭のいい人がそれを詳しく説明してくれることでしょうね
ご冗談でしょう?名無しさん [sage] 2018/03/09(金) 21:33:07.43ID:???
★ 春は日差しと日照時間が延びるのでセロトニン分泌が増えるために
                              幻聴や妄想が悪化します ★
ご冗談でしょう?名無しさん [sage] 2018/03/09(金) 21:39:25.07ID:???
654が劣等感だ
定義に当てはまる
ご冗談でしょう?名無しさん [] 2018/03/09(金) 22:06:23.11:yd/BAA+R
私は、海辺で遊んでいる少年のようである。

ときおり、普通のものよりもなめらかな小石やかわいい貝殻を見つけて夢中になっている。

真理の大海は、すべてが未発見のまま、目の前に広がっているというのに。
ご冗談でしょう?名無しさん [sage] 2018/03/09(金) 22:23:58.47ID:???

当たり前だけど過度に持ち上げると単振動にならんから振り子の動きがカオス的になるんだよ

保存則だけで言えば正しいけど、振り子の動きという意味では実は単純じゃないんだわ
ご冗談でしょう?名無しさん [sage] 2018/03/09(金) 22:27:43.53ID:???
ここの回答者って、エネルギー保存則もわからないんですね
ご冗談でしょう?名無しさん [sage] 2018/03/09(金) 22:40:33.32ID:???
もしかして単振り子わからない奴がいるのか
ご冗談でしょう?名無しさん [sage] 2018/03/09(金) 22:45:33.81ID:???

こういう人ですね
ご冗談でしょう?名無しさん [sage] 2018/03/09(金) 23:05:49.91ID:???
「わからないんですね」は劣等感だから相手にすんなよ
ご冗談でしょう?名無しさん [sage] 2018/03/09(金) 23:21:14.26ID:???
劣等感婆でも流石に単振り子はわかりそう
ご冗談でしょう?名無しさん [] 2018/03/10(土) 01:16:46.36:ZIDPYY8/

p=mvは v<<c では正しいが、v〜c では正しくない。
ご冗談でしょう?名無しさん [sage] 2018/03/10(土) 01:19:14.53ID:???
でもmvは動力学的運動量と呼ばれるちゃんとした量ですよね
ご冗談でしょう?名無しさん [sage] 2018/03/10(土) 01:31:29.39ID:???
質問です
スターリングサイクルの効率がカルノーサイクルと同じだとされる理由は何ですか?
分母に等積加圧の熱量が入っているせいで導いた効率の式が整理できません
ご冗談でしょう?名無しさん [sage] 2018/03/10(土) 01:36:13.25ID:???

理想気体なら
ということじゃないの
ご冗談でしょう?名無しさん [sage] 2018/03/10(土) 01:39:45.87ID:???

理想気体で計算しました
そもそも効率って
(1サイクルで系がした仕事)/(1サイクルで系が得た熱量(奪われた熱量は考慮しない))
でいいんですよね
わけわかめです
ご冗談でしょう?名無しさん [sage] 2018/03/10(土) 01:47:08.37ID:???
どうせ入ってきた熱はそのまま出てくるから、勘定にいれてないんじゃないですか?
ご冗談でしょう?名無しさん [sage] 2018/03/10(土) 01:50:43.67ID:???
どんな系でも
入った熱と出た熱が等しいなら効率に含めないってこと?
効率の定義にそのルールが暗黙に存在するのかな
ご冗談でしょう?名無しさん [sage] 2018/03/10(土) 02:11:31.33ID:???
でぼぼぼーん
ご冗談でしょう?名無しさん [] 2018/03/10(土) 05:12:08.55:S1Dcprxi
ニュートン卿は月は何故に落ちてこないのか?という素朴な疑問から万有引力の法則を発見した

これは英文学者の中野好夫先生の「スウィフト考」を読まなくても

万有引力の法則を完全に理解すれば、逆二乗の公式のmは月のmoonの頭文字のmだとヒラメクはずだ
ご冗談でしょう?名無しさん [sage] 2018/03/10(土) 06:34:02.13ID:???
「全」を完全に消滅させたらどうなりますか?
ご冗談でしょう?名無しさん [sage] 2018/03/10(土) 10:37:33.47ID:???

2つの等積変化の熱の出入りは,再生によって再利用する
とすればカルノー効率になる
ご冗談でしょう?名無しさん [sage] 2018/03/10(土) 11:01:48.45ID:???

この「再生」の作業をする機械は
結局熱源なんじゃないの
効率に含めなくていい特別な熱源なのか
だとしたら効率の定義の分母に暗黙の仮定を課していることになるけど
ご冗談でしょう?名無しさん [sage] 2018/03/10(土) 12:28:01.75ID:???

少しは自分で調べろよ
スターリングサイクルの解説なんて
いくらでもころがってるから
ご冗談でしょう?名無しさん [sage] 2018/03/10(土) 12:49:32.75ID:???

その「いくらでも転がってる解説」に納得がいかないんですよ
調べまくっても
「熱力学的な考察をするときに再生器をどう扱うのか」
「再生器を熱源とするのか」
「再生器は熱源だがサイクルの系に含まれていい理由」
が載ってないです
すっごくもやもやします
ご冗談でしょう?名無しさん [sage] 2018/03/10(土) 12:58:46.79ID:???

「ほとんどの物質で成り立つ」と断言しているんだから例外の存在も同時に主張してるのと同じだろ
ご冗談でしょう?名無しさん [sage] 2018/03/10(土) 13:14:15.02ID:???

まじで最後の頼みの綱なんだよ
頼むよ
ご冗談でしょう?名無しさん [] 2018/03/10(土) 13:15:09.56:ZIDPYY8/

だから、mvはその近似式だよ。正しくはγmv
ご冗談でしょう?名無しさん [sage] 2018/03/10(土) 13:21:18.61ID:???

v=cをγに代入するとどうなるんですか?
ご冗談でしょう?名無しさん [sage] 2018/03/10(土) 13:57:31.30ID:???
揚げ足取りが生き甲斐かよ
ご冗談でしょう?名無しさん [sage] 2018/03/10(土) 14:00:13.84ID:???
揚げ足取りも何も、代入しないと運動量求められませんよ?
ご冗談でしょう?名無しさん [sage] 2018/03/10(土) 14:46:14.85ID:???
ムニちゃーん しんぷ
ご冗談でしょう?名無しさん [sage] 2018/03/10(土) 14:49:45.89ID:???

それだけ考えられて,計算もできるんだから,自分が得た結論で納得すれば
いいんだけどな
転がってる変な解説は,どうでもいいよ

>スターリングサイクルの効率がカルノーサイクルと同じだとされる理由は何ですか?

(いろんな意味で)理想的な再生器があって,それと動作物質からなる系を考え,
二つの熱源との間で熱をやり取りする***というサイクルを考えると,
***と効率を定義すれば,それはカルノーサイクルと同じになる

「熱力学的な考察をするときに再生器をどう扱うのか」
好きなように扱え
上ではサイクルを行う系の一部とした

「再生器を熱源とするのか」
熱源と扱いたければ好きにしろ
上では熱源とはしなかった

「再生器は熱源だがサイクルの系に含まれていい理由」
そんな理由はない
そうしてはいけない理由もない

こういう風に定式化したら,こうなりました
全体でつじつまが合っていればいい
ご冗談でしょう?名無しさん [] 2018/03/10(土) 14:53:40.25:ZIDPYY8/

式が意味を持たないから使えない。
一方、p=mvだとv=cでも式が意味を持つから、そこで間違った
思い込みをする馬鹿が出る。
ご冗談でしょう?名無しさん [sage] 2018/03/10(土) 14:57:12.68ID:???

使えないのになんであなたはその式持ち出したんですか?
ご冗談でしょう?名無しさん [sage] 2018/03/10(土) 14:59:46.49ID:???

再生器を系の中に入れると等積加圧の時に熱力学第二法則を破っちゃう気がするんだけども無視してオッケーなのかな
実は破ってないのか、それとも破ってるけどそれを無視しているのか
ご冗談でしょう?名無しさん [sage] 2018/03/10(土) 15:02:46.70ID:???

>(いろんな意味で)理想的な再生器があって
ご冗談でしょう?名無しさん [sage] 2018/03/10(土) 15:04:13.13ID:???

まじか
ずるいなー
ご冗談でしょう?名無しさん [] 2018/03/10(土) 15:05:12.88:ZIDPYY8/

p=mvは正しい式ではないからだよ。
何度も言わせるな。
ご冗談でしょう?名無しさん [sage] 2018/03/10(土) 15:13:48.85ID:???


なに二人でいちゃついてんだ
ご冗談でしょう?名無しさん [sage] 2018/03/10(土) 15:15:35.12ID:???

第二法則を破っているわけではないよ

カルノーサイクルだって
準静断熱操作にしてエントロピー一定にしてるだろ
それと同じような理想化
ご冗談でしょう?名無しさん [sage] 2018/03/10(土) 16:13:43.17ID:???

あぁそうか
気体の温度が一つのスターリングサイクルだと、準静的な等積変化にしても熱が外に出てっちゃうけど
気体の温度が二つあるスターリングサイクルなら準静的な等積変化であれば熱が外に出ていくことは無いね
めっちゃスッキリした
てことは二気体なら等温変化と等温以外の準静的な変化でカルノーサイクルと同じ効率に出来るってことだな
不思議っすね! ありがとうございます
ご冗談でしょう?名無しさん [sage] 2018/03/10(土) 16:28:41.48ID:???

光の運動量も求められない式が正しい式なんですか?
ご冗談でしょう?名無しさん [sage] 2018/03/10(土) 16:30:33.00ID:???

あっでも
カルノーサイクルと違って計算が面倒になりそう
違う温度の気体が変化するわけだから一筋縄ではいかないよねこれ
ご冗談でしょう?名無しさん [] 2018/03/10(土) 17:16:40.65:ZIDPYY8/

p=mvでも光の運動量が求められないのは同じことなんだが。
ご冗談でしょう?名無しさん [sage] 2018/03/10(土) 17:21:58.59ID:???

では、正しい求め方を教えてください
ご冗談でしょう?名無しさん [] 2018/03/10(土) 18:08:03.88:ZIDPYY8/

だから、 に書いた通り、γに依存しない E/p=c^2/v の関係式が
光子にも当てはまることを使えばいい。

あるいは、古典的な電磁気学から光の放射圧がP=u/3 (uはエネルギー
密度)になることと、気体運動論的な考察から速度v、運動量pの粒子
による圧力が P=2p(nv/6) (nは粒子密度)になることを用いて、
v=c、u=nhνを代入すれば、 nhν/3=pnc/3となるので p=hν/c
が求まる。
ご冗談でしょう?名無しさん [sage] 2018/03/10(土) 18:13:33.37ID:???

なぜは使えないんですか?
ご冗談でしょう?名無しさん [sage] 2018/03/10(土) 18:40:44.36ID:???
こじつけの式だから
ご冗談でしょう?名無しさん [sage] 2018/03/10(土) 18:53:39.13ID:???
イチャモン付けたいだけの劣等感など相手にすんな
ご冗談でしょう?名無しさん [] 2018/03/10(土) 19:18:49.08:ZIDPYY8/

γ=1/0になるからだって、何度言わせれば…
ご冗談でしょう?名無しさん [sage] 2018/03/10(土) 19:51:19.72ID:???
質問です。
電磁気学や量子力学によく出てくる計算で、有限サイズ(L*L*L)の箱を考えて
周期境界条件或いは境界では0条件をつけて 波数空間での状態数を数えたりしますよね。
L → +∞ の極限とって和(Σ)から 積分(∫ )に移行したりとか。
あれって、箱の形を矩形のボックス以外、例えば ドーナツ型、洋梨型、なんでもアリにして
それでも結果が同じになる事はどうしたら証明できるんでしょうか?
物理的な直感では「明らかだろ」と思うんですが、その辺りちゃんと書いてある本とかありますか?
ご冗談でしょう?名無しさん [sage] 2018/03/10(土) 19:56:28.43ID:???
積分表面で場の大きさが0になる議論なら積分表面の形はどうでもいい。
ご冗談でしょう?名無しさん [sage] 2018/03/10(土) 19:59:33.52ID:???
略した議論はできそうだけど
数学的に厳密にとなるとどうなるんだ?
周期だけで見ても3次元方向だけじゃ足りない?
ご冗談でしょう?名無しさん [sage] 2018/03/10(土) 20:15:37.61ID:???

発散するから無視するとかとんでもないですね

で?その式が使えないから、どういう運動量の定義式使うわけですか?
ご冗談でしょう?名無しさん [sage] 2018/03/10(土) 20:23:56.37ID:???
数学で厳密な議論のできる物理学者は世界でも数人しかいないってどこかで聞いた
ご冗談でしょう?名無しさん [sage] 2018/03/10(土) 20:37:26.10ID:???

普通同じ結果にはならない
有限サイズ効果の一種
ご冗談でしょう?名無しさん [sage] 2018/03/10(土) 20:52:09.17ID:???

問題設定が曖昧だからよくわからんが、もともとR^3で偏微分方程式を解くこととすると
R^3での解の存在、一意性、初期値に対する連続性は箱からの体積無限大の極限では議論しない、R^3の上で最初からやる。
電磁気学や量子力学に出てくる問題は最初から一意性があるから変な極限を取らない限りOK
てなかんじ。
ご冗談でしょう?名無しさん [sage] 2018/03/10(土) 21:11:49.37ID:???

ちょっと何言ってるか分かんない
ご冗談でしょう?名無しさん [sage] 2018/03/10(土) 22:08:03.71ID:???
ムーニーちゃんしんぷだ〜い
ご冗談でしょう?名無しさん [sage] 2018/03/10(土) 22:23:42.97ID:???

数学の偏微分方程式論の本を読みなさいということだな
入門なら

偏微分方程式論 金子

量子力学は関数解析の知識が必要だけど

量子現象の数理 新井
ご冗談でしょう?名無しさん [sage] 2018/03/10(土) 22:32:59.20ID:???

普通熱力学的考察をするときは気室が一つじゃない?
最初スターリングサイクルの効率を計算したときは気室が一つだとして考えてた
でも実際のエンジンは二つで、その間に再生器をくっつけてあるって分かって根本的な仮定が異なってたんだなって
ご冗談でしょう?名無しさん [sage] 2018/03/10(土) 22:57:20.82ID:???

一つでもいいんじゃないの
ご冗談でしょう?名無しさん [sage] 2018/03/10(土) 23:09:42.16ID:???

そりゃあさ
等積過程のときに都合よく熱を奪ったり与えたりすればいいんだろうけど
それってもう外部熱源と区別できなくない?
区別しろって言われたらそれまでだし、区別しなかったらカルノーサイクルと同効率にならないし
ご冗談でしょう?名無しさん [sage] 2018/03/10(土) 23:30:25.98ID:???

それはそのとおり

変な解説や本に惑わされず先に進もう
704 [sage] 2018/03/10(土) 23:49:12.48ID:???
曖昧とのことなので具体例をあげます、
例えば マクウェル・ボルツマン分布を量子力学的に導くとかそういうやつです。
ttp://http://hep.ph.liv.ac.uk/">ttp://hep.ph.liv.ac.uk/~hock/Teaching/StatisticalPhysics-Part3-Handout.pdf
この p.6 にあるような話です。

有限矩形ボックスなら
波動関数 ψ(x, y, z) = A sin(kx*x) sin(ky*y) sin(kz*z)
を指定するパラメータとして波数(kx, ky, kz) が離散的になっていて、当然エネルギーも離散的です。
で、このサイズを無限大極限に持っていくと連続的な(というか一様な)状態密度が得られるわけです。
ここから分配関数Z やエネルギー密度分布を計算する。
いろんな教科書で似たりよったりな導出をやってますよね。

今度は 任意形状の有限サイズの空間を考えます。
周期境界条件は矩形ボックスでしか使えないので、境界で0ってことにします。
やはり何らかの量子数の組で波動関数が指定できるはずです。
ドーナツ(トーラス)程度なら解析解があるかもしれません。
この波動関数の形状や状態分布も矩形ボックスの場合とは異なるでしょう。
でもここから無限大極限をとれば、同じエネルギー密度分布が得られるはず。
直感的にも実験的にそうなんだし、そうに決まってる、てのは置いておいて、
ちゃんとした保証が欲しいです。

ご冗談でしょう?名無しさん [sage] 2018/03/10(土) 23:55:41.48ID:???

肝はそこなんだね
等積変化を熱源由来の熱とするか系内の熱の移動とするかで効率が変わる
ただそれだけだったんだ
その事実をどう解釈するのかとか、現実世界でどう再現するのかは工学の領域になるわけか(それで満足できない人は具体的な効率計算をするのかもしれないけど)

数学の道具で物理が語られ
物理の道具で工学が語られる
典型的な例だなぁうん

ありがとう!スッキリした
ご冗談でしょう?名無しさん [sage] 2018/03/11(日) 00:18:19.11ID:???
ご職業なんですか?煽りじゃなくて、かなり詳しそうなんで気になって
ご冗談でしょう?名無しさん [sage] 2018/03/11(日) 00:41:56.74ID:???
太鼓の聞き分け。
ご冗談でしょう?名無しさん [] 2018/03/11(日) 01:36:31.47:dof84fUy

>発散するから無視するとかとんでもないですね

とんでもなくはないでしょ。v=cで使えないことが自明なんだからとてもよく出来てる。
値が計算できるのに使えないって式(p=mv)のほうがよっぽどとんでもない。

>で?その式が使えないから、どういう運動量の定義式使うわけですか?

p=Ev/c^2 という定義式でいいんじゃないの?なぜわからん?
ご冗談でしょう?名無しさん [sage] 2018/03/11(日) 02:15:03.05ID:???

なぜそのように「定義」されているんですか?
本当の定義式はラグランジアン使うんじゃないですか?
ご冗談でしょう?名無しさん [sage] 2018/03/11(日) 02:49:55.78ID:???
「全」は何円ですか?
ご冗談でしょう?名無しさん [sage] 2018/03/11(日) 03:49:03.65ID:???

きれいにp-V図を書いて、そこに断熱変化の補助線を書いてみたら分かるん
じゃない? 熱の移動がどちら向きなのか分かるよ。
411 [] 2018/03/11(日) 04:48:42.65:Iqjt3ZPL
質問します!

の続き
地球・星から見た時計
     出発   ターン開始   ターン終了   到着
地球  0      1        2        3
星    0     1        2        3
宇宙船 0      0.5      1        1.5

宇宙船から見た時計
地球   0      0.25     2.75      3
星    0.75    1       2        2.25
宇宙船 0     0.5       1        1.5

1. 新しく、星の時計を加えてみました、これで正しいでしょうか?
2. ターンしているときの星の時計Δt'と宇宙船の時計Δtはうまく
Δt'=2*Δtになっていて、円盤の中心の慣性系と回転座標系の
時間の関係になっているのですが、地球の時計のΔt''=5*Δtの関係が
よく分かりません。どう計算したら良いのでしょうか? 地球との距離
0.866c[光秒]と関係あるのでしょうか? 
ttp://iup.2ch-library.com/i/i1894862-1520711212.png
ご冗談でしょう?名無しさん [sage] 2018/03/11(日) 10:16:50.36ID:???

統計力学はたぶんのっていない。

>有限矩形ボックスなら・・・
一般形状の固有値を周期的、ディリクレ境界条件とかよくわかってる奴で挟んでおいて熱力学的極限をとる
てなかんじかな。論文にはなってると思うけど教科書はすぐにはおもいつかん。
ご冗談でしょう?名無しさん [sage] 2018/03/11(日) 10:17:50.07ID:???

非線形屋さんか
ご冗談でしょう?名無しさん [sage] 2018/03/11(日) 10:20:49.28ID:???
非線形の教授は
「普通の(線形な)理論は一回解析的に解いたら後は数値を当てはめるだけ。
 非線形は毎回解く必要があるからちょっとパラメータいじるだけでなんでも研究になる」
と言ってたな
ご冗談でしょう?名無しさん [sage] 2018/03/11(日) 11:03:41.19ID:???
太鼓の音を聞いてその形状を求める問題、というつもりなのだが

太鼓の形を聴けるか?
ttp://www.geocities.jp/ikuro_kotaro/koramu/5755_g4.htm
ご冗談でしょう?名無しさん [sage] 2018/03/11(日) 11:15:30.15ID:???
人間がコロコロ態度を変えるのは非線形応答そのものだな
ご冗談でしょう?名無しさん [sage] 2018/03/11(日) 11:36:58.08ID:???

こっちだな、但し量子統計力学なので作用素環の知識もいる

量子統計力学の数理 新井
ご冗談でしょう?名無しさん [sage] 2018/03/11(日) 12:31:02.19ID:???
そんな難しいもんやってると一生があっという間に終わるぞ
ご冗談でしょう?名無しさん [sage] 2018/03/11(日) 12:59:16.40ID:???

「○○は数人しかいない」は良く使われる素人騙しのハッタリ
ご冗談でしょう?名無しさん [sage] 2018/03/11(日) 13:03:28.75ID:???

自分で簡単に証明できるやん
任意形状の内と外から箱で挟んで評価するだけ
ご冗談でしょう?名無しさん [sage] 2018/03/11(日) 13:11:29.20ID:???

あっそうなんだ
確かにこれ大学の説明会で聞いただけだしそれで感心しちゃってた
ご冗談でしょう?名無しさん [sage] 2018/03/11(日) 13:51:28.81ID:???

研究に値することだよ、ゴールはそこではないが


数学がある程度わかっていないと証明は無理
ご冗談でしょう?名無しさん [sage] 2018/03/11(日) 14:21:36.27ID:???

それ俺が既に指摘済みだがw
ご冗談でしょう?名無しさん [sage] 2018/03/11(日) 14:49:47.53ID:???
ムニちゃんしんぷダノン
ご冗談でしょう?名無しさん [sage] 2018/03/11(日) 15:25:36.70ID:???
,
小さい箱(S)、大きい箱(L)で挟みこんで、
元の空間(M)の固有関数(エネルギーE) を、Lで固有関数展開するとやはりエネルギーEにピークを持つ。
サイズを大きくしたらピーク幅はいくらでも小さくなるはず。
同様に Sの固有関数(エネルギーE') を、の固有関数で展開 〜...
S → M → L の各エネルギー状態に対応した状態が存在するので、
状態数の分布が S, L 挟み撃ちで一致する → エネルギー密度分布も一致する。
みたいなストーリーですかね。
これエネルギー縮退がないならいいんですが、実際は縮退ありますよね。
1対1対応をちゃんと見つけられるんですかね?という疑問がわきます。
ご冗談でしょう?名無しさん [sage] 2018/03/11(日) 16:13:27.08ID:???

分かりやすく言うと枠組みと必要な数学を勉強しないと無理だということだ。俺がいった言葉だけでもわかっていないだろう
。お前がいってることを定理として書けるのか?ましてや証明なんか無理無理。
ご冗談でしょう?名無しさん [sage] 2018/03/11(日) 16:22:55.09ID:???
分かんない、お手上げ
ご冗談でしょう?名無しさん [sage] 2018/03/11(日) 16:29:07.97ID:???
誰が誰やら分からないのに、
> 俺がいった言葉だけでもわかっていないだろう
言われてもなー
ご冗談でしょう?名無しさん [sage] 2018/03/11(日) 16:41:14.17ID:???
お前なんかしらねー
ご冗談でしょう?名無しさん [sage] 2018/03/11(日) 16:42:13.63ID:???
光の運動量の「定義」はp=Ev/c^2だそうです
どういうことですか?
ご冗談でしょう?名無しさん [sage] 2018/03/11(日) 16:55:38.15ID:???
光に限らん。
ご冗談でしょう?名無しさん [sage] 2018/03/11(日) 16:55:59.72ID:???
よくある物理の数学に関するちょとした疑問は思いつきで解けるような問題ではありません(笑)
ご冗談でしょう?名無しさん [sage] 2018/03/11(日) 17:45:31.54ID:???
数学を勉強する気がなければ気にしてもしょうがないとも言える
ご冗談でしょう?名無しさん [sage] 2018/03/11(日) 20:35:11.30ID:???

どういうことですか?
ご冗談でしょう?名無しさん [] 2018/03/11(日) 21:11:40.07:0q4YlPXK
スペースエレベーター - サイエンスフィクションは人類の未来? Space Elevator – Science Fiction or the Future of Mankind?
ttp://rainbow.cafemix.jp/?sop:v/qPQQwqGWktE!UUsXVk37bltHxD1rDPwtNM8Q#MIX
ttp://i.ytimg.com/vi/qPQQwqGWktE/mqdefault.jpg
ご冗談でしょう?名無しさん [sage] 2018/03/11(日) 22:48:55.23ID:???

光以外のどんな粒子にでもその関係式は成立する。
ご冗談でしょう?名無しさん [sage] 2018/03/11(日) 22:53:51.70ID:???

どうしてですか?
ご冗談でしょう?名無しさん [sage] 2018/03/11(日) 23:07:35.16ID:???
自然がそうなってるから。
ご冗談でしょう?名無しさん [sage] 2018/03/11(日) 23:16:02.15ID:???
pの定義はどうなっているんですか?
ご冗談でしょう?名無しさん [sage] 2018/03/11(日) 23:27:11.45ID:???
物理法則はものすごく普遍的絶対的なもので、何があろうと絶対に破られないもんだ
っていう妄想をいつ捨て去れるか
ご冗談でしょう?名無しさん [sage] 2018/03/11(日) 23:27:47.53ID:???
物理学におけるパラメーターの定義についてもまた然り
ご冗談でしょう?名無しさん [sage] 2018/03/12(月) 03:31:54.96ID:???

相対論的運動量はmvではうまく行かない

ローレンツ変換できる4元運動量ベクトルの導入

不変長=-(mc^2)^2, (cp0)^2=(mc^2)^2+(c|p↑|)^2

m=0でE=p/c
p↑=0でE=mc^2
だったかな。
ご冗談でしょう?名無しさん [] 2018/03/12(月) 08:51:17.97:GyHlPDE5
重力は空間を歪めるとよく聞きますが
電磁力や強い力、弱い力で空間は歪みますか
ご冗談でしょう?名無しさん [sage] 2018/03/12(月) 10:30:50.01ID:???
>重力は空間を歪めるとよく聞きますが
違います。エネルギー・運動量が時空を歪めた結果が重力です

>電磁力や強い力、弱い力で空間は歪みますか
エネルギー・運動量があれば時空を歪めます
ご冗談でしょう?名無しさん [sage] 2018/03/12(月) 11:11:58.58ID:???

物理法則ってそういうもんだからね。そんなものが存在し得ない
という証拠が見つかるまではそうだと認めざるを得ない。
良くも悪くも経験科学なんですよ。
ご冗談でしょう?名無しさん [sage] 2018/03/12(月) 11:16:23.19ID:???
物理法則は絶対だけど、人間がそれを正しく理解できないから、「絶対じゃないんじゃない?」という、疑惑が生まれる
ご冗談でしょう?名無しさん [sage] 2018/03/12(月) 11:31:44.13ID:???
実験の結果を説明できないとねぇ。それが第一義。
ご冗談でしょう?名無しさん [sage] 2018/03/12(月) 11:39:06.28ID:???
疑いたいから疑ってるだけだろ(笑)
ご冗談でしょう?名無しさん [sage] 2018/03/12(月) 11:50:48.56ID:???
現代物理は時間的経済的理由から実験や観測で証明しにくいスケールになってきてるよね
どうしても理論が先行しちゃう

理論が正しいと言い切りたいなら実験で証明する必要がある
じゃあ原理的に実験不可能なら?
これからの物理学がどうなるのか不安
ご冗談でしょう?名無しさん [sage] 2018/03/12(月) 13:21:34.91ID:???
原理的に実験不可能なものは物理と言わん
物理に関係したそういうものは解釈と呼ばれる
ご冗談でしょう?名無しさん [sage] 2018/03/12(月) 14:34:12.62ID:???
自分で実験もできないくせにw
ご冗談でしょう?名無しさん [sage] 2018/03/12(月) 14:57:43.29ID:???
理論屋は何日もかかる器機の調整とか膨大なデータ処理とか無くて羨ましい
本当にな!
ご冗談でしょう?名無しさん [sage] 2018/03/12(月) 17:00:19.10ID:???
そこが面白いんじゃないか
ご冗談でしょう?名無しさん [] 2018/03/12(月) 18:52:55.15:ny9QXN8k
宇宙戦艦ヤマトのコスモタイガーとかコスモゼロって
宇宙空間で飛行機の形をしてることに意味はあるの?
(´・ω・`)
ご冗談でしょう?名無しさん [sage] 2018/03/12(月) 18:55:52.78ID:???
その前にヤマトの形が意味無いから
ご冗談でしょう?名無しさん [sage] 2018/03/12(月) 19:11:34.74ID:???
真空には量子ゆらぎがあるらしいけど水中とか空気中には無いの?
無いとしたらなんでないの?
ご冗談でしょう?名無しさん [sage] 2018/03/12(月) 20:05:56.60ID:???
真空ってなんだと思う?
ご冗談でしょう?名無しさん [sage] 2018/03/12(月) 20:22:58.88ID:???

松本零士作品に科学的整合性を求めることに意味はあるの?
ご冗談でしょう?名無しさん [sage] 2018/03/12(月) 21:27:29.45ID:???

wikiでは圧力が低いところってあるけど量子レベルだと水でもスカスカだからなんで起きないのかなって思った
ご冗談でしょう?名無しさん [sage] 2018/03/12(月) 21:35:25.76ID:???

その真空とは違うの、素粒子の世界の話
ご冗談でしょう?名無しさん [sage] 2018/03/12(月) 21:36:49.01ID:???
ちがわい ちがわい
ご冗談でしょう?名無しさん [sage] 2018/03/12(月) 22:54:18.83ID:???
偽の真空とか真の真空が素粒子の真空?
準安定状態が偽の真空とか書いてあるけど量子ゆらぎが身近に起きるのかわかんない
ご冗談でしょう?名無しさん [sage] 2018/03/13(火) 00:15:58.14ID:???

数値データ処理なら山程あるぞ
ご冗談でしょう?名無しさん [] 2018/03/13(火) 01:03:14.14:tJHrQzcv

遅レスですまんな。
d'Q=dU+pdV
だな。
理想気体としよう。U=c・nRT, pV=nRTなんだよね。
代入してU=c・pVだな。d'Q=cVdp+cpdV+pdV=cVdp+(c+1)pdV
等積ならdV=0, d'Q=cVdp, c>0, V>0なら熱の移動は必ずdpの符号に
従うわけ。

高熱源T1, 低熱源T2として、等温過程で熱の出入りを疑わないのは不思議
だが、まあ納得してるとして、www
等温膨脹で(p1, V1)->(p2, V2)となるとすれば、p2=(V1/V2)p1
等積でT2まで下げるとp3=(T2/T1)p2
等温で圧縮すればp4=(V2/V1)p3かな。適当に確かめてくれ www
∫d'Q2=∫cV2dp[p2, p3]
∫d'Q4=∫cV1dp[p4, p1]
を足してゼロを確かめれば良いはずだね。
ご冗談でしょう?名無しさん [sage] 2018/03/13(火) 01:15:51.17ID:???
の続き、
(p1, V1), (p2, V2), (p3, V2), (p4, V1)
が分かれば、d'Wは計算ができ、
等温ではdU=0, d'Q=-d'Wが分かれば、
熱効率ηは計算できるね。
ご冗談でしょう?名無しさん [sage] 2018/03/13(火) 01:21:17.58ID:???

一回人を貶さないと気が済まないのかよー
素直じゃないなぁ
まぁありがとうございます
馬鹿にしたくなる気持ちは良くわかります
実際熱は移動してないって思ってました
気室が一つの時と二つの時で色々ごっちゃになって訳分かんなくなってました
ご冗談でしょう?名無しさん [sage] 2018/03/13(火) 01:23:47.29ID:???

具体的な計算方針を示してくれてありがとう!
ちょっと意地悪な案内人だったけど根は良い奴だよね
ご冗談でしょう?名無しさん [sage] 2018/03/13(火) 01:51:16.68ID:???

等温でのd'Wはp=nRT1/Vだから d'W=-d'Q=((nRT1)/V)dVだと分かるよね?
ご冗談でしょう?名無しさん [sage] 2018/03/13(火) 01:54:39.25ID:???
の続き
スマン符号を間違えた。
d'Wはp=nRT1/Vだから d'W=-d'Q=-((nRT1)/V)dV
符号周りは確認してくれ〜
ご冗談でしょう?名無しさん [sage] 2018/03/13(火) 02:03:27.30ID:???

うん
積分したら対数になるよね
ご冗談でしょう?名無しさん [sage] 2018/03/13(火) 03:31:52.61ID:???

理想気体のエントロピーS=nRln(TV/n)を使って良いのなら、
dS=nR((1/T)dT+(1/V)dV)で
d'Q=TdS=nRdT+nR(T/V)dV
というのも、いちいち圧力の計算をしなくてもいいんだが...
ご冗談でしょう?名無しさん [sage] 2018/03/13(火) 03:58:43.32ID:???

α飛んでたな、スマン。www
ご冗談でしょう?名無しさん [sage] 2018/03/13(火) 09:13:57.58ID:???

分かる訳がないから心配するな
ご冗談でしょう?名無しさん [sage] 2018/03/13(火) 09:32:19.09ID:???

水でも起きてる

>wikiでは圧力が低いところってあるけど
んなもん鵜呑みにするな
ご冗談でしょう?名無しさん [sage] 2018/03/13(火) 09:34:40.30ID:???
「ゆらぎ」を蜃気楼のように見えるものだと勘違いしてるんだろう
ご冗談でしょう?名無しさん [sage] 2018/03/13(火) 09:57:48.57ID:???
かげろうの間違いだわ
ご冗談でしょう?名無しさん [sage] 2018/03/13(火) 10:24:25.93ID:???

教えてくれてありがとう
ご冗談でしょう?名無しさん [sage] 2018/03/13(火) 10:31:18.30ID:???

プローブのエネルギースケール次第では見える
ご冗談でしょう?名無しさん [sage] 2018/03/13(火) 10:32:49.39ID:???

プローブって何?
ご冗談でしょう?名無しさん [sage] 2018/03/13(火) 10:52:05.17ID:???
カシミール効果は実験的に確かめられているようだが
ttp://web.mit.edu/kardar/www/research/seminars/Casimir/PRL-Lamoreaux.pdf
ご冗談でしょう?名無しさん [sage] 2018/03/13(火) 10:55:25.99ID:???
動的カシミール効果というのもあるようだ
ご冗談でしょう?名無しさん [sage] 2018/03/13(火) 11:19:02.13ID:???
の訂正
d'Q=dU+pdVで
理想気体ならU=c・nRT, pV=nRT
∂T・U=c・nR, ∂V・U=0, p=nRT/Vだから
d'Q=c・nR dT+nRT/V dV
dS=c・nR/T dT+nR/V dV
だな。
等積なら、d'Q=c・nR∫dT
等温T=T0なら、d'Q=nRT0・∫(1/V)dV
ただ、第3法則T=0でS=0は成り立たない。
ご冗談でしょう?名無しさん [sage] 2018/03/13(火) 11:44:43.66ID:???

等積ならd'Q=Cv・n(T2-T1)だから、同じ温度差の間で移動して
もどってくれば電'Q=0, 電'W=0, 電S=0
等温ならd'Q=nRT1・ln(V2-V1)だから、V1->V2、V2->V1で
計算しても良いんだが、片方のΔSは片方の-ΔSに等しいはず、
η=(Q1-Q2)/Q1なら、
=(T1ΔSーT2ΔS)/T1ΔS=(T1-T2)/T1でカルノーサイクルと同じ
カルノーサイクルでは断熱過程でΔS=0だったから、最初から↑の式が
成り立つ。
ご冗談でしょう?名無しさん [sage] 2018/03/13(火) 14:21:23.25ID:???

最近のH立のSPMすごいな
ご冗談でしょう?名無しさん [sage] 2018/03/13(火) 18:39:50.83ID:???
ムニちゃんポポダノンのおじさん
ご冗談でしょう?名無しさん [] 2018/03/14(水) 09:32:32.29:1WYgtu/e
太極図、エネルギー潮流でできる地図。
上記 登記
ご冗談でしょう?名無しさん [sage] 2018/03/14(水) 12:43:19.25ID:???
惨めな奴
ご冗談でしょう?名無しさん [sage] 2018/03/14(水) 13:05:43.09ID:???

実は、ヤマトの形には、ガミラスの目を欺くために沈没船に偽装する、という意味がある。
ご冗談でしょう?名無しさん [sage] 2018/03/14(水) 13:41:33.25ID:???
ホーキング博士死去。
ご冗談でしょう?名無しさん [sage] 2018/03/14(水) 13:46:58.09ID:???
ブレイクスルースターショット計画はどーなるかな
ご冗談でしょう?名無しさん [] 2018/03/14(水) 14:47:22.13:ORqV7WtR
熱力学関数ってS,TとV,pとN,μの組み合わせで合計8通りあってそのうち(T,p,μ)の組み合わせはギブスなんちゃらの関係式で0になるから7通りあると思うんですけど
U(S,V,N),H(S,p,N),F(T,V,N),J(T,V,μ),G(T,p,N)は名前があって残りの(S,V,μ)と(S,p,μ)ってなんか名前あるんですか?
ご冗談でしょう?名無しさん [sage] 2018/03/14(水) 15:52:13.49ID:???
ないよ
ご冗談でしょう?名無しさん [sage] 2018/03/14(水) 21:35:18.29ID:???
ムーニーちゃんしんぷなんだい(><;)
ご冗談でしょう?名無しさん [sage] 2018/03/14(水) 23:14:10.09ID:???

変数的には対称なんだけれど使い道があるか、だね。
U -> 本質
H -> 等温・等圧での熱の出入りそのもの
F -> 等温・等積での取り出せるエネルギー、エンジンなど
G -> 等温・等圧で取り出せるエネルギー、化学反応・相転移など
このぐらいあれば、だいたい用が足りるわけだ。後は使いやすいものを
使うわけだけれど、使い慣れてないものでの計算や証明は説得力が
ないかもね。
ご冗談でしょう?名無しさん [sage] 2018/03/15(木) 01:02:43.13ID:???
クラマース関数とかって何だっけ
ご冗談でしょう?名無しさん [sage] 2018/03/15(木) 09:22:36.00ID:???
ドイツ人コーチ
ご冗談でしょう?名無しさん [sage] 2018/03/15(木) 15:47:39.93ID:???
数学の問題かもしれませんがどちらが正しいのですか?
ttps://i.imgur.com/zn0K89g.jpg
ttps://i.imgur.com/R3yzp9Q.jpg
ご冗談でしょう?名無しさん [sage] 2018/03/15(木) 17:09:48.50ID:???
リンク見るなんて危ない事せんわ
ご冗談でしょう?名無しさん [] 2018/03/15(木) 18:59:15.36:rrpoqzL9
表面張力って調べたところ流体力学でやるみたいなんですが、高校力学ではやりませんよね?
あと、他にも面白い力学の分野があれば教えて頂きたいです!
ご冗談でしょう?名無しさん [sage] 2018/03/15(木) 20:39:22.35ID:???

両者は全く同じ
ご冗談でしょう?名無しさん [sage] 2018/03/15(木) 20:42:47.43ID:???
解析力学面白いですよ
ご冗談でしょう?名無しさん [] 2018/03/15(木) 20:55:02.37:d7eyqQUI

超電導とか超流動とかで調べてみると面白い実験動画が載ってる
ご冗談でしょう?名無しさん [sage] 2018/03/15(木) 21:13:34.13ID:???
ホーキング博士が亡くなったのがアインシュタインの生まれた日ってすごい偶然だな。アインシュタインが転生して跡を継いだのかな。
ご冗談でしょう?名無しさん [sage] 2018/03/15(木) 21:49:04.50ID:???
すごい偶然とはジャンボ宝くじの1等が当たった人位な、確率を考えれば300円当たりくらい。
ご冗談でしょう?名無しさん [sage] 2018/03/15(木) 22:57:28.08ID:???

この場合の式変形ですと両者でQ(0)の有無が変わってしまいました。何故なんですか?
ttps://i.imgur.com/vZ1DNEA.jpg
ご冗談でしょう?名無しさん [sage] 2018/03/15(木) 23:09:12.96ID:???

上の方ではQ(x)が内力と言っているのにQ(0)が外力とか言ってるのがおかしいんじゃない?
ご冗談でしょう?名無しさん [sage] 2018/03/15(木) 23:18:08.57ID:???

誰の何という本かは知らないが、積分変数と本当の変数をごちゃまぜにしてる時点で救いようのないアホだから
その本は窓から投げ捨てるか、燃やして暖を取るかしか利用方法がない。
その本で何かを理解しようとするのはハイリスクノーリターン。
ご冗談でしょう?名無しさん [sage] 2018/03/15(木) 23:20:14.34ID:???

dQ(x)=-q(x)dxをある値からある値まで積分すると定積分になりますが、この場合任意のxにおけるQ(x)を求めるので不定積分扱いのような気もしてきました。
なのでdQ(x)/dx = -q(x) ⇒Q(x)=∫-q(x)dx +Cで初期条件Q(0)よりCを出すのかなと
ご冗談でしょう?名無しさん [sage] 2018/03/15(木) 23:33:21.67ID:???

ちゃんと書けば、(3.8)の両辺をx=0からx=ξまで定積分すると
Q(ξ) - Q(0) = -∫[0, ξ] q(x) dx (正しい3.9)
になって、ξ=0のとき両辺とも0になって辻褄が合う。
とにかくその本は捨てろ。
ご冗談でしょう?名無しさん [sage] 2018/03/15(木) 23:39:19.01ID:???

812として上げた慣性モーメントの計算は定積分なので1枚目のような計算でもいいんですか?この本のものは結局不定積分という認識でいいのでしょうか
ご冗談でしょう?名無しさん [sage] 2018/03/15(木) 23:47:42.17ID:???
よくないです。
は積分変数と積分範囲の変数を混同してるので完全に間違った式になっています。
は両者を区別しているので正しい計算です。
ご冗談でしょう?名無しさん [sage] 2018/03/15(木) 23:50:08.13ID:???

812のどちらが正しい計算名のですか?
ご冗談でしょう?名無しさん [sage] 2018/03/15(木) 23:51:21.42ID:???
どちらも正しい計算名のですよ
ご冗談でしょう?名無しさん [] 2018/03/16(金) 04:05:19.58:cArXEK1C
U=ΣPi Ei
なら、
dU=ΣdPi Ei +Σ Pi dEiで、
δW=ΣdPi Ei, δQ= Σ Pi dEiでいいんでしょうか?
感覚的に分かりにくいんですが、どういうモデルがありますか?
ご冗談でしょう?名無しさん [sage] 2018/03/16(金) 09:35:50.55ID:???

ちなみに、ホーキング博士が生まれた日がガリレオの亡くなった日だ。
ご冗談でしょう?名無しさん [sage] 2018/03/16(金) 11:07:00.94ID:???

円周率の日でもあるね
ご冗談でしょう?名無しさん [sage] 2018/03/16(金) 16:10:31.79ID:???
ぷっちー ポポ
ご冗談でしょう?名無しさん [] 2018/03/16(金) 18:18:23.17:oaGuW55l
アホばっかで吹くわ。
まったく分かってないな、サルどもが。

本来、dy/dxからyを求めるのは積分とは言わんのよ。積分とは定積分のことだからな。
この場合は「yを求める」って言えばいいだけだ。

微分してyとなる関数の記号として∫を使うだけで、これに定数Cを加える。
これを便宜上不定積と言っているが、その実体は積分ではない。微分してその関数になる関数を求めることを
あえてそう称してるだけだ。大事なことだからもう一度書くと、これは本来の積分ではないんだよ。

言ってみれば、不定積分はただの積分モドキにすぎん。
お前らはヒトモドキのサルだろ?、それと一緒だわ。
くっくっく

一方、dyから両辺を定積分してyを求めること、これが本来の積分なんだよ。
よって画像の間違いは、(3.8)が不要なことだ。(3.7)を定積分すればよい。
∫dQは差分dQの総和だから当たり前に両端の差Q(xb)-Q(xa)になる。
これが本来の積分なんだよ。

それを知らず、わざわざ必要のないdQ/dxの形にして、しかもそれを定積分と間違った称し方をしている時点で
まったく何も分かってないわ。
∫dQ/dx・dx → ∫dQなんだからわざわざdQ/dxで記述する必要ないってーの、ってか
これすら分かってないんだよな、これ書いてるヤツ。

どんだけ低レベルなアホが書いてんだよ。
どんどんアホばっかになっていってんな。

さすが若いサルどもはキチガイジミンを支持してるアホが多数派なわけだ。
ゆとりのアホザルばっか。
くっくっく
ご冗談でしょう?名無しさん [] 2018/03/16(金) 18:25:03.22:oaGuW55l
おっと、訂正だ。

× これを便宜上不定積と言っているが、
〇 これを便宜上不定積分と言っているが、

しかしよくもまあ
こんなレベルで理学本を書けるよな。

ワシなら恥ずかしくて舌かんで氏ぬレベルだわ。
これもすべて、高校数学で積分を不定積分から定積分を導くという完全に
誤った教え方をしているからこうなるんだよな。

くっくっく
ご冗談でしょう?名無しさん [sage] 2018/03/16(金) 18:31:34.96ID:???
くっくっく気持ち悪い
ご冗談でしょう?名無しさん [sage] 2018/03/16(金) 18:34:23.51ID:???
クルル伍長がペコポン星人に何か言いたげです
ご冗談でしょう?名無しさん [] 2018/03/16(金) 18:44:34.86:oaGuW55l
dQとdxで定積分すりゃいいだけものを
わざわざまったく不要なdQ/dxの形にして、しかもそれなら不定積分と称すべきところを
定積分と称したり、日本の若造どもは終わってんな。

そりゃ、レポートもコピペ、試験もコピペ、卒論もコピペなわけだ。
思考力がほとんどないサルばっか。

くっくっく
ご冗談でしょう?名無しさん [sage] 2018/03/16(金) 18:52:06.86ID:???
NG
ご冗談でしょう?名無しさん [sage] 2018/03/16(金) 19:56:41.77ID:???

懲りずに宝くじ300円当たりやってるのか
有史以来の有名な科学者は沢山いるから誕生または死亡の日付を合わせるのは簡単にできる。
この手の「奇跡」トリックは昔から詐欺に使われてる。
ご冗談でしょう?名無しさん [sage] 2018/03/16(金) 20:09:02.74ID:???

何をそんなに必死になってるんだ?
ホーキングに親でも殺されたか?
ご冗談でしょう?名無しさん [sage] 2018/03/16(金) 20:15:59.05ID:???
ホーキング詐欺にバカが騙されないようにな
ご冗談でしょう?名無しさん [sage] 2018/03/16(金) 20:18:43.96ID:???
物理学者が「同じ日付」なんて無意味なものにこだわって死んだり生まれたりするわけねえだろ
ご冗談でしょう?名無しさん [sage] 2018/03/16(金) 20:23:17.10ID:???
戸籍記録等の誕生日と実際の誕生死亡日時は時代が古いほど一致しなのが当たり前。
ご冗談でしょう?名無しさん [sage] 2018/03/16(金) 20:24:01.10ID:???

なにいってんのコイツ
あの本の著者もアホだが比較にならんくらいアホだな
ご冗談でしょう?名無しさん [sage] 2018/03/16(金) 20:30:40.43ID:???

たまたま誕生日や命日が一致したという事実を示しただけなのに、何を騒いでいるんだ?
単なる偶然に過ぎないのに、それに何らかの意味を感じてしまう既知外なんて、まず居ないだろ。
万一居たとしても、明らかに既知外なんだから、ちゃんと社会から隔離されているだろうさ。
居もしないバカやありもしない被害があると思い込んで大騒ぎして、おかしいぞ。

あっ、そうか!さては、お前自身がその稀有な既知外だったんだろ。
かつて自分が騙されたから、他の人も騙されるに違いない、と思い込んでいるんだな。
安心しろ。そんなバカはお前だけだ。
ご冗談でしょう?名無しさん [sage] 2018/03/16(金) 20:39:07.41ID:???
生まれ変わりとか言う奴は基地外か詐欺師
ご冗談でしょう?名無しさん [sage] 2018/03/16(金) 21:15:52.33ID:???
でも鼻の下のくぼみは、神様が前世のことを言ってはいけないよ、と指で押した跡なんだぜ
ご冗談でしょう?名無しさん [sage] 2018/03/17(土) 00:10:05.33ID:???
特殊マウンティング
ご冗談でしょう?名無しさん [sage] 2018/03/17(土) 02:27:01.17ID:???
ぷち ぷらーい
ご冗談でしょう?名無しさん [sage] 2018/03/17(土) 15:03:55.51ID:???

遅レスすまん。

どこから手を付けて良いか難しいんだけれど、
ミクロシステムで全ての熱力学的変数・関数を期待値で表現すると、
Sは熱力学的なパラメータ(E, V, N)が消えて、情報工学でもあるように
S=-kΣ(Pi*logPi)
なのよね。統計力学のSの定義参照のこと。-logPiが情報量、
それの期待値がエントロピーなわけ。
dS=-k Σ(logPi+1) dPiだから、dPi=0でdS=0でd'Q=0なのね。
残りがd'Wになる、感覚的にはこうかな。エネルギー準位が変わらずに
その間を遷移するのが仕事、エネルギー準位が変わるのが熱、大雑把に
そういうこと。
ご冗談でしょう?名無しさん [sage] 2018/03/17(土) 15:54:02.73ID:???
ムニちゃーんポポ
ご冗談でしょう?名無しさん [] 2018/03/17(土) 16:01:28.66:pb0UsEfM
量子コンピュータは状態が重ね合わせられたまま全パターン同時に計算できるということだけど、全パターンの中から必要な結果をどうやって取り出すのかがイメージできませぬ。
ご冗談でしょう?名無しさん [sage] 2018/03/17(土) 16:05:28.76ID:???
そうですか
ご冗談でしょう?名無しさん [sage] 2018/03/17(土) 16:43:13.12ID:???

Uはどんな変数の関数なのかよく考えろ
わからなければ
熱力学からやり直し
ご冗談でしょう?名無しさん [sage] 2018/03/18(日) 10:33:24.26ID:???
無になってもう二度と有になりたくないのですが、どうすれば良いですか?
自殺をしても無駄ですか?
ご冗談でしょう?名無しさん [sage] 2018/03/18(日) 10:53:10.87ID:???
残念ですがエネルギーも情報も無になりません
ご冗談でしょう?名無しさん [sage] 2018/03/18(日) 11:32:20.15ID:???
最初から無だから
ご冗談でしょう?名無しさん [sage] 2018/03/18(日) 11:44:28.92ID:???
最初から無だー>思考停止ー>成仏
ご冗談でしょう?名無しさん [sage] 2018/03/18(日) 13:09:18.52ID:???
NEWTON別冊でも買っとけ
sage [] 2018/03/18(日) 19:18:54.88:MOQXF5vj
質問です。

最新の超ひも理論というか万物理論というか、の動向を教えてくれませんか?
2010年以降、まったく科学雑誌も何も読んでおらず、最新の研究てどうなってるのかサッパリなので。

自分が見た中で最新は、超ひも理論とツイスター理論を統合とかっていう情報を見たな―というところまで。
※ぶっちゃけ詳しくないので、分かり易く説明してもらえると有難いです。理論的にというより直観的に分かるように。
ご冗談でしょう?名無しさん [sage] 2018/03/18(日) 19:27:06.96ID:???
これがゆとりか・・・
ご冗談でしょう?名無しさん [sage] 2018/03/18(日) 19:29:06.49ID:???
コピペだよ
ご冗談でしょう?名無しさん [sage] 2018/03/18(日) 19:30:27.82ID:???
わからないんですね(笑)
sage [] 2018/03/18(日) 19:46:32.95:MOQXF5vj

うむ、分からなくなったので教えて欲しいのだ。
ご冗談でしょう?名無しさん [sage] 2018/03/18(日) 19:47:45.70ID:???

うん、自分が別のスレッドで質問書いたやつで、返答無かったのでこっちに書いたのだ。
ご冗談でしょう?名無しさん [sage] 2018/03/18(日) 19:49:33.14ID:???

超ひも理論以外がメジャーになってたら教えてくれですね。
これが最終正解かも分からんので。
ご冗談でしょう?名無しさん [sage] 2018/03/18(日) 20:05:25.93ID:???
どうやら仏教の「空」の思想は言葉で説明できるものではないようだな。
ご冗談でしょう?名無しさん [sage] 2018/03/18(日) 20:07:21.44ID:???
まぁ、でも強いて言うなら仏教の「空」ってのは、「有るけど無い」みたいな感じなんだそうな。
ご冗談でしょう?名無しさん [sage] 2018/03/18(日) 20:17:29.77ID:???
本当は何もないけど法力によって見えるようになっているのが今
これを知ったとき宇宙開闢の瞬間を感じたわ
ご冗談でしょう?名無しさん [sage] 2018/03/18(日) 21:01:27.77ID:???
なんで物理板ってキチガイやポエマーが多いの?
ご冗談でしょう?名無しさん [sage] 2018/03/18(日) 21:35:49.84ID:???

まず、大きさがない素粒子では今の重力理論ではどこまで行っても距離の自乗に
比例して力が作用すれば無限大になってしまう。重力ポテンシャルも同様。
大きさが合った方=ひもが分かりやすい。
また、いろいろな素粒子がまた増え始めている。クオークも2*3世代で、それも
エネルギーが大きくなったらまた4世代目がでるかも知れない。これはこれまでの
還元主義で新しい基本粒子を作る話では解決しない。
質量とエネルギーが波のように周波数で増えてくれた方が説明できる、かも知れ
ないという話。
ご冗談でしょう?名無しさん [sage] 2018/03/18(日) 21:42:33.87ID:???

そやね。F=Gm1m2/r^2なら、r=0で無限になっちゃうもんね。握手会でビッグバンおきまくり。

クオークやレプトンが3族あるのは知ってたけど、4族以降が出てきても不思議じゃあ無いのか。
まあ、禁止する理由も無いか。

回答してくれてありがとですー。
ご冗談でしょう?名無しさん [sage] 2018/03/18(日) 21:43:40.24ID:???

ハエがウンコだと思って飛びついて来るんだろ
ご冗談でしょう?名無しさん [sage] 2018/03/18(日) 22:19:26.06ID:???

>r=0で無限になっちゃうもんね。
現代の繰り込み理論解釈では、そもそも無限大のエネルギーで計算するから無限大が出る
>クオークやレプトンが3族ある・・・禁止する理由も無いか。
大型加速器による実験から軽いニュートリノは3世代しか無いことが判っている
ので標準理論に当てはめればクオークやレプトンも3世代しか無いことになる。
CRENのLHC加速器やスーパーカミオカンデの実験でも新理論の兆候はまだ無い。
ご冗談でしょう?名無しさん [sage] 2018/03/18(日) 22:27:01.89ID:???

握手で質量がゼロ距離に収束しますか?
ご冗談でしょう?名無しさん [sage] 2018/03/18(日) 22:40:42.32ID:???
無限大のエネルギーでピンポイントに殴りつけると無限大のエネルギーで反発し返してくる。
ご冗談でしょう?名無しさん [sage] 2018/03/18(日) 23:32:44.44ID:???
太陽中心の核融合で生成したヘリウム原子核が
太陽の重力を振り払って宇宙空間にでるためには
どれくらいのエネルギーが必要なんでしょう?

出れ来るまでの時間が何秒ぐらいですか?

重力加速度=重力定数×質量/重心からの距離^2

重力定数は6.67*10^-11
太陽質量は1.99*10^30 kg
太陽半径は6.96*10^8 mで二乗すると4.84*10^17

よって、太陽表面の重力加速度=(6.67*1.99/4.84)*10^2=274 m/s^2
(これはまあまあ近い)

太陽中心。例えば半径1mの重力加速度は1.33*10^20 m/s^2
(本当かな)
ご冗談でしょう?名無しさん [sage] 2018/03/18(日) 23:53:03.97ID:???

繰り込み理論が分からぬ〜教えてもらえぬか。

第三までしか無いのが証明されてるのか。
ご冗談でしょう?名無しさん [sage] 2018/03/19(月) 00:01:56.05ID:???

太陽の中だから、
その物体から中心方向にある物に、中心方向に向けて引っ張られて(密度×1mの球体?)、
その物体から外の方にある物に、外方向に向けて引っ張られるから、
計算が複雑になるのかな?
ご冗談でしょう?名無しさん [sage] 2018/03/19(月) 00:34:09.88ID:???
でぼぼぼーん
ご冗談でしょう?名無しさん [sage] 2018/03/19(月) 01:31:34.00ID:???

あまり聞かないから、特に進展無いんじゃねーの
ご冗談でしょう?名無しさん [] 2018/03/19(月) 10:47:37.49:wydgTs7Z
この上の式から下の式への式変形が分かりません。

本の文章には「左右両辺のψ1に関する項、ψ2に関する項がそれぞれ打ち消し合う。」と書いています。ψがどこから出てきたのか?など検討がつかないです。



✳︎そもそも上の式はシュレディンガー方程式を満たす波動関数をψ1、ψ2とし、それぞれ定数c1、c2の定数を掛けて足したものです。


ttps://i.imgur.com/sn8MMas.jpg
ご冗談でしょう?名無しさん [] 2018/03/19(月) 10:48:05.69:wydgTs7Z
この上の式から下の式への式変形が分かりません。

本の文章には「左右両辺のψ1に関する項、ψ2に関する項がそれぞれ打ち消し合う。」と書いています。ψがどこから出てきたのか?など検討がつかないです。



✳︎そもそも上の式はシュレディンガー方程式を満たす波動関数をψ1、ψ2とし、それぞれ定数c1、c2の定数を掛けて足したものです。
ttps://i.imgur.com/TCY1Wo8.jpg
ご冗談でしょう?名無しさん [] 2018/03/19(月) 10:49:48.54:wydgTs7Z
申し訳ないです
何回も送ってしまいました
ご冗談でしょう?名無しさん [sage] 2018/03/19(月) 10:50:20.98ID:???
ψ=c1ψ1+c2ψ2とすると、上の式は、ψに関するシュレーディンガー方程式になって、ψはこの方程式の解となっています
よって、ψ1やψ2がシュレーディンガー方程式の解ならば、ψもシュレーディンガー方程式の解になると言うことを言っています
ご冗談でしょう?名無しさん [sage] 2018/03/19(月) 11:05:26.00ID:???

太陽の内部の中心近くはモノがいっぱいだからそう簡単には出てこれない
ついでに重力のせいで太陽内の密度は一様ではないから、その式では重力の計算は無理
ご冗談でしょう?名無しさん [sage] 2018/03/19(月) 14:39:28.80ID:???
ムニちゃーんポポ
ご冗談でしょう?名無しさん [sage] 2018/03/19(月) 16:04:25.34ID:???
小学校の勉強も碌に理解できてない状態から東大を目指すとなると合格するのに何年ぐらいかかると思いますか?
ご冗談でしょう?名無しさん [sage] 2018/03/19(月) 16:08:37.78ID:???
必死にすれば一年くらいで合格できるから周りの雑音に負けずにがんばれ
ご冗談でしょう?名無しさん [sage] 2018/03/19(月) 16:14:38.88ID:???
何年もそうやって同じこと言ってる人には無理でしょうね
ご冗談でしょう?名無しさん [sage] 2018/03/19(月) 16:19:39.41ID:???

マジですか!!!!!!!!!!!!!!!!!!!!??????????????
どんなに頑張ってもその状態からだと最低でも4、5年はかかるのではないでしょうか?
本当に1年で合格可能なのでしょうか?
ご冗談でしょう?名無しさん [sage] 2018/03/19(月) 16:24:38.77ID:???

出来ますとも
1年間1秒たりとも5chを閲覧しなければね
ご冗談でしょう?名無しさん [] 2018/03/19(月) 17:33:10.61:4vOnfCAV
どんだけ無能なんだアホザルどもが。

>まず、大きさがない素粒子では今の重力理論ではどこまで行っても距離の自乗に
>比例して力が作用すれば無限大になってしまう。重力ポテンシャルも同様。
>大きさが合った方=ひもが分かりやすい。

質量も電荷も
宇宙に存在するものすべては
どれだけ細分化しようが点ではなく密度で存在するから
無限大に発散することはない。

質量密度や電荷密度で存在しているのだから
r=0で発散などしない。

例えば万有引力の微小極限では
分子にある質量はrの3乗に比例して小さくなるから
分母にあるrの2乗で発散することなくゼロに収束する。
つまり、微小な球同士の接触では双方の万有引力の極限はゼロである。

こんな簡単なことも分からずに
笑かすなクズザルども。

なーーーーーーにがヒモ理論だよ
とっとと舌かんで氏ね詐欺ザルどもが。
今の理論物理学なるものは完全な虚構なんだよバーーーーーーカ

くっくっく
ご冗談でしょう?名無しさん [] 2018/03/19(月) 17:45:05.78:4vOnfCAV
お前らは人に与えられたデタラメなエサをくってるだけの
思考力のないサルにすぎん。

ちょっと考えればr=0で無限大に発散することなどありえず、
それは実際にこうして宇宙は存在してるのだから
あらゆるモノは密度で存在することが当たり前に示唆されてんのに
繰りこみ理論とかヒモ理論とか素粒子論もすべてが
極めて低能なサルによるデタラメにすぎんわ。

アホくさいサルどもが
くっくっく
ご冗談でしょう?名無しさん [sage] 2018/03/19(月) 18:03:07.70ID:???
自分の思考という小さい針の穴から天を覗くのはさぞ苦しかろう
ご冗談でしょう?名無しさん [] 2018/03/19(月) 18:11:23.12:4vOnfCAV
アサナガとかユカワとか
あんな妄想デタラメ論で脳減る症とか
腹の底から笑えるわ。

毎年毎年、あの症は物理学的には
本当に無価値で滑稽だ。

くっくっく
ご冗談でしょう?名無しさん [] 2018/03/19(月) 18:18:43.76:3qEtFfFg

シュレディンガー方程式ih∂ψ/∂t=Hψの偏微分方程式を形式的に変数分離して解く、則ちψ(x,y,z,t)=φ(x,y,z)χ(t)としてシュレディンガー方程式に代入せよ
則ちihφdχ/dt=χHφが従う
両辺をχφで割ると
ih/χ・dχ/dt=1/φ・Hφを得る
左辺はtのみの関数、右辺は空間座標のみの関数となるが両辺が等しいことより
ih/χ・dχ/dt=1/φ・Hφ=C(定数)
故にχ=exp(-i/hCt),Hφ=Cφ
Hφ=Cφは二階の微分方程式なので独立な解が2つ存在する
此れをφ_1,φ_2とすると、元のψは
ψ_1=exp(-i/hCt)φ_1,ψ_2=exp(-i/hCt)φ_2の2つの独立な解がある
シュレディンガー方程式は線形だから
重ね合わせψ=c_1ψ_1+c_2ψ_2も解になる

PS
実数Cは固有エネルギーEである
χの指数-i/hCtはらCがエネルギーの次元を持つとき、プランク定数はエネルギーかける時間の次元を持つことから、無次元になることを確認できる
ご冗談でしょう?名無しさん [sage] 2018/03/19(月) 20:57:09.17ID:???

>アサナガ
もしかしてトモナガ?
ご冗談でしょう?名無しさん [sage] 2018/03/19(月) 21:25:00.82ID:???

↑これが物理板の実力です
専門板なのに異常にレベルが低い
せいぜい数学の少しできる高校生レベル
ご冗談でしょう?名無しさん [sage] 2018/03/19(月) 21:33:12.37ID:???
一言「線型性」で済むところをグダグダ余計なこと垂れ流すあたりアホだよね
ご冗談でしょう?名無しさん [sage] 2018/03/19(月) 21:36:11.84ID:???

>Hφ=Cφは二階の微分方程式なので独立な解が2つ存在する

ここが致命的ですね
ご冗談でしょう?名無しさん [sage] 2018/03/19(月) 21:39:17.69ID:???
勝手に固有関数の線型結合に限定してるところも頭悪い
多分の問題と区別できてないんだろうな
ご冗談でしょう?名無しさん [sage] 2018/03/19(月) 21:42:27.23ID:???

その昔、橋下氏をあえて「ハシシタ」と表記した週刊アホ日(←これもか)の記事があったけど、
それと同類の、わざと誤読してあげつらう下品な芸だとは思うが、
くっくっくだから本気で間違えている可能性もなきにしもあらず
ご冗談でしょう?名無しさん [sage] 2018/03/19(月) 21:54:57.44ID:???
量子力学学んだばかりの学生の渾身の回答をフルボッコにしてやるなよ
ご冗談でしょう?名無しさん [sage] 2018/03/19(月) 21:57:19.95ID:???
解答する側も勉強になる
輪講みたいでいいスレじゃん
ご冗談でしょう?名無しさん [sage] 2018/03/19(月) 22:14:17.29ID:???
くっくっくみたいなのが交ざった輪講とかゴミでしかない
ご冗談でしょう?名無しさん [sage] 2018/03/19(月) 22:17:37.67ID:???
あれは輪に入ってないからノーカン
897 [] 2018/03/19(月) 22:43:16.61:wydgTs7Z
897です

すいません、ボコしてください
何処が間違っているのか教えてください
ご冗談でしょう?名無しさん [sage] 2018/03/19(月) 22:58:59.83ID:???

まずは自演した理由を教えていただけますか?
ご冗談でしょう?名無しさん [] 2018/03/19(月) 23:07:37.36:wydgTs7Z

質問した後に自分で考えて答えを書いてみました
これでは答えがあっていたら自演だと責められるだろうと思い別のIDを用いて答えを書き込みました
その後ご指摘があり、自分の解釈による答えに誤りがあると認識し、その誤りを詳細に知りたいがために回答したIDで先ほどのレスをしようと思いましたが、質問したIDでレスしてしまった次第です

申し訳ありません
ご冗談でしょう?名無しさん [sage] 2018/03/19(月) 23:18:27.32ID:???

別IDで書き込むことを自演と言うんですよ
同じIDで自分で考えたんで採点してくださいと言えばいいだけの話ですよね
ご冗談でしょう?名無しさん [sage] 2018/03/19(月) 23:24:23.66ID:???
過ぎたことを
ご冗談でしょう?名無しさん [] 2018/03/19(月) 23:51:13.05:wydgTs7Z
成る程、反省します

本題に戻ります

>なぜ固有函数を線型結合に限定しているか
Hφ=Eφに於いてハミルト二アンHはエルミート演算子であり、その固有函数は完全系を作るからだと思ったからです

勿論「同じ」固有エネルギーの解の線型結合でなくてもいいですね
一般解は
ψ=Σexp(-i/hE_nt)(cn1φn1+cn2φn2)
但しEnはn番目の固有エネルギー、φnはEnの固有函数

何か反論があればお願いします
ご冗談でしょう?名無しさん [sage] 2018/03/19(月) 23:55:04.31ID:???
本題に戻るとは式変形などではなくただのψの定義
ご冗談でしょう?名無しさん [sage] 2018/03/19(月) 23:56:15.28ID:???

∂f(x,y)/∂x=0
この方程式解いてみてください
ご冗談でしょう?名無しさん [sage] 2018/03/19(月) 23:56:46.52ID:???
だった
ご冗談でしょう?名無しさん [sage] 2018/03/20(火) 00:12:35.70ID:???
XとYの多項式をXで微分するのかニャン?f(y)=0?
ご冗談でしょう?名無しさん [] 2018/03/20(火) 00:33:13.20:/oxGZYep
質問失礼しますm(*_ _)m
この問題の(4)なのですが、小球は地面にぶつからずに貫通するというように考えて良いのでしょうか
aとbが衝突するのは必然のように思えて何かモヤモヤしてます…
Bは床と完全非弾性衝突するとか?

ttps://i.imgur.com/VQ9GHR0.jpg
ご冗談でしょう?名無しさん [sage] 2018/03/20(火) 00:34:14.38ID:???

882を見る限り、線形方程式だから、
解の空間が線形空間になる(和とスカラー倍で閉じている)っていうだけで、
それ以上のことは言っていない。

ちょっと質問の意図がわからないです。
そもそも文脈がわからない(AB効果か何か?)。


すいません、専門外のうえ長く離れているのでわかりません。
ちょっと適当なことを言っている人がいるので注意しておきますが、
くりこみは今現在ある程度の基礎付けはあって、ひも理論に頼らずとも、
「摂動的に」くりこみ不可能なアインシュタイン重力理論を、非摂動の範囲では
くりこみ可能にすることができる可能性というのは示唆されています。
(Weinbergのasymptotic safety scenarioとかで検索すると出てくるはず。)
ご冗談でしょう?名無しさん [sage] 2018/03/20(火) 02:06:05.66ID:???
世界に存在する全ての本を読んだらどうなるのでしょうか?
ご冗談でしょう?名無しさん [] 2018/03/20(火) 03:38:11.35:c87y4/da
>この問題の(4)なのですが、小球は地面にぶつからずに貫通するというように考えて良いのでしょうか

そう考えるしかないな。
「地面」ではなく空中の「基準面」とすべきであって、
これが入試問題だったら間違いなく炎上するであろうバカが作成した問題ってことだな。

くっくっく
ご冗談でしょう?名無しさん [sage] 2018/03/20(火) 09:52:14.30ID:???

小球Aが、ではなく、Bが最初に落下する前に、の書き間違いだろうな
ご冗談でしょう?名無しさん [sage] 2018/03/20(火) 13:39:45.42ID:???

くりこみって摂動計算でしか意味ないんじゃないの?
ご冗談でしょう?名無しさん [] 2018/03/20(火) 13:41:32.31:y/12MLC/
高校レベルの物理の質問です
発電所で作られた電気を一定の電力Pで送電するときは電圧Vをできるだけ大きくすることでP=IVの関係から電流Iが小さくなり
送電線で発生するジュール熱(=エネルギーの損失)を抑えられるとのことですが
Q=VIt=I^2Rt=V^2t/Rの式から見るにVをを大きくしたらQもどんどん大きくなりませんか?
Q=I^2Rtを見たらIが小さければQも小さいですが、他の2式を見るにVとQは比例してますしよく分かりません……
ご冗談でしょう?名無しさん [sage] 2018/03/20(火) 13:45:34.58ID:???
Q=VIt=V^2t/R
このVは抵抗にかかる電圧のことですね
P=VI
このVは電源の電圧です

これら2つは異なるものなんですね
抵抗にかかる電圧を求めるにはどうすれば良いかと言うと、やはり電流から求めるしかないわけです
ご冗談でしょう?名無しさん [sage] 2018/03/20(火) 14:00:59.66ID:???
世界に存在する全ての本を読んだらどうなるのでしょうか?
ご冗談でしょう?名無しさん [sage] 2018/03/20(火) 15:44:04.15ID:???
ヒマラヤのうんこ
ご冗談でしょう?名無しさん [] 2018/03/20(火) 17:14:20.50:y/12MLC/

無関係の式だったんですね…
調べたらなんか式も複雑っぽいので「送電線のロスを減らすにはVを上げる」と丸暗記しようと思います
ありがとうございます
ご冗談でしょう?名無しさん [sage] 2018/03/20(火) 18:13:39.45ID:???
V=IR
W=IV=V^2/R=I^2*R
R=1000で同じ電力1000ワットをI,Vを指定して送った時、消費電力Wが大きくなるか考えよう。
V=1000の時、消費電力Wは1000   (1000ワットおくるには、1000ワット余分に必要)
I=1000の時、消費電力Wは1000^3  (1000ワット送るには、1000^3ワットも余分に必要になる)

つまり、同じ電力を送ろうとしたときI,Vどちらが高くすれば、消費電力が小さくなるか明らかになるよね?
ご冗談でしょう?名無しさん [] 2018/03/20(火) 19:15:27.89:c87y4/da
P=V^2/Rな。

そりゃ、Vを大きくすれば当たり前にPも大きくなるが
6600ボルトで送電したものを電柱の変圧器で200や100ボルトに落として
家庭にある分電盤で200や100ボルトで使っている。

要は6600ボルトの高圧で送って電流を小さくし、
家庭では200や100ボルトで使っているから電流は大きい。
このとき、Vを6600ボルトで考えてはいかん。送電電圧をいくら大きくしたって
電柱の変圧器で降圧するから、Vを大きくすればPも大きくなるということはない。

送電線での電力損失をなくすために高圧で送って送電電流を
小さくしてんだよ。各家庭で流れている電流の総和ははるかに大きい。

くっくっく
ご冗談でしょう?名無しさん [sage] 2018/03/20(火) 19:41:49.24ID:???
くっくっく気持ち悪い
ご冗談でしょう?名無しさん [sage] 2018/03/20(火) 20:00:34.26ID:???

いいえ、ちゃんと意味があります。

くりこみ可能性の定義とは、
「有限個の裸の結合定数を、理論のカットオフに応じて微調整することで、
くりこまれた結合定数(相関関数)を一定に保ちながら、
カットオフを無限大に飛ばせる(連続極限をとれる)こと」です。

(ややこしい言い回しですが、これ以上シンプルには言えません)

特に、この操作が摂動論の各次数で行えるとき、
摂動的にくりこみ可能といいます。

くりこみ群で有名な K. G. Wilson は、
「裸の理論を、くりこみ群の臨界面(critical surface)に近づける」
という、非常に一般的な非摂動的連続極限の取り方を提案しています。

Wilsonの見方では、裸の理論が住んでいる理論空間と、
臨界面とが交わりをもつことを、くりこみ可能と呼ぶわけです。
ご冗談でしょう?名無しさん [sage] 2018/03/20(火) 20:05:31.96ID:???

全く説明になってない。
P=V^2/R
ならVが小さい方が効率が良いという結論になる。
損失のPを計算する場合のVは送電線での電圧降下が考慮すべき量であって
発電所の送電電圧ではない。
ご冗談でしょう?名無しさん [sage] 2018/03/20(火) 20:14:00.00ID:???
ムニちゃーんしんぷ
ご冗談でしょう?名無しさん [sage] 2018/03/20(火) 20:14:55.07ID:???

前提として、
1. Iは共通
2. 発電所から見てV0で同じ電力P0を送る
3. 送電ロスPtを減らしたい
つまり
P0=V0・I
Pt=I^2・Rt
だからI=V0/P0で
Pt/P0=((P0/V0)^2・Rt)/P0
=P0・Rt/V0^2
送電ロスは(1/V0)^2に比例して減るのね。
ご冗談でしょう?名無しさん [sage] 2018/03/20(火) 20:53:11.27ID:???
の訂正
誤 だからI=V0/P0で

正 だからI=P0/V0で
ご冗談でしょう?名無しさん [sage] 2018/03/20(火) 20:53:12.47ID:???
リーマン幾何学って高度な座標変換についての幾何学なわけじゃないですか、これで運動方程式を(相対論とは関係なく)座標変換すればラグランジュ力学とかいらなくならないですか?
ご冗談でしょう?名無しさん [sage] 2018/03/20(火) 20:58:10.33ID:???

そう思うなら自分でやってみて、その成果を発表して下さい。
迷ってる場合じゃありません。
ご冗談でしょう?名無しさん [sage] 2018/03/21(水) 00:11:05.66ID:???
相対論の問題です。
電車の車両に両隣の車両からそれぞれ1人ずつ人が入ってきた。
電車の中の人からすると、2人は同時に入ってきた。
電車を外から眺めていた人からすると、2人が車両に入った時刻にt秒の差があった。
電車が時速50kmで走行していて、車両の長さが20mとすると、tはいくらか、という問題の答えを教えてください。
ご冗談でしょう?名無しさん [sage] 2018/03/21(水) 01:21:08.21ID:???

列車の20mは静止長でOK?
ご冗談でしょう?名無しさん [sage] 2018/03/21(水) 01:29:17.98ID:???

20mが静止長ならこうかな?
x'=γ(x-βct)
ct'=γ(ct-βx)

dx'=γ(dx-βcdt)
cdt'=γ(cdt-βdx)

dt=0で
dx'=γdx
cdt'=-γβdx

cdt'=-βdx'
dt'=-(β/c)dx'
β=(50*10^3*60*60)/(3*10^8)
Δt=-(50*10^3*60*60)/(3*10^8)/(3*10^8)*20
=4.0*10^-8[s]
ご冗談でしょう?名無しさん [sage] 2018/03/21(水) 07:53:22.59ID:???

回答、ありがとうございます
やはり答えは(β/c)dxですよね
そうすると具体的には3e-15秒になると思うんですが(941の方の計算では電車の速さの時速→秒速の計算に単純ミスがあるようです)、『大栗先生の超弦理論入門』に30兆分の1秒≒3e-14秒と書いてあったので、計算方法を知りたくなりました
ご冗談でしょう?名無しさん [sage] 2018/03/21(水) 11:02:15.74ID:???

それが実行可能な場合は「くりこみ」なんて意識せず実行しそうだな
ご冗談でしょう?名無しさん [sage] 2018/03/21(水) 11:47:00.68ID:???

桁を間違えたんだろうね。3e-15秒が正しい。
ご冗談でしょう?名無しさん [sage] 2018/03/21(水) 12:59:11.27ID:???
ムニちゃんダンスをおどれ
ご冗談でしょう?名無しさん [sage] 2018/03/21(水) 14:46:40.01ID:???
東京大学理学部数学科卒 → 東京大学大学院数理科学研究科数理科学専攻修士課程修了 →
東京大学大学院数理科学研究科数理科学専攻博士課程修了

このルートを辿るには最低でも秀才以上じゃないと無理ですか?
ご冗談でしょう?名無しさん [sage] 2018/03/21(水) 15:19:42.26ID:???
ヒマラヤダンスをおどれ
ご冗談でしょう?名無しさん [sage] 2018/03/21(水) 17:39:20.05ID:???

シンプレクティック幾何知らないの?
ご冗談でしょう?名無しさん [sage] 2018/03/21(水) 21:19:35.12ID:???
的外れにボケで返すな
ご冗談でしょう?名無しさん [sage] 2018/03/21(水) 21:36:04.81ID:???
力学を微分位相幾何学方面で定式化したのがシンプレクティック幾何だろ。
が気が触れて接触構造なのを自覚しろよ。
ご冗談でしょう?名無しさん [sage] 2018/03/21(水) 23:51:08.92ID:???


すまん、*60*60じゃなくて/60/60だった www
ご冗談でしょう?名無しさん [sage] 2018/03/22(木) 01:16:20.81ID:???
ぷっちー ポポ ぷちぷらーい
ご冗談でしょう?名無しさん [] 2018/03/22(木) 03:30:15.16:MwS5Lopu
地球は太陽の周りをスッゲー速さで回ってるのに
宇宙に行ったスペースシャトルは
なんで地球に帰還できるの?
(´・ω・`)
ご冗談でしょう?名無しさん [sage] 2018/03/22(木) 05:42:32.66ID:???
走ってる電車で上にジャンプしてみろ
ご冗談でしょう?名無しさん [sage] 2018/03/22(木) 10:01:55.89ID:???
ムニちゃーん ポポ
ご冗談でしょう?名無しさん [sage] 2018/03/22(木) 11:44:01.36ID:???

ボケを解説すんな
ご冗談でしょう?名無しさん [sage] 2018/03/22(木) 13:58:27.32ID:???
最強の概念は何ですか?
ご冗談でしょう?名無しさん [sage] 2018/03/22(木) 13:59:21.85ID:???
頭にウンコを載せてみたら
ご冗談でしょう?名無しさん [sage] 2018/03/22(木) 14:04:24.29ID:???
東京大学大学院数理科学研究科の博士課程入学試験と宇宙飛行士選抜試験ってどっちの方がムズイ?
ご冗談でしょう?名無しさん [sage] 2018/03/23(金) 00:15:56.47ID:???
ムーニーちゃんしんぷとセバスチャンしんぷはどっちの方が凄いですか?
ご冗談でしょう?名無しさん [sage] 2018/03/23(金) 00:49:42.06ID:???

強烈な材質ができて、新幹線がスケルトンになったら怖いと思うだろうな www
ご冗談でしょう?名無しさん [sage] 2018/03/23(金) 00:56:10.36ID:???
手が机をすり抜ける確率は0じゃないって聞くけどあれ嘘だよね
ご冗談でしょう?名無しさん [sage] 2018/03/23(金) 01:03:45.23ID:???
多分0じゃないとは思うけどもな。
どれくらいなのか俺っちには計算できないけど、なんとなくグラハム数の逆数よりは大きい程度じゃないの?
ご冗談でしょう?名無しさん [sage] 2018/03/23(金) 01:13:34.05ID:???
キミの体が核爆発を起こす確率が0じゃないのと同じ
ご冗談でしょう?名無しさん [sage] 2018/03/23(金) 01:13:43.01ID:???
とりあえず0じゃないよって言っておけば量子力学ってすげーなるほどーみたいな反応をさせられているような気がしてならない
ご冗談でしょう?名無しさん [sage] 2018/03/23(金) 01:15:30.17ID:???
もしかして1つの水素分子が自発的に2つの水素原子に分かれる確率みたいなもん?
ご冗談でしょう?名無しさん [sage] 2018/03/23(金) 01:22:28.91ID:???

一般に個別の理論の連続極限をとる問題は非自明なので、
意識せずくりこむことはありえない。

どのように結合定数(counter term)を選ぶか、どのように微調整するかを決定する必要があるから。

摂動論の範囲ですら、くりこみ可能性の証明はめっちゃ大変。
梅沢先生とかが最初にやったはずだけど、森公式、収束定理、
いずれも計算を追うだけで時間掛かった記憶がある。
ご冗談でしょう?名無しさん [sage] 2018/03/23(金) 01:24:50.99ID:???
一年以内に北朝鮮の核ミサイルが飛んでくる確率でも考えろ
ご冗談でしょう?名無しさん [sage] 2018/03/23(金) 01:41:51.51ID:???
いじわる
ご冗談でしょう?名無しさん [sage] 2018/03/23(金) 03:22:06.50ID:???
宇宙論的にそもそも宇宙が始まったこと自体がトンネル効果のせいだってりろ理論もあるけどな。
陽子崩壊を観測する名目でニュートリノ天文学おっぱじめるような話もあるので気長に観測でもしてたら?。宇宙波動関数のユニタリ変換を延々と。
ご冗談でしょう?名無しさん [sage] 2018/03/23(金) 06:54:10.87ID:???
親切な人!ありがとう
ご冗談でしょう?名無しさん [sage] 2018/03/23(金) 09:34:37.64ID:???
・手が机をすり抜けた後の状態にある確率
・手が机をすり抜ける途中状態にある確率
前者が厳密には0でないのは量子力学ハッタリ雑学でよくある話だけど
後者も0ではないよね? これは *いしのなかにいる* みたいな状態として観測されるの?
ご冗談でしょう?名無しさん [sage] 2018/03/23(金) 09:37:09.89ID:???
春の夢
ご冗談でしょう?名無しさん [sage] 2018/03/23(金) 12:59:30.12ID:???
手が無事に机をすり抜ける確率より手の原子がバラバラになる確率の方が遥かに大きい
ご冗談でしょう?名無しさん [sage] 2018/03/23(金) 13:44:27.17ID:???
「全」ってのは正確に言うと、「「有」の全て」ってことなのでしょうか?
ご冗談でしょう?名無しさん [sage] 2018/03/23(金) 17:51:10.29ID:???
ムーニーちゃんしんぷだ〜い
ご冗談でしょう?名無しさん [sage] 2018/03/23(金) 18:49:53.49ID:???
自殺をしたら地獄に落ちるというのは本当なのでしょうか?
ご冗談でしょう?名無しさん [] 2018/03/23(金) 20:59:50.48:0FZ7dcqK
エネルギー解析物質、エネルギーを物質に変換、エネルギーフォトジェニック変換、エネルギー錬成、エネルギー貴世石変換、エネルギー物質変換で、食べ物を作る。

上記 登記

ソーラーエネルギーで、蛍光灯など光に変換して、植物を育てる方が早い。イゼルローン要塞や、プラント要塞、食料プラントなどの方が安全で安い。

上記 登記
ご冗談でしょう?名無しさん [sage] 2018/03/23(金) 21:40:53.80ID:???
嵐コメが続いてるようでレス番が飛んでるw
ご冗談でしょう?名無しさん [] 2018/03/23(金) 22:29:19.00:bXu4GGJh
【UFO】 この事件が起きたのは、ライト兄弟が初飛行に成功した1903年より、21年前のことです
ttp://rosie.5ch.net/test/read.cgi/liveplus/1521806322/l50
ご冗談でしょう?名無しさん [sage] 2018/03/24(土) 00:25:57.17ID:???
週末にはゴミが増える
ご冗談でしょう?名無しさん [sage] 2018/03/24(土) 02:50:50.67ID:???
ちがわい ちがわい
ご冗談でしょう?名無しさん [sage] 2018/03/24(土) 08:34:54.93ID:???
ムーニーちゃん、うんこ
ご冗談でしょう?名無しさん [] 2018/03/24(土) 12:23:27.45:ui6ljtpi
最強の食料自給船は、イゼルローン型ナデシコ

最強の食料自給要塞は、この時代は、イゼルローン型ナデシコ

上記 登記
ご冗談でしょう?名無しさん [sage] 2018/03/24(土) 13:05:57.74ID:???
ほらゴミだ
ご冗談でしょう?名無しさん [sage] 2018/03/24(土) 13:14:57.39ID:???
ここがゴミ箱だと知っての発言ならいまさらだな
ご冗談でしょう?名無しさん [sage] 2018/03/24(土) 14:24:12.28ID:???
超絶論破人間になりたいのですが、やはり数学の全分野を究めるのが近道ですか?
ご冗談でしょう?名無しさん [sage] 2018/03/24(土) 15:00:40.20ID:???
最強の概念は「無」ですか?
ご冗談でしょう?名無しさん [] 2018/03/24(土) 15:10:42.36:VYaebc+z
よろしくお願いいたします。

>>ttps://ocw.kyoto-u.ac.jp/ja/general-education-jp/basic-physical-chemistrya/pdf/lect10b.pdf

これのθ2積分がわかりません。
∫(0→π)sinθ2/√r1^2+r2^2*-2r1r2cosθ2 dθ2

やりかたでもいいので教えて頂けてますでしょうか。
ご冗談でしょう?名無しさん [sage] 2018/03/24(土) 15:53:09.75ID:???

t = cos(θ_2)
とでも置いて置換積分すれば
ご冗談でしょう?名無しさん [] 2018/03/24(土) 16:34:57.72:VYaebc+z
恥ずかしながら、その後どうすればいいんでしょうか。

t = cos(θ_2) dt=-sin(θ_2)dθ_2

-∫(0→π)dt(r_1^2+r_2^2-2r1r2t)^-1/2

でしょうか?その後どうすればいいでしょうか。
ご冗談でしょう?名無しさん [] 2018/03/24(土) 16:41:16.54:VYaebc+z
解決しました。ありがとうございました。
数学に全然自信がないうえ、忘れたことが多すぎです。
ありがとうございました。
ご冗談でしょう?名無しさん [sage] 2018/03/24(土) 23:07:23.63ID:???

初等関数で解けるものしか載っていないのが教科書だね〜
ご冗談でしょう?名無しさん [sage] 2018/03/25(日) 00:16:59.23ID:???
■ちょっとした物理の質問はここに書いてね224■
ttps://rio2016.5ch.net/test/read.cgi/sci/1521904578/
ご冗談でしょう?名無しさん [sage] 2018/03/25(日) 02:51:14.16ID:???
でぼぼぼーん
ご冗談でしょう?名無しさん [sage] 2018/03/25(日) 12:34:28.88ID:???

オマエと違ってゴミ箱なんて思ってねーぜ
週末にゴミが増えると知ってるだけだ
ご冗談でしょう?名無しさん [] 2018/03/26(月) 13:06:34.75:8e7ks0UK
理学部物理学科はバチカン市国なので工学部の応用物理工学科に進学するべきだ。
ご冗談でしょう?名無しさん [sage] 2018/03/27(火) 12:42:52.21ID:???
理学部に劣等感か

勢い5万以上のスレをメールでお知らせするサービス、実施中!!
憧れボディをGETしたい!その夢、ボニックで!

新着レスの表示

2ch勢いランキング アーカイブ 物理板ランキング

凡例:

レス番

100 (赤) → 2つ以上レスが付いている
100 (紫) → 1つ以上レスが付いている

名前

名無しさん (青) → sage のレス
名無しさん (緑) → age のレス

ID

ID:xxxxxxx (赤) → 発言が3つ以上のID
ID:xxxxxxx (青) → 発言が2つ以上のID

このページは2ch勢いランキングが作成したアーカイブです。削除についてはこちら